Med Surg III Final Packet 1 of 2

Pataasin ang iyong marka sa homework at exams ngayon gamit ang Quizwiz!

If aortic arch dissection - neurologic deficits such as _?_ may occur:

(altered LOC, weakened or absent carotid/temporal pulses, dizziness, syncope

Etiology and Pathophysiology of acute pericarditis: Noninfectios (6)

(uremia, acute MI, neoplasms, trauma, radiation, dissecting aortic aneurysm)

Treatment for head/neck cancer may include: _?_-involves removal of 1 vocal cord or part of a cord, requires a temporary tracheostomy.

*Hemilaryngectomy

_?_ is a spectrum of clinical abnormalities caused by excess corticosteroids, particularly glucocorticoids

.Cushing syndrome:

Normal lipase is from _?_ to _?_ Normal amylase is from _?_ to _?_

0 to 160 0 to 130

What amount qualifies as "adequate" urine output

0.5 -1 ml/kg/hr. If it is less, add more fluid

On the Glasgow Coma Scale, "Eyes Opening" ranges from _?_ to _?_

1 - none 2- to pain 3. - to voice 4 - spontaneoulsy

On the Glasgow Coma Scale, "Motor Response" ranges from _?_ to _?_

1 - none 2. abnormal extension 3 - abnormal flexion 4. withdraws from pain 5 - localizes to pain 6 - obeys commands

On the Glasgow Coma Scale, "Verbal Response" ranges from _?_ to _?_

1 - none 2. incomprehensible sounds 3. - inappropriate words 4 - confused 5 - oriented

Name 5 important assessments post-CABG

1 airway/pulmonary care 2. HR (pre-load, afterload) 3. Chest tube patency 4. Neuro assessment 5. bowel sounds

After AKI, Renal healing and function may take up to _?_

1 year.

Collaborative intervention:-- #4 -- (Hemothorax) 1. 2. 3. 4.

1. ABC 2. Hypovolemic shock -- give fluid/blood through large bore 3. 100% NRB O2 4. chest tube

Treatment of pleural effusion: 1. 2. ID and treat underlying causes 3. Chest tube or _?_ 4. Chest X-ray

1. ABC's 3. thoracentesis

Chest tube is attached to a drainage device. 3 characteristics:

1. Allows air and fluid to leave the chest 2. Contains a one-way valve to prevent air & fluid returning to the chest 3. Designed so that the device is below the level of the chest tube for gravity drainage

Name 4 cancer promoting factors:

1. Dietary fat 2. Obesity 3. Cigarette smoking 4. Alcohol consumption

2 "classes" of Disorders of the Anterior Pituitary Gland

1. Hyperpituitarism 2. Hypopituitarism

List 7 complications of ARDS:

1. MODS 2. Sepsis 3. VAP 4. Barotrauma 5. Volutrauma 6. Stress Ulcers 7. Renal failure

5 Treatments for Cushing syndrome

1. Monitor V/S, 2. fluid and electrolyte status, 3. Monitor for & prevent infection; 4. Prevent injury - avoid exposure to extreme temperatures and stress; 5. Emotional support (due to changes in appearance)

8 Clinical Manifestations of HHS

1. Polyuria 2. Polydipsia 3. Blurred vision 4. H/A 5. Orthostatic hypotension 6. Change inLOC 7. Seizures 8. Reversible paralysis

Cancer is normally an orderly process with three stages which are:

1. initiation 2. promotion 3. progression

4 'umbrella" complications post-CABG

1. neuro complications 2. post - op bleeding 3. pneumonia 4. cardiac tamponade

Hyperparathyroidism increases the risks of 8 things...

1. stroke 2. HF 3. A fib 4. Cardiomyopathy 5. renal failure 6. shingles 7. Bone fx/pain 8. HTN

What is normal specific gravity of urine?

1.010 to 1.025

In the kidney donor, we want creatinine to stay below _?_

1.4

What is a normal RAP? What other measurment corresponds to RAP?

2-8 CVP

The nurse is caring for a patient who suffered an inhalation injury from a wood stove. The carbon monoxide level is 12%. The nurse would anticipate which of the following signs and symptoms? 1. coma 2. flushing 3. dizziness 4. tachycardia

2. flushing

Etiology/Patho of Bowel Obstruction: 1. Distention d/t decreased absorption of fluids and electrolytes (F/E's) and stimulates intestinal secretions 2. increased pressure in the _?_ (as the fluid increases) which impairs capillary permeability & causes leakage of fluid and electrolytes into the peritoneal cavity 3. leads to decrease in _?_which 4. leads to _?_ 5. which leads to decreased blood flow to bowel 6. which leads to ischemia, necrosis, bowel perforation.

2. lumen of the bowel 3. circulating blood volume 4. hypotension/hypovolemic shock

What is normal CI

2.2-4

What is normal PAS?

20-30; pulmonary artery systolic pressure

What is the lowest Glasgow Coma Scale score? Highest?

3 15

Greater than _?_ in diameter is considered to be an abdominal aortic aneurysm (AAA)- the greater the diameter, the greater the risk for rupture

3 cm.

Client Education for HHS and DKA: 1. provide the patient with education to prevent recurrence; 2. encourage all to wear a medical alert bracelet 3. take measures to decrease dehydration - drink at least_?_unless contraindicated 4. monitor glucose _?_ when ill, and continue to take insulin 5. consume liquids with carbohydrates and electrolytes when unable to eat solid foods

3 liters of water/day every 4 hours

When drawing blood for serum lipase and amylase, draw _?_ to _?_ ml; use the tube with the _?_ top.

3 to 5 ml red

Collaborative Intervention: Chest injury #2: (Open Pneumo) ________________ ________________

3-sided dressing; chest tube

Practice Question: A patient is burned over the entire face, back of the head, upper half of the anterior torso, circumferential burns to the lower half of both arms, & upper half of the posterior torso. Using the rule of 9's, what would be the extent of the burn injury? 1. 18% 2. 24% 3. 36% 4. 48%

3. 36%

The nurse is assessing the functioning of a chest tube drainage system in a client who has just returned from the recovery room following a thoracotomy with wedge resection. Which are the expected assessment findings? Select all that apply. 1. Excessive bubbling in the water seal chamber 2. Vigorous bubbling in the suction control chamber 3. Drainage system maintained below the client's chest 4. 50 ml of drainage in the drainage collection chamber 5. Occlusive dressing in place over the chest tube insertion site 6. Fluctuation of water in the tube in the water seal chamber during inhalation and exhalation

3. Drainage system maintained below the client's chest 4. 50 ml of drainage in the drainage collection chamber 5. Occlusive dressing in place over the chest tube insertion site 6. Fluctuation of water in the tube in the water seal chamber during inhalation and exhalation

After 3 weeks of radiation therapy, a patient has lost 10 pounds and does not eat well because of mucositis. An appropriate nursing diagnosis for the patient is: 1. Risk for infection related to poor nutrition. 2. Ineffective self-health management related to refusal to eat. 3. Imbalanced nutrition: Less than body requirements related to oral inflammation and ulceration. 4. Ineffective health maintenance related to lack of knowledge of nutritional requirements during radiation therapy.

3. Imbalanced nutrition: Less than body requirements related to oral inflammation and ulceration

The nurse determines that the patient in shock has progressed beyond the compensated stage when laboratory tests reveal: 1. Increased blood glucose levels. 2. Increased serum sodium levels. 3. Increased serum potassium levels. 4. Increased serum calcium levels.

3. Increased serum potassium levels.

During assessment of a patient with a spinal cord injury at the level of T2 at the rehabilitation center, which of the following findings would concern the nurse the most? 1. A heart rate of 92 2. A reddened area over the patient's coccyx 3. Marked perspiration on the patient's face and arms 4. A light inspiratory wheeze on auscultation of the lungs

3. Marked perspiration on the patient's face and arms

With IICP, HOB is at _?_ unless contraindicated. Patient should not be sitting at 90 degrees because _?_ and their head should be aligned.

30% 90 degrees will increase abd pressure which will increase ICP

Normal insensible loss is _?_ ml/hr. Burns GREATLY increase this amount

30-50

While on dialysis, B/P must be taken every_?_ minutes, or more frequently if indicated

30-60 (**Usually every 15 minutes)

Surviving Sepsis Campaign: Fluid resuscitation for sepsis-induced hypoperfusion, at least _?_of intravenous _?_ fluid be given within the first _?_ hours; target MAP _?_ mmHg Following initial fluid resuscitation, additional fluids guided by frequent reassessment of hemodynamic status. _?_ in addition to crystalloids when patients require substantial amounts of crystalloids

30ml/kg crystalloid 3 65 Albumin

After an aneurysm repair, call the MD for any fever that is _?_ or above. Also call if there is any drainage, redness, etc.

37.8

Perioteneal dialysis may require up to _?_ exchanges per day; Fluids should be _?_

4 warmed to body temp

What is the fluid therapy goal for the first 24 hours post burn per Parkland/Baxter formula?

4 ml LR/kg/%TBSA.

What is normal PAD

4-12; pulmonary artery diastolic pressure

What is normal CO

4-8

When assessing a patient in shock, the nurse recognizes that the hemodynamics of shock include: 1. Normal cardiac output in cardiogenic shock. 2. Increase in central venous pressure in hypovolemic shock. 3. Increase in systemic vascular resistance in all types of shock. 4. Variations in cardiac output and decreased systemic vascular resistance in septic shock.

4. Variations in cardiac output and decreased systemic vascular resistance in septic shock.

The nurse is caring for a patient who sustained superficial partial-thickness burns on the anterior lower legs and anterior thorax. Which of the following does the nurse expect to note during the resuscitation/emergent phase of the burn injury? 1. decreased heart rate 2. increased urinary output 3. increased blood pressure 4. elevated hematocrit levels

4. elevated hematocrit levels

The nurse is assisting a health care provider with the removal of chest tube. The nurse should instruct the client to take which action? 1. exhale slowly 2. stay very still 3. inhale and exhale quickly 4. perform the valsalva maneuver

4. perform the valsalva maneuver

Post-aneurysm surgery Limit HOB to _?_to prevent flexion of graft

45 degrees

Malnutrition in cancer patients: Fat and muscle depletion Supplements when _?_weight loss is noted Patient has potential for protein and calorie malnutrition Monitor total protein, albumin and prealbumin levels. Enteral or parenteral nutrition may be required

5%

What is normal ICP

5-15 mm Hg (worry if > 20)

Collaborative Care for aneurysm - Surgical repair - Recommended with asymptomatic aneurysms _?_or larger

5.5 cm in diameter

suspect HIT if _?_% or more reduction in platelets OR platelet count < _?_

50 150,000

What is normal HR

60-100

What is normal mixed venous Hgb

60-80

According to the American Spinal Injury Association (ASIA) Impairment Scale, _?_ is the worst classification while _?_ is the best

A E

What is Acute Kidney Injury (AKI)?

A term used to encompass the entire range of the syndrome including a very slight deterioration in kidney function to severe impairment

What are the 2 type of tissue typing that must be done prior to transplant? Some organ transplants require greater histocompatibility match than other organs. Corneas, for example can be accepted by nearly any individual because corneas are avascular, and no antibodies reach the cornea to cause rejection

ABO blood (do not need same Rh factor) HLA (human leukocyte antigen) typing

_?_ should be started within the first 24 hours post-sTEMI. These drugs help prevent remodeling and HF.

ACE

5 lab tests for Hep are: Plus: HAV & HBV (antibodies are _?__) HCV: (antibodies indicate _?_) May have liver biopsy

ALT, AST, GGT, Alkaline phosphatase (ALP), serum bilirubin HAV & HBV (antibodies are protective) HCV: (antibodies indicate chronic disease)

The following are predisposing conditions to _?_, Anaphylaxis Disseminated intravascular coagulation (DIC) Nonpulmonary systemic diseases Opioid overdose (Heroin) Severe head injury Shock states Transfusion-related acute lung injury

ARDS

The following are predisposing conditions to _?_, Aspiration of gastric contents Viral/bacterial PNA Sepsis Trauma

ARDS

The following are predisposing conditions to _?_, Chest trauma Embolism Near-drowning O2 toxicity Radiation PNA Acute pancreatitis

ARDS

_?_ is a sudden and progressive type of acute respiratory failure in which the alveolar-capillary interface is damaged

ARDS

Damage to alveoli, decreased surfactant, and decreased alveolar complicance are characteristics of _?_

ARDS (Acute Respiratory Distress Syndrome)

Sepsis clinical criteria infection + change in "SOFA" (sepsis related organ failure assessment) greater than or equal to 2. 6 criteria are:

ARDS (decreased paO2/FiO2) decreased GCS hypotension or vasopressors increased bilirubin decreased platelets Increased creatinine/oliguria

_?_ is a sudden and progressive form of acute respiratory failure in which the alveolar capillary membrane becomes damaged and more permeable to _?_

ARDS: intravascular fluid

Liver DIagnostic tests: What are the normal lab values for the following? AST ALT LDH (elevated in alcohol abuse) Serum Ammonia

AST 10-30 ALT 10-40 LDH < 200 Ammonia 15-45 mcg/dl

Cervical and thoracic injuries cause paralysis of _?_ and/or __?_ Patient cannot cough effectively Leads to _?_ or _?_ Artificial airway provides direct access for pathogens Important to ↓ infections _?_pulmonary edema may occur Pulmonary edema may occur in response to fluid overload

Abdominal muscles Intercostal muscles; atelectasis or pneumonia Neurogenic

The patient is taking lansoprazole (Prevacid) for chronic management of peptic ulcer disease. The nurse advises the patient to take which of these over the counter medications if needed for a headache? Naproxen (Aleve) Ibuprofen (Motrin) Acetaminophen (Tylenol) Acetylsalicylic (Aspirin)

Acetaminophen (Tylenol)

5 disorders of the anterior pituitary related to overproduction:

Acromegaly (due to ↑GH when adult) Gigantism (due to↑GH when child) Cushing syndrome (due to↑ACTH) Hyperthyroidism (due to↑TSH) Prolactinomas (due to ↑prolactin)

5 disorders of the anterior pituitary related to overproduction: Acromegaly (due to _?_ when adult) Gigantism (due to_?_ when child) Cushing syndrome (due to_?_ ) Hyperthyroidism (due t_?_ ) Prolactinomas (due to _?_ )

Acromegaly (due to ↑GH when adult) Gigantism (due to↑GH when child) Cushing syndrome (due to↑ACTH) Hyperthyroidism (due to↑TSH) Prolactinomas (due to ↑prolactin)

_?_ - Signs within 48 hours of injury Similar signs and symptoms to brain tissue compression in increased ICP Patient can appear drowsy, confused, or unconscious Ipsilateral pupil dilates and becomes fixed with significant elevations in ICP

Acute Subdural Hematoma

How do you distinguish between MI pain and acute pericarditis pain?

Acute pericarditis pain is worse with inspiration and worse when lying supine

_?_ occurs within first 6 months after transplantation; it is mediated by patient's lymphocytes, which have been activated against the donated (foreign) tissue or organ Usually reversible with additional _?_therapy, which may include _?_ or _?_

Acute rejection immunosuppressive corticosteroid doses or polyclonal or monoclonal antibodies.

1. 2. Assess for other injuries Control bleeding Obtain cervical spine x-rays, CT scan, or MRI Prepare for stabilization with cranial tongs or traction Monitor resp status, vital signs, level of consciousness, oxygens sat, cardiac rhythm, urine output Keep warm Monitor urinary retention, hypertension

Administer O2/mechanically vent 2 large bore IV lines

Adrenal Cortex Hypofunction aka _?_

Adrenocortical insufficiency

_?_ is a hormone that is secreted by the anterior pituitary. It stimulates the adrenal glands to release corticosteroids

Adrenocorticotropic Hormone (ACTH)

8 Non-traumatic causes of rhabdomyolysis:

Alcohol/drug use such as heroin/cocaine, Extreme muscle strain (exercise), Prescription drugs: antipsychotics, statins, Heat stroke/high body temp, Seizures or delirium tremens (DTs), Diabetic ketoacidosis, Viral infections such as influenza, Sepsis, Previous hx of rhabdo

_?_ is a mechanical device that helps the heart pump blood. This device is inserted into the aorta, the body's largest artery. It is a long, thin tube called a catheter with a balloon on the end of it

An intra-aortic balloon pump (IABP)

Drug Therapy for Burns: _?_ and _?_ - IV is preferred route _?_due to likelihood of anaerobic burn wound contamination. Antimicrobial agents _?_ prophylaxis

Analgesics and sedatives; Tetanus immunization DVT

What are 6 s/e of monoclonal antibodies such as muromonab-CD3 (Orthocione OKT3)?

Anaphylaxis Fever Chills Nausea Vomiting Dyspnea Chest Pain

The patient with chronic gastritis is being put on a combination of medications to eradicate H. pylori. Which drugs does the nurse know will probably be used for this patient? Antibiotic(s), antacid, and corticosteroid Antibiotic(s), aspirin, and antiulcer/protectant Antibiotic(s), proton pump inhibitor, and bismuth Antibiotic(s) and nonsteroidal antiinflammatory drugs (NSAIDs)

Antibiotic(s), proton pump inhibitor, and bismuth

2 hormones secreted by the posterior pituitary

Antidiuretic hormone (ADH) Oxytocin

_?_ Results in obstruction of flow from left ventricle to aorta during systole

Aortic stenosis

_?_ - produce specific antibodies that bind to tumor cells; can often be detected in serum and saliva

B cells

How do the catecholamines work?

Binds to adrenergic receptor on cells -

_?_ is a spinal cord syndrome which results in ipsilateral loss of motor function, position, vibratory sense and vasomotor paralysis or contralateral loss of pain/temperature sensation below the level of injury

Brown-Sequard syndrome

_?_ is useful as tumor markers to monitor effectiveness of therapy or indicate tumor recurrence

Carcinoembryonic antigen (CEA)

Neurological manifestations of DIC include 2 things

Changes in LOC, Coma

_?_ occurs over months or years; Is irreversible and can occur for unknown reasons or from repeated episodes of acute rejection Results in _?_ and _?_ of the organ. No definitive treatment, rather supportive --- poor prognosis

Chronic rejection fibrosis and scarring

Post-hyperparathyroidism surgery: Assess _?_ and _?_

Chvostek's (facial muscles) & Trousseau's signs (carpal spasms)

_?_ is a type of blunt trauma caused by impact of parts of the body against other objects

Contrecoup trauma:

_?_ - is an excision into the cranium to cut away bone flap

Craniectomy

_?_is caused by a pituitary gland tumor (usually benign) that over-secretes the hormone ACTH, thus overstimulating the adrenal glands' cortisol production

Cushing disease

_?_ posturing occurs because of injury to voluntary motor tracks

Decorticate posturing (flexor)

_?_: resection of part of esophagus and anastomosis of segment of colon to remaining part

Esophagoenterostomy

_?_: resection of part of esophagus and anastomosis of remaining portion to stomach

Esophagogastrostomy

3 parts of the Upper GI

Esophagus Stomach Small Intestines

Who must you screen for Viagra use prior to giving Nitro? Why

Everyone; because it is also used for P HTN

3 nursing dx for valve problems

Excess fluid volume Decreased cardiac output Ineffective self-health management

_?_: head is suddenly and forcefully accelerated forward, causing extreme flexion of neck.

Flexion injury

_?_ jaundice causes: blood transfusion reactions, sickle cell crisis, hemolytic anemia

Hemolytic

Hep _?_ is blood-borne and there is no treatment

Hep G

S/S of small pneumothorax (2)

If small: mild tachycardia & dyspnea

ABC's- Immediate post injury problems include Maintaining a patent airway Adequate ventilation Adequate circulating blood volume to maintain adequate blood pressure _?_-Preventing extension of cord damage (secondary damage) Stabilize cervical spine Must log roll patient till spine cleared to prevent further trauma

Immobilization

A massive gastrointestinal bleed has resulted in hypovolemic shock in an older patient. What is a priority nursing diagnosis? Acute pain Impaired tissue integrity Decreased cardiac output Ineffective tissue perfusion

Ineffective tissue perfusion

Mutations can be _?_ or _?_

Inherited mutation Acquired mutation

_?_ may cause capillary leak syndrome

Interleukins

How does CO2 affect cerebral blood flow?

It causes vasodilation which will increase ICP d/t increased blood volume

Musculoskeletal manifestations of DIC include 2 things

Joint/bone pain

Nursing Management of Aortic Dissection POST -OP: care similar to OAR -- include patient and caregivers

Long term medical care for HR and BP control - need antihypertensives for life - need to understand drug regimen and side effects (dizziness, depression, fatigue, erectile dysfunction) - discuss side effects with MD before d/c med; follow up CTs and MRIs regularly scheduled

Clinical manifestations of hepatitis classified into 2 phases: Acute & Chronic 4 Chronic S/S

Malaise Easy fatigability Hepatomegaly Elevated liver enzymes (AST, ALT)

_?_ occurs when the pressure gets so high that it pushes the heart and great vessels into the unaffected side of the chest These structures are compressed from external pressure and cannot expand to accept blood flow

Mediastinal shift

What are 2 additional management techniques for Mitral Regurg (besides managing with medication)?

Mitral Annuloplasty (Ring repair) Cardiac resynchronization therapy (BiV Pacemaker/AICD)

_?_ Abnormality of mitral valve leaflets and the papillary muscle or chordae; Allows the leaflets to prolapse back into the left atrium during systole; Unknown cause

Mitral Valve Prolapse:

_?_ - Majority of adult cases result from rheumatic heart disease. Causes scarring of valve leaflets and chordae tendineae _?_ develop with adhesions between commissures of the leaflets.

Mitral Valve Stenosis Contractures

List the 5 valvular dysfunctions, in decreasing order of occurrence:

Mitral stenosis Mitral regurgitation Mitral valve prolapse Aortic stenosis Aortic regurgitation

3 effects of cancer treatment on Bone Marrow Suppression:

Neutropenia (more w/ chemo); thrombocytopenia (all ); anemia (all)

Name 2 CCB's used to treat P HTN. What is their mechanism of action?

Nifedipine, dilitazem Cause smooth muscle dilation; lower PAP (Pulmonary artery pressure)

Acromegaly:

Occurs when sustained GH secretion begins during adulthood. Bones increase in thickness and width. Not common - one out of 3 million adults diagnosed each year.

Kidney manifestations of DIC include 2 things

Oliguria, failure

Severe hypomagnesium leads to suppression of_?_

PTH secretion

_?_ bind to DNA and RNA, miscoding information &/or inhibiting DNA replication, causes cell death. An example is cisplatin (Platinol-AQ) with s/e of ototoxicity, nephrotoxicity, neurotoxicity and peripheral neuropathy which may not resolve with cessation of drug

Platinum Drugs

_?_ is often present in cor pulmonale (secondary to COPD)

Polycythemia

In _?_ the peak inspiratory pressure is predetermined and the Vt delivered to the patient varies.

Pressure ventilation

_?_return after resolution of spinal shock; may complicate rehabilitation

Reflexes

_?_is caused by the breakdown of skeletal muscle

Rhabdomyolysis

4 causes of Chronic aortic regurgitation:

Rheumatic heart disease Congenital bicuspid aortic valve Syphilis Chronic rheumatic heart conditions

What is the most common cause of ARDS?

Sepsis

_?_ is when blood exits the heart w/o participating in gas exchange. What are the 2 types?

Shunting; anatomic and intrapulmonary

Types of Bowel Obstruction: _?_ is a blockage of the lumen of the intestine in one spot.

Simple Obstruction

_?_ is a cancer that forms in squamous cells, which are epithelial cells that lie just beneath the outer surface of the skin. These cells also line many other organs, including the stomach, intestines, lungs, bladder, and kidneys. These cells look flat, like fish scales, when viewed under a microscope; sometimes called epidermoid carcinomas

Squamous cell carcinoma

4 causes of open pneumothorax

Stab wound gunshot wounds Thoracotomies Accidental chest tube removal

The American College of Cardiology Foundation (ACCF) and American Heart Association (AHA) classify heart failure as Stage A through Stage D. Stage A is characterized by _?_ while Stage D is characterized by _?_

Stage A - patient at high risk for HF d/t other conditions/symptoms Stage D - patients with refractory HF and severe symptoms at rest

_?_ are like a scaffold that hold open vessels. What are 2 types?

Stents; Bare metal Drug eluting

Labs for Bowel Obstruction: CBC - an increase in WBC may indicate _?_ or _?_ Serum Electrolytes - Obstruction in _?_ causes quick dehydration Amylase BUN (Blood Urea Nitrogen)/Creatinine Hemoccult Stool- check for occult blood

Strangulation Perforation Small Bowel

IE Causative Organisms: Most common causative organisms are _?_ and _?_. Two other causes are _?_ and _?_

Streptococcus viridans Staphylococcus aureus Viruses Fungi

_?_- Occurs within 2 to 14 days of injury After initial bleeding, these structures may appear to enlarge overtime

Subacute Subdural Hematoma

Name 3 obstructive oncologic emergencies:

Superior vena cava syndrome Spinal cord compression Third space syndrome

What do each of the following stand for in the TNM Classification T N M

T - tumor size and invasiveness N - presence/absence of spread to lymph nodes M - metastasis to different locations

Paraplegia(total or partial) involves a spinal cord injury in areas _?_ through _?_ and _?_ through _?_

T1-T12 L1 - L4

_?_ is a type of cancer treatment which interferes with cancer growth by "targeting" specific cellular receptors and pathways important for tumor growth

Targeted Therapy

Complications and Management after Thoracic Surgery: _?_ and tracheal shift Infection

Tension pneumothorax

Some cases of sudden cardiac death (SCD) involve disturbances in _?_.

The conduction system (such as prolonged QT syndrome and Wolff-Parkinson-White syndrome)

Sensitivity

The degree of negative pressure created by the patient's attempt to breathe that will stimulate the ventilator to either administer a full breath, or assist the patient with positive air pressure. 2 L/min

The home care nurse visits a 34-year-old woman receiving peritoneal dialysis. Which statement, if made by the patient, indicates a need for immediate follow-up by the nurse? "Drain time is faster if I rub my abdomen." "The fluid draining from the catheter is cloudy." "The drainage is bloody when I have my period." "I wash around the catheter with soap and water."

The fluid draining from the catheter is cloudy."

The nurse is providing discharge instructions to a patient with diabetes insipidus. Which instructions regarding desmopressin acetate (DDAVP) would be most appropriate? The patient can expect to experience weight loss resulting from increased diuresis. The patient should alternate nostrils during administration to prevent nasal irritation. The patient should monitor for symptoms of hypernatremia as a side effect of this drug. The patient should report any decrease in urinary elimination to the health care provider.

The patient should alternate nostrils during administration to prevent nasal irritation.

6 types of burns

Thermal Chemical Electrical Smoke and inhalation Radiation Cold thermal

S/S of __?__ are hypovolemia, hypotension, decreased CVP, weight gain, edema and SOB. Treatment is f/e replacement and albumin

Third space syndrome

The thoracic cavity is defined by:

This space is defined by: Sternum anterior Thoracic vertebrae posterior Ribs lateral Diaphragm inferior

_?_ creates an opening in the chest wall through which a chest tube is placed, which allows air and fluid to flow out of the chest

Thoracostomy

_?_: Surgical incision of chest wall. Ribs are spread. Pleura is opened. _?_: Removal of 1 or more lung segments of lung lobe. _?_: Removal of small localized lesion that occupies only part of a lung segment. __?__: Removal of one or more lobes of the lung. _?_: Removal of entire lung

Thoracotomy Segmental resection Wedge resection Lobectomy Peumonectomy

For thoracic aneurysm repair - _?_incision; Aneurysm excised, graft or prosthesis sewn onto aorta

Thorocotomy or median sternotomy

2 Clinical manifestations of acute mitral valve regurg:

Thready peripheral pulses cool, clammy extremities

HIT patho: _?_5-10 days after Heparin _?_destruction and vascular endothelial injury result from immune-mediated response to Heparin _?_are formed, leading to thrombocytopenia and platelet fibrin thrombi (white clot)

Thrombocytopenia Platelet Antibodies

_?_ is a hormone that is secreted by the anterior pituitary. Its job is to stimulate the thyroid to secret T3 and T4

Thyroid stimulating hormone (TSH)

Vt

Tidal Volume. Usually 6 - 10 mL/kg (ideal body weight)

_?_ is the volume of gas delivered to a patient during each ventilator breath. Normal is _?_

Tidal volume (Vt) 6-10 ml/kg

What is the role of the immune system in cancer

To detect tumor-associated antigens (TAAs)

Collaborative Care in Aneurysms - Goal:

To prevent aneurysm rupture.

A patient is brought to the ED having experienced blood loss related to a GI bleed. Fresh frozen plasma (FFP) is ordered and transfused. The nurse understands that the rationale for transfusing FFP in this client is To treat the loss of platelets To promote rapid volume expansion To increase the H/H To promote clotting

To promote clotting

Gigantism:

Too much GH as child; Onset occurs before closure of epiphyses, therefore can grow as tall as 8 feet.

_?_ inhibit normal enzymes that function to make reversible breaks & repairs in DNA that allow for flexibility of DNA in replication. An examples is irinotecan (Camptosar)

Topoisomerase Inhibitors:

What test is performed before paralyzing a patient for Ventilation therapy?

Train-of-Four (TOF) of the ulnar nerve

Treatment for HIT: Stop heparin, switch to direct _?_ (argatroban=Acova) or _?_ (rivaroxaban = Xarelto; apixaban = Eliquis) NS flush for IVs If clotting severe, plasmapheresis, protamine to counteract circulating Heparin, thrombolytic agents/surgery for clots No platelet transfusion: could make it worse NO LMWH; NO HEPARIN....never, never, never, never again

Treatment for _?_: direct thrombin inhibitor (argatroban=Acova) Factor Xa inhibitor (rivaroxaban = Xarelto; apixaban = Eliquis)

Treatment of Secondary Pulmonary Arterial Hypertension (SPAH)

Treatment: treat underlying primary disorder

A patient complains of nausea. When administering a dose of metoclopramide (Reglan), the nurse should teach the patient to report which potential adverse effect? Tremors Constipation Double vision Numbness in fingers and toes

Tremors

Classification of Aneurysms

True or false aneurysms

Primary nursing concerns for patient with ET tube

Unplanned (inadvertent) extubation Aspiration

The patient with increased ICP from a brain tumor is being monitored with a ventriculostomy. What nursing intervention is the priority in caring for this patient? Administer IV mannitol (Osmitrol). Ventilator use to hyperoxygenate the patient Use strict aseptic technique with dressing changes. Be aware of changes in ICP related to leaking CSF.

Use strict aseptic technique with dressing changes.

Surgical therapy for valve repair or replacement : _?_is typically the surgical procedure of choice. _?_may be required for certain patients. a third choice is _?_

Valve repair Valvular replacement TAVR

6 Indications for the use of Peritoneal Dialysis:

Vascular access problems Diabetic Less complicated than HD Fewer dietary restrictions Can be done at home Fewer BP problems *Note: because PD exchange takes place all day, everyday, functions more like kidney

What might a dressing be covered in to be "occlusive". Why??

Vaseline or bacitracin ointment; makes a better seal

4 Drug Therapies for Cirrhosis Complication of Esophageal Varices:

Vasopressin, Octreotide Propranolol Histamine 2 receptor blockers

4 drug therapies for esophageal varices

Vasopressin, Octreotide Propranolol Histamine 2 receptor blockers

Etiology and Pathology of IE _?_ : the primary lesions of IE (consists of _?_, leukocytes, platelets, and _?_ ) Adhere to the _?_ or _?_ Embolization of portions of vegetation into circulation

Vegetation fibrin; microbes valve or endocardium

_?_ is a common problem accompanying SCI during the first 3 months. If is difficult to detect in a person with SCI

Venous thromboembolism (VTE)

What is the main thing we want to avoid with mechanical ventilation??

Ventilator Associated Pneumonia (VAP). 9 - 27% of all intubated patients.

9 Complications associated with ARDS:

Ventilator-Associated pneumonia (VAP) Barotrauma Stress ulcers Renal Failure Multiple Organ Dysfunction Syndrome (MODS) Oxygen toxicity Tension Pneumo Dysrhythmias Decreased CO

The _?_ is designed to support a failing heart with flow assist (usually as a bridge to transplant)

Ventricular assist device (VAD)

_?_ and _?_ cause the majority of cases of sudden cardiac death (SCD)

Ventricular tachycardia, Ventricular fibrillation

What is the gold standard for ICP measurement?

Ventriculostomy

Common causes of _?_ are: Enteroviruses, arboviruses, HIV, HSV Signs & Symptoms Headache, fever, photophobia, stiff neck Diagnosis Xpert EV test, lumbar puncture

Viral meningitis

Neurological manifestations of DIC include 5 things

Vision changes, Dizzy, HA, Mental status changes, Irritability

When caring for a patient with a biliary obstruction, the nurse will anticipate administering which vitamin supplements (select all that apply)? Vitamin A Vitamin D Vitamin E Vitamin K Vitamin B

Vitamin A Vitamin D Vitamin E Vitamin K

Mayo Clinic: _?_, _?_ and _?_ (2 vitamins and a medication) dramatically Reduce Mortality From Sepsis

Vitamin C, B1 and Hydrocortisone

2 therapies for Clotting abnormalities

Vitamin K Blood products

8 Renal Replacement Therapy (RRT) indications Volume overload resulting in_?_ or _?_ ↑ K+ Metabolic acidosis BUN > _?_ mg/dL Significant decrease in _?_ Pericarditis, pericardial effusion or _?_

Volume overload resulting in cardiac or pulmonary compromise ↑ K+ Metabolic acidosis BUN > 120 mg/dL Significant decrease in mental status Pericarditis, pericardial effusion or cardiac tamponade

6 Renal Replacement Therapy (RRT) Indications

Volume overload resulting in cardiac or pulmonary compromise (i.e. Pericarditis, pericardial effusion or cardiac tamponade) ↑ K+, Metabolic acidosis, BUN > 120 mg/dl (normal is 6-20) Significant decrease in mental status,

Types of Bowel Obstruction: _?_occurs when the lumen of the intestine is blocked in two different spots; the obstruction is proximal to the isolated segment of bowel.

Volvulus

Types of Bowel Obstruction: Strangulation/Gangrene - occurs if there is not immediate attention/treatment. _?_, _?_, and _?_ are the most common causes.

Volvulus, hernias, or adhesions

Important Laboratory tests with endocarditis: Blood cultures _?_ with differential _?_ Chest x-ray

WBC Echocardiography (TEE)

What determines if an electrical burn causes a type of dysrhythmia?

Where the victim was in his cardiac cycle

_?_ and _?_ have a higher risk of AAA than African Americans, Hispanics, and Asian Americans

White & Native Americans:

Treatment for DKA- Insulin therapy: Withheld till fluid resuscitation begun; bolus followed by insulin drip at _?_; Don't drop glucose to rapidly

Withheld till fluid resuscitation begun; bolus followed by insulin drip at 0.1 units/kg/hr; Don't drop glucose to rapidly

What is an impella?

a catheter device that improves pumping

Remember! Frequently hypoxemic respiratory failure is caused by _?_. For example COPD with _?_ and _?_

a combination of 2 or more causes; COPD with PNA and ARDS

What does a normal ICP waveform look like?

a descending staircase

The staging classification system is based on _?_ rather than on cell appearance. Assignment is completed after the diagnostic workup and determines treatment options.

a description of the extent of the disease

What is normal for each of the following? a. CVP/RAP b. PAP c. PAWP/LAP d. CO

a. CVP/RAP = 2-8 b. PAP = 12-16 (<25 at rest; < 30 exercise) c. PAWP/LAP = 6-12 L/min d. CO = 4-8 L/min

The MD wrote an order this am for Mrs. Hart to receive Metoprolol (Lopressor) 12.5 mg po BID. Mrs. Hart is a first day post op Coronary Artery Bypass Graft surgery patient. Mrs. Hart's blood pressure is 110/70. Her heart rate is 98 beats per minute NSR. The RN knows that metoprolol will do which of the following for Mrs. Hart: a. Decreases heart rate and reduces myocardial oxygen consumption b. Increases heart rate and reduces myocardial oxygen consumption c. Causes vasoconstriction to increase blood pressure d. Causes vasodilation to increase blood pressure

a. Decreases heart rate and reduces myocardial oxygen consumption

What is considered "normal" for each of the following? a. HR b. Mixed venous hemoglobin O2 sat c. Arterial hemoglobin O2 sat

a. HR 60-100 b. Mixed venous hemoglobin O2 sat 60-80 c. Arterial hemoglobin O2 sat 95-100

2 Signs/symptoms of Acute ascending aortic dissection:

abrupt onset of excruciating chest and/or back pain radiating to the neck or shoulders

8 s/s that blunt trauma has resulted in other conditions:

absent breath sounds anxiety decreased breath sounds dyspnea hemoptsis increased RR increased HR restlessness

Two types of hypovolemic shock are _?_ and _?_

absolute relative

What is the drug treatment for myocarditis?

abx and NSAIDS

Nursing interventions for flulike syndrome Administration of _?_ before treatment and every 4 hours after treatment _?_ to control severe chills (use with caution) Rest periods Temp & VS monitoring Assist with ADLs Monitor for adequate oral intake because _?_

acetaminophen IV meperidine (demeral) fluid shift will cause hypotension

What is a renal complication associated with ARDS?

acute kidney injury

The following are diagnostic studies for _?_ AST, ALT, GGT Total protein, albumin Serum bilirubin PTT; PT/INR Ammonia level (arterial if possible) Ultrasound, EGD, CT, MRI CBC, CMP, tylenol level, Viral hepatitis serologies Tylenol level Viral hep panel Cholesterol level (d/t abnormal fat metabolism) Analysis of ascites fluid Liver biopsy

acute liver failure

_?_- a condition caused by inflammation of the pericardial sac/pericardium

acute pericarditis

Because ARDS causes _?_, the subjective and objective data that you should obtain from a person with ARDS are the same as those for _?_

acute respiratory failure; acute respiratory failure

Urinary complications post burn: Most common complication of urinary system in emergent phase is _?_ . Adequate __________________can counteract obstruction of tubules.

acute tubular necrosis (ATN); fluid replacement

When testing for Cushing Syndrome, a cortisol level is taken prior to the administration of dexamethasone. Then another cortisol level is taken. With Cushing Syndrome, we will see _?_

an increase in cortisol

_?_ shunting is when the blood exits the heart w/o participating in gas exchange due to a defect in the heart

anatomic shunting

3 hematologic clinical manifestations of CKD

anemia bleeding infection

Malignant hypothermia occurs in susceptible patients when exposed to certain _?_ (succinylcholine, or anectine) especially when given via _?_

anesthetic agents; inhalation

Clinical manifestations of Mitral Valve Prolapse: May or may not present with _?_ If chest pain occurs, episodes tend to occur in clusters, especially during stress. Pain may be accompanied by _?_ palpitations, and syncope. Pain does not respond to_?__treatment.

angina (chest pain) dyspnea, antianginal

_?_ is a class of drugs that interfere with DNA and RNA growth by substituting for the normal building blocks of RNA and DNA. Examples are: fluorouracil (5-FU) methotrexate (Trexal) hydroxyurea (Hydrea) cladribine (Leustatin)

anitmetabolites

People are living long enough to develop severe _?_ . It is an illness of _?_. More than 8,000 Americans die from AS annually Many are too old or too sick for _?_ so instead, they may opt for _?_

aortic stenosis aging open aortic valve replacement (AVR) Transcatheter Aortic Valve Replacement (TAVR)

_?_ occurs and sometimes may continue for weeks or months after the initial SCI

apoptosis

Medications like Ketoconazole and aminoglutethimide are used for patients with Cushing Syndrome who _?_. These meds are not used all of the time because _?_

are not candidates for surgery; the are toxic in doses necessary to inhibit cortisol secretion

ADH is also known as _?_ or _?_

arginine vasopressin (AVP) or vasopressin

When is the best time to give chemo? Why?

around 7am highest DNA activity

Mrs. Thorne has a subarachnoid hemorrhage (SAH) which may be a result of bleeding from a cerebral aneurysm. You know that SAH resulting from a cerebral aneurysm is characterized by:

arterial bleeding into the subarachnoid space

4 Vascular Access Sites for dialysis:

arteriovenous fistulas arteriovenous grafts temporary and semipermanent venous caths

Dialysis Vascular Access Sites: _?_-- best patency, least complications - artery and vein, usually cephalic, or forearm - insert 3 months prior to use

arteriovenous fistulas (AVF)

Dialysis Vascular Access Sites: _?_ is made of teflon and acts as "a bridge",; may use in 2-4 weeks, has bruit and thrill to be assessed. No _?_ or _?_ in that arm

arteriovenous grafts (AVG) no BP or lab draws in that arm

If a patient has both decerebrate and decorticate posturing, how would you score their "motor response" on the GCS? Why?

as a 3; they have both extension (2) and flexion (3), so they get the highest score

In the spinal cord, ascending tracts are _?_, while descending tracts are _?_

ascending - sensory descending - motor

Collaborative care for _?_ includes: Rest Admin of B-complex vitamins Avoid alcohol; minimize/avoid: aspirin/tylenol/ NSAIDS Sodium restriction May give albumin Diuretic therapy (loop & potassium sparing) Spironolactone : antagonist of aldosterone IV antibiotics Paracentesis Peritoneovenous Shunt If ammonia elevated: Lactulose (Cephulac) and neomycin or rifaximin (Rifampin)

ascites/liver failure

Recent research has described the therapeutic benefits associated with _?_ treatment for sepsis.

ascorbic acid

_?_ is associated with hepatic encephalopathy. It is also called liver flap

asterixis

_?_ donors: If pt. has some intact neuro function & there will be an "end of life" discussion or if the family has already elected to remove the pt. from life support. Gift of Hope evaluates to see if client meets criteria for _?_ donation &/or tissue donation

asystolic organ asystolic organ

Nadir is the time when bone marrow activity and WBC counts are at their lowest levels after the chemo treatment. To reduce immunosuppression, combination chemotherapy avoid using drugs with nadirs that occur _?_

at or near the same time.

Cervical and thoracic SCI cause paralysis of abdominal and intercostal muscles. Therefore the patient cannot cough effectively enough to remove secretions, leading to _?_ and _?_

atelctasis pneumonia

_?_ results in ARDS d/t decreased production of surfactant

atelectasis

With fractured ribs, encourage coughing and deep breathing to prevent _?_ and _?_

atelectasis and PNA

_?_ is the excision and removal of atheroslerotic plaque by cutting, shaving or grinding

atherectomy

Urinary retention common in acute spinal cord injuries and spinal shock Spinal shock: Bladder is _?_ and _?_ Indwelling catheter inserted Increased risk of infection Post acute phase: Bladder may become _?_ Loss of inhibition from brain Resulting in reflex emptying

atonic and overdistended ' hyperirritable

2 Systemic diseases that may cause GI bleeding:

blood dyscrasias renal failure

Very important to measure _?_ in patient with spinal cord injury if they are complaining of headache

blood pressure

What will urine look like post-renal transplant? What volume of fluids will be given?

bloody; replace urine output ml for ml

In _?_ trauma, external injuries may appear minor but impact may cause severe, life-threatening internal injuries such as ruptured spleen.

blunt

What are the 3 classifications of chest trauma?

blunt, penetrating, contrecoup

If crack in collection device, CT inserted into_?_, replace old one with new collection device If the chest tube becomes dislodged, _?_

bottle of sterile H2O; apply pressure over the insertion site with a dressing that is tented on one side to allow for the escape of air

If an x-ray shows free gas/air in abdominal cavity, what is that indicate?

bowel perforation

If the vegetation from IE travels through the left side of the heart, where might it manifest? (5 places)

brain limb kidney liver spleen

Signs & Symptoms of _?_ Headache, fever, N/V, increased ICP (drowsiness, confusion, seizures), or focal symptoms due to area of abscess

brain abscess

_?_ is a collection of pus within the brain tissue due to local or systemic infection Primary Cause: Extension from ear, tooth, mastoid, or sinus infection Primary Organisms: Streptococci & staph aureus

brain abscess

Prolonged hypoxia may lead to _?_ and _?_. Ischemia will increase the permeability of the intestinal wall, increasing the risk for _?_

brain damage, GI ischemia, perforation

_?_ is the most common cause of death after initial resuscitation post-return of spontaneous circulation

brain injury

What are the 3 essential components of intracranial pressure?

brain tissue blood CSF

_?_ and _?_ are very important to prevent pathogens from entering artificial airways

bronchial hygiene chest physiotherapy

If we suspect that a patient has smoke inhalation, a _?_ is needed within the first few hours to assess the _?_

broncoscopy lower airway

How is dyspnea measured? What is considered mild? Moderate? Severe? Very severe?

by the ability to speak; mild = complete sentences moderate = phrases 1 word at a time = severe can't speak = very severe

A patient with aspiration PNA develops respiratory distress. Which diagnostic finding would indicate to the nurse that the patient has developed acute respiratory distress syndrome (ARDS) a. PaCO2 of 31 mm Hg and pH of 7.51 b. PaO2 of 64 mm Hg on 24% FiO2 c. PaO2 of 42 mm Hg on 80% FiO2 d. PaCO2 of 70 mm Hg and pH 7.29

c. PaO2 of 42 mm Hg on 80% FiO2

In an episode of MH, muscle metabolism is dramatically increased secondary to an increase in _?_ within the muscle. This causes muscles to contract, ATP hydrolysis, and heat production.

calcium

_?_ is a rare disorder characterized by recurrent flares of massive leakage of plasma and other blood components from blood vessels into neighboring body cavities and muscles. This leads to swelling. The symptoms result from a sudden and unexplained increase in the leakiness of small blood vessel (capillary) walls. Unless treated, massive fluid shifts result in a sharp drop in blood pressure that can lead to organ failure and death.

capillary leak syndrome

2 metabolic clinical manifestations of CKD

carbohydrate intolerance hyperlipidemia

_?_ are the most common types of cancer; the name indicates cancer of epithelial cells

carcinoma

Death is legally defined as either brain death or __________ death.

cardiac

7 Late signs of malignant hyperthermia are:

cardiac arrest DIC Myoglobinuria Elevated creatine phosphokinase Elevated temp Hypo/Hypercalcemia Mottled cyanosis

Tissue Donor Criteria: Any pt. who suffers _?_ is potentially eligible The pt. must have a known time of death or have been witnessed as alive within the _?_ prior to being found deceased

cardiac death 24 hours

A 56-year-old woman with type 2 diabetes mellitus and chronic kidney disease has a serum potassium level of 6.8 mEq/L. The nurse should assess the patient for fatigue. flank tenderness. cardiac dysrhythmias. elevated triglycerides.

cardiac dysrhythmias.

Plan for patient with IE: Patient will have normal _?_ Perform ADLs without _?_ Understand therapeutic regimen to prevent _?_

cardiac function fatigue recurrence

Patients who survive SCD required _? and _?_ to determine if they've had an MI. Additionally, most have CAD, so a _?_ is indicated. Results may indicate PCI or CABG

cardiac markers EKG cardiac cath

Signs of _?_ include decreased CO, decreased BP, JVD, pulsus paradoxus, muffled heart sounds, sudden cessation of chest tube drainage and widened cardiac silhouette on x-ray.

cardiac tamponade

Treatment for _?_ may include: pericardial window, drain O2 vasopressors IV fluids if hypotensive or hypovolemic

cardiac tamponade

Based on the following numbers, the patient has _?_ CI = 1.9 SVR 1550 PAWP 17 Low urine output Mixed venous sat 50%

cardiogenic shock

Adrenal Medulla: Releases _?_

catecholamines

Prevention of Cirrhosis cease _?_ early identification of _?_or other causes Nutrition - regenerate liver tissue; small frequent meals, high calorie caution - _?_ and _?_ use Rest - restore liver tissue: keep _?_ caution - pneumonia, pressure ulcers, clots Teaching - signs of complications, skin therapy, counseling for sexual problems, avoidance of hepatoxic meds, abstinence from alcohol (AA), nutrition, rest

cease alcohol consumption early identification of viral hepatitis or other causes Nutrition - regenerate liver tissue; small frequent meals, high calorie caution - protein & sodium use Rest - restore liver tissue: keep HOB elevated caution - pneumonia, pressure ulcers, clots Teaching - signs of complications, skin therapy, counseling for sexual problems, avoidance of hepatoxic meds, abstinence from alcohol (AA), nutrition, rest

Cancer includes 2 defects 1. Defect in _?_ 2. Defect in _?_

cell proliferation cell differentiation

D5W or 1/2 NS will increase _?_

cerebral edema

Post-aneurysm surgery - Monitor for signs of graft occlusion (5 signs)-

changes in pulses, cool to cold extremities below graft, white or blue extremities/ flanks, severe pain, abdominal distention

Nursing Care Pre-Dialysis: 2 methods to check condition of vascular access -

check temperature and skin condition thrill and bruit

Goal of _?_ is to reduce number of malignant cancer cells in tumor site(s). There is some selectivity for killing cancer cells over normal cells however, it exerts its cell-damaging effects on healthy cells also

chemotherapy

The following are possible s/e of _?_ Fatigue Alopecia Metabolic alterations Cystitis

chemotherapy

After heart cath, frequent assessment is done to monitor for _?_ and _?_. This is done to ensure adequate circulation to the heart via the coronary arteries (i.e. to make sure no clots have formed or plaque has dislodged)

chest pain dysrhytmias

The pain in _?_ is described as cramping, burning, gnawing

chronic pancreatitis

_?_ can be distinguished from _?_ in that food and antacids will help the second condition, but not the first

chronic pancreatitis gastric ulcers

Nursing Interventions for _?_ during _?_ Monitor for hemodynamic stability Monitor for respiratory failure Monitor for hypocalcemia Monitor for infection Monitor for hyperglycemia and steatorrhea Maintain IV fluids as ordered Relieve pain NPO - NG tube

chronic pancreatitis; acute pancreatitis

Infiltration of T and B cells characteristic of an ongoing immune-mediated injury is a feature of _?_ rejection

chronic rejection

_?_ drainage appearance is milk whitish....may have blood or sanginous drainage so could be milk white with reddish tinge.

chylothorax

Early manifestations of _?_ are: anorexia, nausea/vomiting, diarrhea or constipation, flatulence, dyspepsia Abdominal pain & palpable liver Fever Lassitude Slight weight loss Enlargement of liver and spleen

cirrhosis

In _?_, the liver cells attempt to regenerate; process is disorganized and results in abnormal blood vessels and bile duct architecture. This distorts liver's normal lobular structure; lobules irregular size and shape with impeded blood flow Decreased liver function d/t above, poor cellular nutrition, and hypoxia Usually insidious, prolonged course

cirrhosis

Later manifestations of _?_: may be severe and result from liver failure and portal hypertension Jaundice Peripheral edema Ascites Hematologic disorders Thrombocytopenia, leukopenia, anemia, coagulation disorders Endocrine disturbances Peripheral neuropathies

cirrhosis

_?_ is a chronic progressive disease of liver characterized by extensive degeneration & destruction of the liver parenchymal cells.

cirrhosis

Symptoms of _?_ Brief disruption in level of consciousness Amnesia Headache Short duration

concussion

A patient does not have spontaneous breathing when the _?_ mode is used. All breaths are machine-delivered and controlled, regardless of any inspiratory efforts the patient may have. There is a preset breathing rate and a preset tidal volume or pressure. When a patient has no respiratory drive, this mode of ventilation must be used.

continuous mandatory ventilation (CMV)

Organ on organ trauma or organ on bone trauma would be classified as _?_ trauma. When this occurs, organs and vessels can be damaged

contrecoup

Ascending aortic dissection - disruption in blood flow to _?_and aortic valve insufficiency; see angina or new murmur

coronary arteries

The term _?_refers to any one of these three types of hormones produced by the adrenal cortex

corticosteroid

Name 6 clinical manifestations of ARDS:

crackles organ failure metabolic acidosis hypoxia/refractory hypoxemia tachycardia ST depression (indicates ischemia)

A final _?_ is done prior to transplant. _?_ is especially important for kidney transplants, and may not be done for lung, liver, and heart.

crossmatch Crossmatching

The _?_ is the "floor" of the thoracic cavity

diaphragm

_?_ is contraindicated in patients with MI less than one week ago, unstable angina, uncontrolled CHF, severe aortic stenosis, SVT, VT, hypotension, etc.

dobutamine stress test

Besides viruses and bacteria, what are 4 other causes of hepatitis?

drugs including alcohol, chemicals, auto-immune diseases, metabolic abnormalities

Name 1 skin reaction d/t radiation

dry to wet desquamation

Common adverse effect of esophageal cancer surgery is _?_ . It is unregulated gastric emptying and rapid delivery of carbohydrates and partially digested food into small intestines

dumping syndrome

To decrease the risk of _?_, minimize liquids at meals, low carb meals help control

dumping syndrome

Nursing Assessment #6.....heart punctured by rib **Inspect: anterior chest trauma, bruising, possible rib fractures **Monitor for _?_ **Cardiac biomarkers (troponin T) are not useful in predicting which pts will have complications r/t the problem

dysrhythmias and signs of heart failure

Skin layers: _?_ - nonvascular, thin as sheet of paper _?_ 30-45 times thicker than epidermis, contains connective tissues with blood vessels, hair follicles, nerve endings, sweat glands, and sebaceous glands _?_contains major vascular, fat, nerves, and lymphatics

epidermis; dermis subcutaneous

How will we treat the anemia that comes with AKI?

epogene to replace erythropoietene

Nursing care post _?_ cancer surgery: ICU for at least 24-48 hrs Usually intubated Multiple drains/tubes Cardiopulmonary monitoring: EKG, Pulse ox, VS q1hr and prn Neuro Status: access q1-4hrs and prn Pain management:

esophageal

Normally, our ventilatory supply is greater than our demand. This is why we are able to _?_

exercise

Most SCD patients have lethal ventricular dysrhythmia. Assessment includes 24-hour Holter monitoring _?_ _?_ and _?_

exercise-stress testing signal-averaged ECG electrophysiology study (EPS)

Clinical manifestations of Cor Pulmonale: subtle; often masked by symptoms of the pulmonary condition 4 Symptoms:

exertional dyspnea tachypnea cough fatigue

An _?_, also known as a ventriculostomy or extraventricular drain, is a device used in neurosurgery to treat hydrocephalus and relieve elevated intracranial pressure when the normal flow of cerebrospinal fluid (CSF) inside the brain is obstructed.

external ventricular drain (EVD)

Aortic Dissection - Results from the "creation of a _?_ between the intima (inner lining) and the media (middle layer) of the arterial wall"

false lumen

4 neurologic clinical manifestations of CKD

fatigue headache sleep disturbances encephalopathy

Etiology and Pathophysiology of IPAH Cause is unknown Affects _?_more than _?_ Pathophysiology is poorly understood Some type of insult to the _?_ may occur, causing a cascade of events leading to vascular scarring, endothelial dysfunction, and smooth muscle proliferation

females more than males pulmonary endothelium

_?_ : Most common type of chest injury resulting from trauma

fractured ribs

_?_ jaundice is caused by blood transfusion reactions, sickle cell crisis, hemolytic anemia

hemolytic

Blood in the pleural space is a _?_

hemothorax

_?_ is insufficient CO2 removal

hypercapnia

Post-aneurysm surgery Monitor for _?_d/t blood loss

hypovolemia/kidney failure

Give 4 examples of viral carcinogens

i. Epstein Barr Virus - Burkitt's lymphoma ii. HIV - Kaposi sarcoma iii. HBV - hepatocellular carcinoma iv. HPV - cervical, anal, head, neck cancer

The clinical staging classification system uses five stages, from _?_ to _?_

in situ to metastasis.

A(n) _?_ is anything in the lung/lung spaces that should not be there

infiltrate

Side effects of _?_ include flu-like symptoms and changes in cognition

interferons

Why is pursed lip breathing helpful to someone with hypercapnia?

it blows off more CO2

Why does the heart become tachy in hypoxia?

it increases its rate to increase blood flow (and thereby O2); not very successful

_?_ may be d/t abnormal tension on the papillary muscles

mitral valve prolapse

_?_ are cancers of plasma cells (a type of immunity cell)

multiple myeloma

Does a patient need a positive blood culture to be septic?

no

_?_ and _?_ are produced by the hypothalamus, but stored in the posterior pituitary

oxytocin ADH

Signs of a _?_ Small bowel obstruction (SBO): the obstruction produces rapid-onset of nausea/Vomiting (bile), Pain

proximal

What lab value will be decreased with pancreatitis?

serum calcium

Normally, ADH is secreted in response to _?_, which is monitored by osmoreceptors in _?_

serum osmolality; hypothalamus

_?_ is an imbalance in supply/demand for O2 and nutrients

shock

Vasodilator therapy with IV _?_ or _?_ is often used to reduce afterload, control HTN and improve CO

sodium nitroprusside (Nitropress) nitroglycerin

Aneurysm pressing on _?_ causes decreased venous return - see JVD, edema face/arms.

superior vena cava

If IICP is d/t brain mass, lesion or hematoma, the patient may require _?_

surgery

HIT results in venous &/or arterial _?_

thrombosis

_?_ burn injuries require early intubation

upper airway injury

Open pneumothorax should be covered with a _?_ Treat open & closed pneumothorax with _?_

vented dressing; chest tube

With _?_ a predetermined tidal volume is delivered with each inspiration and the amount of pressure needed to deliver the breath varies

volume ventilation

Complications and Management after Thoracic Surgery: Hemorrhage: Assess drsg, incisional area, drainage in CT (call if > 100 ml/hr (MD usually orders call for amount). If CT drainage _?_ call MD. Assess _?_ Assess for s/s of hypovolemic shock d/t hemorrhage

warm and free flowing ; vital signs

ADH regulates _?_ and _?_

water balance and osmolarity

Due to hypomotility (as a result of SCI), _?_ and _?_ may develop. These are treated with NG tube and _?_

paralytic ileus gastric distention metoclopramide (reglan)

2 peripheral neuropathy clinical manifestations of CKD

parathesia restless leg syndrome

If in descending aorta: _?_cardiopulmonary bypass

partial

How is a parathyroidectomy performed?

partial or complete; out-patient procedure (endoscopy)

Nursing and collaborative management for patients with burns: Encourage patient and caregiver to _?_. Pain management and nutritional needs individually based. Home care nursing services if needed. _?_ skin. _?_ is frequently required for major burn. Encourage _?_ and _?_ rehab. Encourage and reassurance.

participate in care Moisturize Reconstructive surgery physical and occupational

In histologic grading of tumors, the appearance of cells and the degree of differentiation are evaluated_?_. For many tumor types, _?_grades are used to evaluate abnormal cells based on the degree to which the cells resemble the tissue of origin.

pathologically; four

4 conditions requiring surgery for aneurysm to be done sooner than 5.5 cm+

patient has a genetic disorder (Marfan's or EDS); aneurysm expands rapidly ; pt becomes symptomatic ; risk of rupture is high.

Before surgery, mark _?_with a single-use marker and document any skin lesions.

pedal pulse sites (dorsalis pedis and posterior tibial)

Drug Therapy for Hepatitis Hep C: treat w/ _?_ within 1st 12-24 weeks of infection...decreases development of chronic hep C.

pegylated interferon

Treatment of intraabdominal hypertension and compartment syndrome(2) :

percutaneous cath decompression, leave abdomen open

When alveoli fill with fluid or collapse and the alveolar-capillary membrane thickens, alveoli are adequately _?_, but are not adequately _?_. _?_occurs. Gas exchange does not occur, and unoxygenated blood is returned to the left side of the heart. Hypoxemia persists despite administration of high concentrations of oxygen.

perfused; ventilated Intrapulmonary shunting

In the normal lung, the amount of blood _?_ each minute is approximately equal to the amount of fresh gas that reaches the alveoli each minute_?_ L

perfusing the lungs; (4-5L)

How do you differentiate between pericardial rub and pleural rub?

pericardial rub can still be heard when patient is holding his breath

Nursing Assessment for AA - Special attention to: patient's_?_, renal and neurologic status

peripheral pulses

In neurogenic shock, loss of sympathetic nervous system innervation causes _?_, _?_ and decreased _?_

peripheral vasodilation venous pooling decreased cardiac output

Treatment for GI Perforation: surgical intervention to close perforation, _?_ to clean out abdomen, IV fluids, _?_ (type of med) NG tube to suction, bowel rest.

peritoneal wash Antibiotics

_?_ is a type of anemia; low levels of red blood cells caused by inability to absorb vitamin B12

pernicious anemia

What are the "Loss" criteria for AKI

persistent AKI (a complete loss of renal function > 4 weeks)

Early s/s of cancer in _?_: Vary d/t tumor location C/O persistent unilateral sore throat or ear pain (otalgia) C/O "feels like lump in neck", change in voice quality Hoarseness may be sign of early laryngeal cancer Sore throat/ear pain Swelling/lumps neck Expectoration blood

pharynx/larynx

Possible Findings with _?_ cancer: Mass on direct or indirect laryngoscopy; tumor on soft tissue x-ray, CT, or MRI; positive biopsy

pharynx/larynx

Possible findings with _?_ cancer Gastrointestional: white (leukoplakia) or red (erythoplakia) patches on inside mouth, ulceration of mucosa, asymmetric tongue, exudate in mouth or pharynx, mass or thickening mucosa

pharynx/larynx

Possible findings with _?_ cancer Respiratory: hoarseness, changes in voice quality, chronic laryngitis, nasal voice; palpable neck mass and lymph nodes (tender, hard, fixed); tracheal deviation; dyspnea; stridor (late sign)

pharynx/larynx

The following are later s/s of _?_ cancer: Dysphagia Decreased mobility of tongue/jaw Airway obstruction Cranial nerve neuropathies

pharynx/larynx

Disorders of the Adrenal Medulla: _?_ a tumor of the adrenal medulla that produces excessive catecholamines (epi and norepi) or inherited in persons with multiple endocrine neoplasia

pheochromocytoma

_?_ is an endocrine disorder that can lead to: hypertensive encephalopathy, DM, cardiomyopathy death

pheochromocytoma

Injury or fracture below the level of C4 results in diaphragmatic breathing if the _?_ nerve is functioning

phrenic

_?_ can bring on an episode of phenochromocytoma crises:

physical/emotional stress

Hyperpituitarism is usually a result of _?_. Manifestations result from excess of which 4 hormones?

pituitary tumor; GH, prolactin, ACTH, or TSH.

Treat GBS with _?_ or _?_ No benefit to do both

plasmapheresis or immunoglobulin.

Use _?_ in GBA pts seeking treatment w/in first 2 weeks of symptom development ; Use IV _?_ No _?_ use unless necessary for other diseases

plasmapheresis; immunoglobulin; corticosteroid

Lungs are surrounded by thin tissue called the _?_ , a continuous membrane that folds over itself _?_ lines the chest wall _?_ covers the lung (sometimes called the pulmonary pleura)

pleura; Parietal pleura Visceral pleura

Transudate or exudate in the pleural space is a _?_

pleural effusion

The area between the pleurae is called the _?_ Normally, vacuum (negative pressure) keeps the two pleurae together and allows the lung to expand and contract

pleural space

Water seal is a window into the _?_ Not only for pressure If air is leaving the chest, _?_ will be seen here Air leak meter (1-5) provides a way to measure the leak and monitor over time - getting better or worse?

pleural space; bubbling

Which thoracic surgery does not require a chest tube?

pneumonectomy (removal of entire lung)

Air between the pleurae is a _?_

pneumothorax

Intermittent bubbling in water seal chamber expected if pt has -?_

pneumothorax

_?_ is a complete or partial collapse of lung as a result of an accumulation of air in the pleural space. Can be closed or open.

pneumothorax

_?_ is the adjustment of body temp to room temp Occurs in SCI d/t sympathetic nervous system interruption prevents peripheral temperature sensations from reaching hypothalamus With spinal cord disruption, below level of injury Decreased ability to sweat Decreased ability to shiver Degree depends on level of injury Cervical injuries = greater loss of thermal regulation than thoracic and lumbar injuries

poikilothermism

_?_ is the adjustment of body temperature to room temperature. This occurs in SCI's because of disruption of the sympathetic nervous system

poikilothermism

20 Clinical Manifestations of DKA

polyuria Polydipsia Polyphagia Weight loss Nausea/vomit Blurred vision/H/A Orthostatic hypotension Fruity breath Kussmaul's respirations Metabolic acidosis Change in LOC; Lethargy/weakness Dehydration Poor skin turgor Dry mucous membranes Tachycardia Abdominal pain

A _?_ indicates that the organ recipient has cytotoxic antibodies to the donor and is an absolute contraindication to transplantation.

positive crossmatch

If adjustments in HR, preload and afterload fail to produce significant improvement in CO, contractility can be enhanced with _?_ or _?_

positive inotropes or balloon (IABP)

During inspiration the ventilator pushes air into the lungs under positive pressure. This describes _?_

positive pressure ventilation

__?_ is a spinal cord syndrome in which the dorsal columns are damaged resulting in loss of prprioception

posterior cord syndrome

In _?_, the etiology is due to obstruction

postrenal AKI

Subacute infective endocarditis affects those with _?_ Longer clinical course (over months) _?_ onset

preexisting valve disease; Insidious

PEEP increases intrathoracic pressure, which causes a decrease in _?_ . To maintain cardiac output and tissue perfusion, circulating fluid volume should be adequate.

preload and cardiac output.

In _?_ etiologies of AKI, decreased blood flow is the major conflict

prerenal AKI

The initial mechanical disruption of axons as a result of stretch or laceration is referred to as _?_

primary injury

Diagnosis of DI: History - If patient is overhydrating and hypovolemic then _?_ DI

primary or psychogenic

When are anti-emetic drugs administered to cancer patients?

prior to chemo and food (prn and scheduled)

DKA is most likely caused by _?_

profound deficiency of insulin

S/S of the _?_ stage of shock: Ischemic gut Renal failure Skin cold and clammy (hypothermic) In septic shock can be hypo or hyperthermic Coagulation problems

progressive stage

__?__ is a hormone which stimulates milk production and development of secondary sex characteristics. It is secreted by the anterior pituitary gland

prolactin

Normal cell proliferation includes 3 things: 1. Generation time of cell: proliferation + _?_ 2. _?_ - physical contact stops proliferation 3. Normal cells have a specific rate and only produce 2 cells per mitosis cycle

proliferation equals cell degeneration/death contact inhibition - physical contact stops proliferation

AKI usually affects people with other life-threatening conditions. It is most common after _?_, _?_ or _?_.

prolonged hypotension hypovolemia exposure to nephrotoxic agents (Vanco, IV dye)

Surviving Sepsis Campaign: use _?_ over _?_ position in adult patients with a PaO2/FIO2 ratio <_?_

prone over supine 150.

Prophylactic care for patients with IE: _?_ history of _?_ vascular grafts some congenital heart conditions valvular dysfuntion _?_cardiomyopathy Orthopedic prosthesis low risk

prosthetic heart valves; endocarditis hypertrophic

3 integumentary clinical manifestations of CKD

pruritus ecchyomosis dry, scaly skin

Secondary Pulmonary Arterial Hypertension (SPAH) Primary disease causes a chronic increase in_?_ Can develop after parenchymal lung disease, left ventricular dysfunction, intracardiac shunts, chronic pulmonary thromboembolism, or systemic connective tissue disease.

pulmonary artery pressures

Neurogenic _?_ may occur secondary to a dramatic increase in sympathetic nervous system activity at the time of SCI

pulmonary edema

3 pulmonary clinical manifestations of CKD

pulmonary edema uremic pleuritis pneumonia

Cor Pulmonale: Collaborative Care - GENERAL CARE Treat underlying _?_ Long-term _?_therapy: correct hypoxemia, reduces vasoconstriction & pulm HTN Correct _?_ imbalance _?_ indicated in pts w/ chronic cor pulmonale and chronic hypoxia causing severe polycythemia (hematocrit 65 or greater)

pulmonary problem; O2 electrolyte Phlebotomy

Surgical Interventions for Secondary Pulmonary Arterial Hypertension (SPAH) For patient with chronic PH caused by thromboembolism: _?_ may offer a cure

pulmonary thromboendarteretomy (PTE)

Surviving Sepsis Campaign If shock is not resolving quickly: further hemodynamic assessment (such as assessing cardiac function) to determine the type of shock if the clinical examination does not lead to a clear diagnosis. dynamic over static variables be used to predict fluid responsiveness, where available. 3 Dynamic measures are: _?_, _?_ and _?_

pulse pressure variation, systolic pulse variation, stroke volume variation,

With brain injury, pupil changes are (ipsilateral or contralateral), while motion changes are (ipsilateralor contralateral)

pupils - ipsilateral Motion - contralateral

The _?_score (also known as _?_ ) is a bedside prompt that may identify patients with suspected infection who are at greater risk for a poor outcome outside the intensive care unit (ICU). It uses three criteria, assigning one point for low blood pressure (SBP≤100 mmHg), high respiratory rate (≥22 breaths per min), or altered mentation (Glasgow coma scale<15).

qSOFA quickSOFA

If a subclavian artery is involved in an aortic dissection: the _?_, _?_ and_?_pulse quality and BP readings may be different in the left and right arms

radial, ulnar, and brachial

Three Types of cervical lymph node dissection:

radical neck Modified neck Simple/Selective neck

Setting up suction Vigorous bubbling is loud and disturbing to patients; and will cause _?_ in the chamber, which will lower suction level More is not _?_ If too much, turn down vacuum source until bubbles go away, then slowly increase until they reappear, then stop

rapid evaporation better

In the post acute phase of SCI, the neurogenic bowel becomes _?_ and reflex emptying occurs

reflexic

What is a major side effect of Bisphosphonates (Fosamax)?

reflux/ulcers/GERD

_?_ is defined as the v/q mismatch and shunting of pulmonary capillary blood with hypoxemia which is unresponsive to increased oxygen

refractory hypoxemia

_?_ is hypoxemia which is unresponsive to increased oxygen concentration

refractory hypoxemia

Nursing and Collaborative Management of IPAH: No Cure GOALS: _?_, _?_, and _?_ Need early recognition to decrease progression of disease _?_ provides symptom relief

relieve symptoms, improve quality of life, prolong life; Low-flow O2

In the Diuretic phase of AKI, BUN and creatinine _?_

remain ↑ but may be losing electrolytes

Manifestations of hyperparthyroidism: 2 Renal:

renal calculi, polyuria

Assessment for pleural effusion: pleuritic pain, sharp, increases with _?_ Progressive dyspnea and have decreased _?_ Dry, nonproductive cough (d/t bronchial irritation or mediastinal shift) Tachycardia Elevated temp Decreased breath sounds affected area

respiration chest wall motion on affected side

_?_ is a symptom of underlying pathologic condition; classified as hypoxemic or hypercapnic

respiratory failure

With esophageal and gastric varices, treat _?_ infections promptly and control _?_ to reduce risks of rupture

respiratory infections coughing

Classification of Burn -- Location Location of the burn is related to the severity of the injury Face, neck, chest → _?_ Hands, feet, joints, eyes → _?_ Ears, nose, buttocks, perineum →_?_

respiratory obstruction self-care infection

_?_ is the number of breaths the ventilator delivers per minute. Normal is _?_

respiratory rate (f); 6-20 breaths/min

Most any disorder affecting the _?_ can cause cor pulmonale _?_usually pre-existing condition with Cor Pulmonale

respiratory system Pulmonary Hypertension

Collaborative Care for Valve Stenosis: Prevention of recurrent _?_ and _?_ is essential. Treatment depends on valve involved and severity of disease. Focus on preventing exacerbations of heart failure, acute pulmonary edema, thromboembolism and recurrent endocarditis

rheumatic fever and infective endocarditis

Etiology and Pathophysiology of acute pericarditis: Hypersensitive or autoimmune (3) and Idiopathic

rheumatic fever, drug interactions, rheumatic diseases (unknown)

Aortic valve stenosis is usually discovered during childhood. However, if it is diagnosed in an adult it is likely that they have aortic stenosis from _?_ or _?_ of a normal valve.

rheumatic fever; senile fibrocalcific degeneration

"Chest wall" composed of what 4 things

ribs, sternum, thoracic vertebrae interlaced with intercostal muscle

Pulmonary hypertension increases the workload of the _?_ and causes _?_ (also called cor pulmonale) and eventually heart failure

right ventricle; right ventricular hypertrophy

Initial physical findings of Secondary Pulmonary Arterial Hypertension (SPAH) can include: _?_ hypertrophy signs of _?_ (increased pulmonic heart sound, right-sided 4th heart sound, peripheral edema, and hepatomegaly)

right ventricular right ventricular failure

What are the following signs of? increased pulmonic heart sound right-sided 4th heart sound peripheral edema hepatomegaly

right ventricular failure

For those who are asymptomatic, how is hyperparathyroidism found?

routine Ca screening

_?_ is a name for cancers of the bone and muscle

sarcoma

_?_ traumatic brain injuries are d/t poor O2, blood or glucose supply to the brain. They occur hours to days after the initial insult

secondary

_?_ injury refers to the ongoing progressive damage that occurs after the primary SCI.

secondary injury

Jim's wife, who is a nurse, asks about the medications Jim has received. You correctly tell her that: 1. Dilantin (phenytoin) was given to prevent _?_ 2. _?_ was given to reduce pressure in the brain

seizures ; Osmitrol (mannitol)

Nursing Management of Aortic Dissection - Pre-op

semi-Fowler's position; quiet environment; Administer opioids and sedatives - managing pain may decrease HR and systolic BP; Administer and titrate IV antihypertensive agents (should have continuous ECG and arterial line); Frequent VS checks, 2-3 minutes if necessary until optimal HR and BP are reached; Check peripheral pulses and check pain, restlessness, LOC and anxiety. (when aortic arch is involved, level of consciousness is affected)

In the spinal cord, the dorsal root carries _?_ information to the _?_

sensory, CNS

A patient with SIRS plus a documented or suspected infection has _?_

sepsis

Surviving Sepsis Campaign: SCREENING FOR SEPSIS AND PERFORMANCE IMPROVEMENT recommend that hospitals and hospital systems have a performance improvement program for sepsis including _?_for acutely ill, high-risk patients. associated with improved patient outcomes

sepsis screening

What 5 lab values will be increased with pancreatitis?

serum amylase serum lipase urinary amylase blood glucose serum triglycerides

During Targeting temp management, you do not want your patient to _?_

shiver

Ongoing monitoring for all types of _?_ Level of consciousness Vital signs, including pulse oximetry, peripheral pulses, capillary refill, skin color and temperature. Watch for mottling. Respiratory status Cardiac rhythm Urine output

shock

_?_ is a syndrome characterized by decreased tissue perfusion and impaired cellular metabolism" (Lewis et al, 2014, p. 1631)

shock

_?_ level of injury is the vertebral level where there is the most damage to vertebral bones and ligaments. _?_ level is the lowest segment of the spinal cord with normal sensory and motor function on both sides of the body

skeletal level neurological level

A major consequence of lack of movement due to SCI is the potential for _?_ .

skin breakdown

Hematogenous metastasis includes the cancer cells passing through lymph nodes; some are trapped, some are not i. If not trapped = _?_

skip metastasis

4 Manifestations of Acromegaly: Neuro

sleep apnea, Peripheral nerve damage (neuropathy), H/A & visual disturbances

If a patient does not have enough calories, _?_ may occur. A burned patient may required _?_ % above his normal intake.

slowed wound healing; 50-100%

What is the most common cause of SIADH?

small cell lung cancer

What are some types of tumors that cause SIADH? Why do they do this?

small cell lung cancer, pancreatic ca, lymphoid ca, thymus ca, colorectal and prostate ca; the tumor secretes ADH

What 3 things may interfere with a dobutamine stress test?

smoking caffeine ingestion beta blockers

Nursing Implementation for Valve Problems: Teach patient and caregiver when to seek medical treatment. Design activity to patient's limitations. Be aware of changes in stool; bruising Discourage _?_ Avoid _?_ activity. Assess heart and breath sounds to monitor effectiveness of medications. Recommend medical alert bracelet.

smoking. strenuous

Strategies to Prevent Liver Disease: No more than two alcoholic drinks a day Stop _?_ Be cautious about mixing drinks, combining with drugs OTC & prescription (< _?_ gm/day Tylenol) Avoid exposure to chemicals (esp. pesticides) whenever possible. Maintain a healthful, balanced diet Lose weight, avoid metabolic syndrome. Vaccinate against _?_ No sharing of needles, razors, toothbrushes. Practice _?_. Caution with medication such as _?_ and _?_ Hep B and C

smoking; less than 2 gm per day of Tylenol hepatitis. safe sex Cava Cava (herbal med) and Amiodarone

About 50% of people with acute SCI experience a temporary neurologic syndrome known as _?_

spinal shock

Approx 50% of patients with acute SCI experience _?_ temporary neurologic syndrome ; Characterized by decreased reflexes, loss of sensation, flaccid paralysis below level of injury; Lasts days to months May mask post-injury neurologic function Patient can start active rehab even during this condition

spinal shock

When treating Addison's disease, teach your patient to _?_ . He should also _?_ the dose of steriods during minor stress (like tooth extraction) and _?_ the dose during major stress (like resp. infection, surgery and trauma)

stand slowly due to hypotension; double the dose triple the dose

CABG requires _?_ (opening of sternum) and _?_

sternotomy Cardiopulmonary bypass

Erythropoietin (Epogen, Procrit) is a hematopoietic growth factor which _?_. Its side effects are HTN, HA, thrombosis

stimulates RBC production

Filgrastim (Neupogen) is a hematopoietic growth factor which _?_. Its side effects are bone pain, n/v

stimulates neutrophil production

Interleukin-11 (Neumega) is a hematopoietic growth factor which _?_. Its side effects are fluid retention, edema, tachycardia, dyspnea, mouth sores and nausea

stimulates platelet growth

PTH regulates calcium and phosphate levels by _?_, _?_ and _?_

stimulating bone resorption of calcium Renal tubular reabsorption of calcium activation of vit D

Nursing Implementation for Valve Problems: Prevention of rheumatic valvular disease by diagnosing and treating _?_ infection Providing prophylactic antibiotics for patients with history Patient with history of _?_ must also be treated with prophylactic antibiotics

streptococcal; endocarditis

Catecholamines are essential for the _?_ response

stress response

An Addisonian Crisis may be triggered by 3 things:

stress, sudden withdrawal of corticosteroid replacement therapy, after adrenal or pituitary destruction

Risk factors for _?_ include: Malignancy (metastisis) OB conditions (retained dead fetus)

subacute DIC

Upper GI Bleed Characteristics: Usually _?_onset Severity depends on origin: venous/capillary/ arterial (arterial=_?_ bleeding) Massive upper GI bleed: loss of more than _?_ of blood or 25% of intravascular blood volume

sudden Bright red, profuse 1500 ml

_?_ is unexpected death resulting from a variety of cardiac causes

sudden cardiac death (SCD)

When thinking of the 3 classes of steroid hormones synthesized by the adrenal cortex, think _?_, _?_ and _?_ to help you remember

sugar, salt, and sex

Emotional /psychologic needs of patients and caregivers Complex care that may require much _?_. Patient may experience many emotions: (list 5) Self-esteem may be affected. Open and frequent communication with patient, care-givers, close friends, and the burn team members.

support and counseling; fear, anxiety, anger, guilt, and depression.

Patients who have had a supraglottic laryngectomy must learn the _?_to decrease the risk of aspiration (due to the removal of the epiglottis). Pureed foods may be a better option because they are thicker (more control during swallowing) Bland foods may be better tolerated. Must eat with the head of the bed elevated (high Fowler's position)

supraglottic swallow

After _?_ it is necessary to monitor total protein, albumin, pre-albumin and f/e. The patient may still be NPO when they return home.

suproglottic laryngectomy

Pt with _?_ must learn the supraglottic swallow to decrease the risk of aspiration (due to the removal of the epiglottis). Pureed foods may be a better option because they are thicker (more control during swallowing) Bland foods may be better tolerated. Must eat with the head of the bed elevated (high Fowler's position).

suproglottic laryngectomy

Treatment for Cushing syndrome: If pituitary tumor, treat with _?_

surgery (transphenoidal approach)

_?_ is indicated when upper GI bleeding continues regardless of therapy and when the site of bleeding has been identified

surgical therapy

Pre-op instructions for phenochromocytoma surgery:

sympathetic blocking agents (alpha and beta-blockers)

_?_ is a type of drug class which mediates peripheral vasoconstriction through the stimulation of alpha receptors

sympathomimetic agents

Aortic valve replacement is done for: _?_ patients with severe AS (AoV area < 1.0 cm2, mean gradient > 40 mmHg, PV > 4.0 m/s) The only definitive treatment for Aortic Stenosis is _?_ Medical management is only temporary and mortality approaches 100% at_?_ after onset of symptoms.

symptomatic AVR 2 years

The _?_ mode is only used with a patient who has spontaneous breathing. Mandatory machine-assisted breaths are delivered in between the patient's spontaneous breaths. There is a preset breathing rate and a preset tidal volume or pressure.

synchronized intermittent mandatory ventilation (SIMV) mode

_?_ improves survival and neurologic outcome in patients successfully resuscitated from cardiac arrest

targeted temperature management

_?_ is an active treatment which tries to achieve and maintain a specific body temperature in an effort to improve health outcomes during the recovery period after blood flow to brain has been stopped (ex: during MI or stroke)

targeted temperature managment

_?_ is paralysis of all 4 extremities; _?_ is paralysis of the legs

tetraplegia paraplegia

In the Diuretic phase of AKI, diuresis indicates _?_

that the nephrons have recovered sufficiently to permit urine excretion

What is the closed box theory?

the 3 components in the skull (brain tissue, blood and CSF) are in a state of dynamic equilibrium. If one changes, the others must compensation since the bones of the skull are fused (i.e. a closed box)

The Recovery Phase of AKI: Begins when _?_

the GFR and tubular function have recovered to the extent that BUN and serum creatinine levels start to stabilize and then ↓.

Removal of Chest tube: 1. Instruct the patient to do _?_ 2. Chest tube is clamped and quickly removed by the physician or midlevel provider 3. _?_

the Valsalva maneuver (or take deep breath and hold dependent on MD preference); Occlusive dressing is applied to the site

Phases of Burn Management: Prehospital care priority to remove pt from source of burn, stop _?_ Remove pt from electrical or chemical source Small thermal burns (10% or <) cover with _?_..comfort for pt and protection

the burning process; clean, cool, tap-water dampened towel.

With abdominal aortic aneurysms (AAA), most occur below _?_

the renal arteries.

How do thiazide diuretics help treat nephrogenic DI?

they decrease GFR allowing more water to be reabsorbed

Gerontologic considerations Burns are deeper due to the _?_, loss of elastic fibers, reduction of _?_ , and a decrease in the blood flow - all lead to poorer rates of healing. More complications due to pre-existing medical conditions. _?_ is a frequent complication. Weaning from ventilator may be challenging. _?_ may occur. More challenging rehabilitation phase. _?_ is an important priority.

thinning of the dermal layer, subcutaneous adipose tissue Pneumonia Delirium Prevention

16 Manifestations of SIADH:

thirst fatigue, Hyponatremia causes muscle cramps, weakness, pain, ↓urine output ↑ body weight, Dyspnea on exertion, Cerebral edema occurs leading to lethargy, anorexia, abdominal pain, confusion, H/A, muscle twitching, seizures, coma

Electrical Burns: Result from intense heat from an electrical current. Direct damage to nerves, vessels, and tissues may lead to _?_ and _?_ Current passing through major organs (brain, heart, kidneys) may produce life threatening sequelae

tissue anoxia and death.

shock is a state of inadequate circulation and perfusion with subsequent_?_

tissue hypoxia.

In the anatomic classification of tumors, the tumor is identified by _?_, _?_ and _?_

tissue of origin anatomic site behavior of the tumor (benign vs malignant)

Why will a patient who has hyperparathyroidism need to increase fluids?

to decrease risk of kidney stones

Why might diuretics be used in the treatment of hyper-parathyroidism?

to increase the excretion of calcium

What is the purpose of a Swan cath?

to maximize CO by adjusting HR, preload, afterload and contractility

Post-aneurysm surgery Nursing Interventions Health promotion:

to promote overall health, encourage the patient to reduce CVD risk factors. controlling blood pressure, ceasing tobacco abuse. increasing physical activity. maintaining normal body weight, Maintaining normal serum lipid levels, evaluation of expected outcomes post aortic surgery , Patent airway maintained, Patent arterial graft with adequate distal perfusion, Adequate urine output, No sign of infection

Besides Declomycin, what other medications may be used for SIADH? What kind of medications are they? Who are they administered to?

tolvaptan Conviptan; vasopressin receptor antagonists; euvolemic hyponatremic hospitalized patients

If in ascending aorta: need _?_cardiopulmonary bypass

total

After _?_, the patient is unable to smell, sneeze, swim or smoke

total laryngectomy

False aneurysms may result from _?_, infection, _?_(at the site of the anastomosis of graft to artery), or arterial leakage after removal of cannulae (femoral artery catheters, intraaortic balloon pump devices)

trauma, peripheral artery bypass graft surgery

Hemothorax: may see after thoracic surgery or _?_ Hemothorax is best seen in _?_ Any accumulation of fluid that hides the _?_ is enough to require drainage

trauma; upright CXR; costophrenic angle

What is the method of action for Cinacalcet (Sensipar)? What are 4 side effects?

tricks parathyroid into releasing less PTH joint/muscle pain respiratory infection n/v diarrhea

Preop instructions for phenochromocytoma surgery: Surgical removal of _?_

tumor (adrenalectomy

Mutations cause these genes to lose their ability to suppress

tumor suppressor genes

_?_ - results when there is an osmotic gradient or pressure gradient across the membrane

ultrafiltration

In cerebral hemorrhage, the Hemorrhage/Hematoma manifests as a space occupying lesion accompanied by _?_ and hemiplegia on _?_side and dilated pupil on _?_ side

unconsciousness; contralateral; ipsilateral

Clinical Manifestations and Assessment Findings of Secondary Pulmonary Arterial Hypertension (SPAH) Symptoms d/t _?_ , some are from the _?_ .

underlying disease; SPAH

_?_ is a syndrome that incorporates all the disturbances seen in various systems throughout the body in chronic kidney failure --- all systems affected

uremia

_?_ retention is a common development in acute SCI and spinal shock. While the patient is in spinal shock, the bladder is _?_ and becomes overdistended.

urinary retention; atonic

In the early stages of CKD, patients usually do not report changes in _?_ . As the disease progresses, patients will have increasing difficulty with _?_ and require diuretics. Once on dialysis patients may develop _?_.

urine output; fluid retention anuria

Describe the "diuretics" portion of SIADH treatment:

use loop diuretics (such as Lasix) if Na is at least 125 mEq/l and treatment is for IICP (increased intracranial pressure)

3 musculoskeletal clinical manifestations of CKD

vascular and soft tissue calcification osteomalacia osteotis fibrosia

_?_ is due to loss of vasomotor tone caused by injury Characterized by hypotension and bradycardia Peripheral vasodilation, venous pooling, decreased cardiac output caused by loss of sympathetic nervous system innervation Generally seen in cervical or high thoracic injury

vasogenic/neurogenic shock

Septic shock is defined as sepsis + _?_ and _?_ in the absence of hypovolemia

vasopressors to maintain MAP greater than or equal to 65; serum lactate greater than or equal to 18

Pulse oximetry reveals nothing about _?_; is more about _?_

ventilation; perfusion

Restrictive cardiomyopathy results in restriction of filling of the _?_ which impairs _?_ filling and stretch

ventricles diastolic

Dilated cardiomyopathy is characterized by rapid degeneration of myocardial fibers resulting in _?_ dilation, impairment of _?_ function, _?_ enlargement, and stasis of blood in the _?_

ventricular systolic atrial left ventricle

Exact cause of Guillain Barre Syndrome still unclear Syndrome often preceded by immune system stimulation from _?_, _?_ or _?_ _?_ caused by camphlobacter jejuni bacteria in about 30% of GBS cases _?_ infection frequently

viral infection, trauma, surgery, viral immunization Gastroenteritis Upper resp tract

Etiology and Pathophysiology of acute pericarditis: 4 types of infectious agents:

viral, bacterial, fungal, other (toxoplasmosis, Lyme disease)

Electrical Burns: Severity depends upon _?_, tissue resistance, current pathways, surface area, and _?_of the current flow. Difficult to assess because below skin. Pt at risk for _?_

voltage length of time dysrhythmias

Nursing Care Pre-Dialysis: 5 methods to assess fluid status:

weight, B/P, peripheral edema, lung & heart sounds

In the spinal cord _?_ matter consists of axons carrying info to and from the brain; _?_ matter consists of sensory and motor nuclei

white grey

What will a Chest x-ray of aortic dissection reveal? (2 things)

widening of the aorta and pleural effusion

Idiopathic Pulmonary Arterial Hypertension(IPAH) Appears _?_ If untreated, can be rapid in progression and cause_?_ and death with a few years. Incurable Increased survival rate d/t drug therapy

without an apparent cause; right-sided heart failure

Name 3 diagnostic tools useful in determining GI perforation:

xray = free gas/air in abdominal cavity; CT = visualization of perforation; WBC elevated

Pheochromocytoma characteristics: It is seen in _?_ age

young-middle

Once you determine the amount of fluids to give in the first 24 hours post-burn, how are they given?

½ of total in first 8 hours ¼ of total in second 8 hours ¼ of total in third 8 hours

Growth of aneurysm may be lowered with _?_, _?_ and/or _?_

ß-adrenergic blocking agents (propranolol), statins, and antibiotics such as doxycycline.

AKI Usually results in _?_ urine output

↓ urine output although it is possible to have normal or increased urine output.

Treatment of Hyperparathyroid Treatment focuses on: _?_ serum calcium levels _?_ PTH calcium _?_ phosphorous _?_ alkaline phosphatase levels (if bone disease); Renal function (_?_ and _?_) X-rays - assesses metabolic bone loss; Avoid immobility; High fluid intake and moderate calcium intake; May replace _?_

↓serum calcium levels ↓ PTH, calcium (< 10), phosphorous (<3), and alkaline phosphatase levels (if bone disease); Renal function (creatinine and BUN, as they increase); X-rays - assesses metabolic bone loss; Avoid immobility; High fluid intake and moderate calcium intake; May replace phosphorous

The patient has been newly diagnosed with cancer. Which statement by the nurse best encourages the patient to share feelings about the diagnosis? "You must share your feelings to help you cope effectively with your cancer." "How are you feeling about your diagnosis?" "It is normal to have difficulty talking about it." "Do you feel depressed about having cancer?"

"How are you feeling about your diagnosis?"

When teaching the patient with acute hepatitis C (HCV), the patient demonstrates understanding when the patient makes which statement? "I will use care when kissing my wife to prevent giving it to her." "I will need to take adofevir (Hepsera) to prevent chronic HCV." "Now that I have had HCV, I will have immunity and not get it again." "I will need to be checked for chronic HCV and other liver problems."

"I will need to be checked for chronic HCV and other liver problems."

A patient is taking an antacid for treatment of a peptic ulcer. Which of the following statements best indicates that the patient understands how to correctly take the antacid? "I should take my antacid before I take my other medications." "I need to decrease my intake of fluids so that I don't dilute the effects of my antacid." "My antacid will be most effective if I take it whenever I experience stomach pains." "It is best for me to take my antacid 1-3hrs after meals."

"It is best for me to take my antacid 1-3hrs after meals."

Which statement by the nurse regarding continuous ambulatory peritoneal dialysis (CAPD) would be of highest priority when teaching a patient new to this procedure? "It is essential that you maintain aseptic technique to prevent peritonitis." "You will be allowed a more liberal protein diet once you complete CAPD." "It is important for you to maintain a daily written record of blood pressure and weight." "You will need to continue regular medical and nursing follow-up visits while performing CAPD.

"It is essential that you maintain aseptic technique to prevent peritonitis."

The patient is hospitalized with neutropenia. Which statement by the patient indicates a correct understanding about injury and infection precautions? "Visitors who have a cold must wear a gown." "The nurses must wear sterile gloves if they come into contact with me." "I must be on contact and respiratory isolation." "Nurses must wash their hands before and after caring for me."

"Nurses must wash their hands before and after caring for me."

The nurse instructs a 50-year-old woman about cholestyramine to reduce pruritis caused by gallbladder disease. Which statement by the patient to the nurse indicates she understands the instructions? "This medication will help me digest fats and fat-soluble vitamins." "I will apply the medicated lotion sparingly to the areas where I itch." "The medication is a powder and needs to be mixed with milk or juice." "I should take this medication on an empty stomach at the same time each day."

"The medication is a powder and needs to be mixed with milk or juice."

A patient with a peptic ulcer is diagnosed with a Helicobacter pylori infection. The nurse is teaching the patient about the meds prescribed clarithromycin (biaxin), esomeprazole (nexium), and amoxicillin (amoxil). Which statement by the patient indicates the best understanding of the medication regimen? "My ulcer will heal because these medications will kill the bacteria." "These meds are only taken when I have pain from my ulcer." "The meds will kill the bacteria and stop the acid production." "These meds will coat the ulcer and decrease the acid production in my stomach."

"The meds will kill the bacteria and stop the acid production."

Aortic Dissection - Often misnamed _?_- NOT a type of aneurysm

"dissecting aneurysm"

Stenotic mitral valve assumes-?_ shape caused by thickening and shortening of valve strictures. Result = _?_ ↑ _?_ pressure and volume Hypertrophy of pulmonary vessels Chronic left atrial pressure elevations

"fish mouth" Obstruction left atrial

Viral carcinogens are termed _?_ .

"oncogenic"

The signs and symptoms of primary hyperparathyroidism are those of hypercalcemia. They are classically summarized by: _?_, _?_, _?_, _?_ and _?_

"stones, bones, abdominal groans, thrones and psychiatric overtones".

Many patient who have had a "sudden brush death" develop a _?_ mentality because they fear the recurrence of cardiac arrest.

"time-bomb"

Surviving Sepsis Campaign: Adding either vasopressin (up to _?_ U/min) or epinephrine to norepinephrine with the intent of raising MAP to target, or adding vasopressin (up to 0.03 U/min) to decrease norepinephrine dosage. against using IV _?_ to treat septic shock if adequate fluid resuscitation and vasopressor therapy are able to restore hemodynamic stability. Use steroids if hemodynamically unstable after fluid resuscitation and vasopressor use (hydrocortisone _?_ mg IV q day

0.03 hydrocortisone 200

In secondary injury, petechial hemorrhages are in central gray matter of cord shortly after injury; Appear within _?_ _?_- Resulting hypoxia, decreased oxygen tension below level that meets metabolic needs causes lactate release Infarction- _?_ later may be infarction in gray matter

1 hour'; Metabolites 4 hours

3 characteristics of Dilutional hyponatremia are: 1) serum sodium of _?_ 2) serum osmo of _?_ 3) urine SpGr of _?_

1) serum sodium < 134 meQ/l 2) serum osmo < 280mOsm/kg 3) urine SpGr > 1.025

Rating deep tendon reflexes: Deep tendon reflexes are normal if they are _?_, _?_ or _?_unless they are asymmetric or there is a dramatic difference between the arms and the legs. Reflexes rated as_?_, _?_ or _?_ are usually considered abnormal. In addition to clonus, other signs of hyperreflexia include spreading of reflexes to other muscles not directly being tested and crossed adduction of the opposite leg when the medial aspect of the knee is tapped.

1+, 2+, or 3+ 0, 4+, or 5+

Clinical manifestations of hepatitis classified into 2 phases: Acute & Chronic Acute: lasts_?_ Many will be Asymptomatic If there are s/s, they will be:

1-4 months Malaise Anorexia, nausea/vomiting Abdominal pain, hepatomegaly Can be icteric or anicteric If icteric jaundice, dark urine, light stools, pruritus

The client arrives at the ED following a burn injury that occurred in the basement at home and an inhalation injury is suspected. Which of the following would the nurse anticipate to be prescribed for the client initially? 1. 100% oxygen via an aerosol mask 2. oxygen via nasal cannula at 15L/min 3. oxygen via nasal cannula at 10L/min 4. 100% oxygen via a tight-fitting, nonrebreather face mask

1. 100% oxygen via an aerosol mask

The Referral Process to GOH: It is the responsibility of hospital staff to initiate the organ donation process when: 1. A patient begins to lose _?_ 2. _?_ 3. A patient dies after _?_

1. A patient begins to lose brain stem reflexes 2. The discussion to remove the patient from life support is planned 3. A patient dies after a cardio-pulmonary arrest

During the nursing assessment of a patient with an aortic stenosis, the nurse would expect to find: 1. A systolic murmur. 2. Systolic ejection click. 3. Pericardial friction rub. 4. Brisk, hammering pulses.

1. A systolic murmur.

#6 Collaborative Care -- heart punctured by rib 1. 2 3 4 5

1. ABC's 2. Echo if hemodynamics are unstable 3. fluid and electrolyte balance 4. EKG monitoring 5. pain control

4 Causes of SIADH:

1. ADH secreting tumors (usually cause chronic SIADH) 2. Head trauma and CNS disorders 3. Some medications ; 4. Infections:

Hyperglycemic-hyperosmolar State (HSS) - 6 characteristics:

1. Acute, life-threatening condition 2. Characterized by profound hyperglycemia (greater than 600 mg/dL) 3. hyperosmolarity that leads to dehydration 4. absence of ketosis 5. Gradual onset over several days 6. If left untreated, can lead to coma and death

6 steps of Treatment for DKA

1. Airway management 2. Oxygen administration 3. Large bore IV 4. Correct fluid/electrolyte imbalance: IV infusion 0.45% or 0.9% NaCl 1L/hr (restore urine output/raise blood pressure) 5. When blood glucose levels approach 250 mg/dL: 5% dextrose added to regimen to prevent hypoglycemia. 6. Administer needed electrolytes: Potassium replacement-must watch K level; if pt hypokalemic, giving insulin will cause further decrease in K level

Treatment for DKA: 1. Airway management 2. Oxygen administration 3. Large bore IV 4. Correct fluid/electrolyte imbalance: IV infusion_?_ (restore urine output/raise blood pressure) 5. When blood glucose levels approach _?_, add _?_ to regimen to prevent hypoglycemia. 6. Administer needed electrolytes: Potassium replacement-must watch K level; if pt hypokalemic, giving _?_ will cause further decrease in K level

1. Airway management 2. Oxygen administration 3. Large bore IV 4. Correct fluid/electrolyte imbalance: IV infusion 0.45% or 0.9% NaCl 1L/hr (restore urine output/raise blood pressure) 5. When blood glucose levels approach 250 mg/dL: 5% dextrose added to regimen to prevent hypoglycemia. 6. Administer needed electrolytes: Potassium replacement-must watch K level; if pt hypokalemic, giving insulin will cause further decrease in K level

5 components of Nursing Care for HHS/DKA:

1. Always treat the underlying cause (infection), 2. Rapid isotonic fluid (0.9% NS) 3. Follow with hypotonic fluid (0.45% NS) 4. When serum glucose levels approach 250 mg/dL, add glucose to IV fluids to minimize the risk of cerebral edema associated with drastic changes in serum osmolarity and prevent hypoglycemia 5. Administer regular insulin (Humulin R) 0.1 unit/kg as an IV bolus dose and then follow with a continuous IV infusion of regular insulin at 0.1 unit/kg/hr.

What are 4 hematologic complications associated with ARDS?

1. Anemia 2. Disseminated intravascular coagulation (DIC) 3. Thrombocytopenia 4. Venous thromboembolism

7 Hypophysectomy Post-op Instructions:

1. Avoid vigorous coughing, sneezing, and straining 2. Avoid tooth brushing for 10 days 3. Good oral care 4. HOB elevated 5. Send clear nasal drainage to lab 6. Bacitracin nose drops 7. Hormone replacement

Name 3 drugs that may be used to treat Hyperparathyroidism

1. Bisphosphonates (Fosamax); 2. Cinacalcet (Sensipar) 3. Diuretics

Name 2 Endothelin Receptor Antagonists used to treat P HTN and give their mechanism of action

1. Bosentan 2. Ambrisentan reduce PAP, PVR and MAP Increase CI

What are 6 broad classes of meds used for Pulmonary HTN?

1. CCB 2. Phosphodiesterase (Type 5) enzyme inhibitors 3. Vasodilators (parenteral) 4. Vasodilators (inhaled) 5. Endothelin Receptor Antagonists 6. PO vasodilators

Factors that affect cerebral blood flow are: 1. _?_ 2. _?_ 3. Hydrogen Ion Concentration 4. Trauma, Tumors, Hemorrhage, Stroke

1. CO2 2. O2

What 5 types medications might be prescribed for a patient with AKI?

1. Ca supplements 2. Phosphate binders 3. PPI/H2 blocker 4. Treat low BP with drugs to increase squeeze 5. Lower K using insulin/bicarb or diuretics

1. Cancer proliferates without _?_ 2. Cancer cells do not respect _?_ 3. Cancer cells may not proliferate more rapidly than normal cells but they may _?_

1. Cancer proliferates even without degeneration 2. Cancer cells do not respect the boundaries of other cells 3. Rate of proliferation in cancer cells is not more rapid than normal cells, but they may produce more than 2 cells per mitosis cycle = pyramid effect

Six types of incomplete spinal cord injury:

1. Central cord syndrome 2. Anterior Cord Syndrome 3. Brown-Sequard syndrome 4. Posterior cord syndrome 5. Conus Medullaris Syndrome 6. Cauda Equina Syndrome

Carcinogens - most can be detoxified and excreted. 3 Types:

1. Chemical carcinogens 2. Radiation 3. Viral

4 components of treatment of DI:

1. Correct underlying cause - essential 2. IV and oral fluids (depending on patient condition) 3. Medications 4. Patient teaching for long term management

Filling in the following for the progression of increase of ICP 1. 2. Tissue Edema 3. 4. Compression of Ventricles and Blood Vessels 5. Decreased Cerebral Blood Flow (CBF) 6. Decreased O2 with death of brain cells 7. Edema around necrotic tissue 8. 9. Accumulation of CO2 10. Vasodilation 11. 12. Death

1. Cranial Assault 2. Tissue Edema 3. Increased ICP 4. Compression of Ventricles and Blood Vessels 5. Decreased Cerebral Blood Flow (CBF) 6. Decreased O2 with death of brain cells 7. Edema around necrotic tissue 8. Increased ICP with compression of brain stem & respiratory center 9. Accumulation of CO2 10. Vasodilation 11. Increased ICP d/t high blood volume 12. Death

Filling in the following for the progression of increase of ICP: 1. Cranial Assault 2. 3. Increased ICP 4. Compression of Ventricles and Blood Vessels 5. Decreased Cerebral Blood Flow (CBF) 6. Decreased O2 with death of brain cells 7. 8. Increased ICP with compression of brain stem & respiratory center 9. Accumulation of CO2 10. 11. Increased ICP d/t high blood volume 12. Death

1. Cranial Assault 2. Tissue Edema 3. Increased ICP 4. Compression of Ventricles and Blood Vessels 5. Decreased Cerebral Blood Flow (CBF) 6. Decreased O2 with death of brain cells 7. Edema around necrotic tissue 8. Increased ICP with compression of brain stem & respiratory center 9. Accumulation of CO2 10. Vasodilation 11. Increased ICP d/t high blood volume 12. Death

Filling in the following for the progression of increase of ICP: 1. Cranial Assault 2. Tissue Edema 3. Increased ICP 4. 5. Decreased Cerebral Blood Flow (CBF) 6. 7. Edema around necrotic tissue 8. 9. Accumulation of CO2 10. Vasodilation 11. Increased ICP d/t high blood volume 12. Death

1. Cranial Assault 2. Tissue Edema 3. Increased ICP 4. Compression of Ventricles and Blood Vessels 5. Decreased Cerebral Blood Flow (CBF) 6. Decreased O2 with death of brain cells 7. Edema around necrotic tissue 8. Increased ICP with compression of brain stem & respiratory center 9. Accumulation of CO2 10. Vasodilation 11. Increased ICP d/t high blood volume 12. Death

Filling in the following for the progression of increase of ICP: 1. Cranial Assault 2. Tissue Edema 3. Increased ICP 4. Compression of Ventricles and Blood Vessels 5. 6. Decreased O2 with death of brain cells 7. Edema around necrotic tissue 8. Increased ICP with compression of brain stem & respiratory center 9. 10. Vasodilation 11. Increased ICP d/t high blood volume 12.

1. Cranial Assault 2. Tissue Edema 3. Increased ICP 4. Compression of Ventricles and Blood Vessels 5. Decreased Cerebral Blood Flow (CBF) 6. Decreased O2 with death of brain cells 7. Edema around necrotic tissue 8. Increased ICP with compression of brain stem & respiratory center 9. Accumulation of CO2 10. Vasodilation 11. Increased ICP d/t high blood volume 12. Death

Name 4 medications that may be useful in treating DI

1. Desmopressin acetate (DDAVP) an analog of ADH given PO or as a spray 2. Vasopressin (Pitressin) 3. Carbamazepine (Tegretol - antiseizure med 4. chlorpropamide (Diabenese; an anti-diabetic med)

Pharmacological therapy for IPAH: 1. _?_ promote vasodilation of pulmonary blood vessels, reduce right ventricular overload, and reverse remodeling. 2. _?_ for peripheral edema. 3. _?_ for prevention of thrombus formation.

1. Drug therapy 2. Diuretics 3. anticoagulants

List 5 s/s of hypercapnia

1. Dyspnea 2. tripod position 3. pursed lip breathing 4. decreased RR 5. decreased tidal volume

What are 2 cardiac complications associated with ARDS?

1. Dysrhythmias 2. Decreased CO

Two of the strongest predictors of sudden cardiac death (SCD) are:

1. Ef less than 30% 2. ventricular dysrhythmias following MI

What are the 3 areas of the Glasgow Coma Scale?

1. Eyes Opening 2. Verbal Response 3. Motor response

What are the 5 components of a ventilator bundle?

1. HOB 30-45 degrees 2. Daily sedation holiday 3. Peptic ulcer prophylaxis 4. Venous thromboembolism prophylaxis 5. Daily oral care with chlorhexidine

7 Clinical Manifestations of Cushing Syndrome

1. HTN (excess mineralcorticoid), secondary to fluid retention ; 2. Menstrual irregularities; 3. Hirsutism in women; 4. Gynecomastia & impotence in men; 5. Hypokalemia; 6. Inhibition of immune response; 7. Glucose metabolism altered - glucose intolerance and ↑gluconeogenesis by the liver

3 characteristics of Addison's Disease:

1. Hypofunction of the adrenal cortex; 2. Autoimmune disorder resulting in antibodies destroying adrenal gland 3. All 3 classes of corticosteroids are deficient

Client Education regarding HHS/DKA; 5 indications for patient to call MD:

1. Illness last more than 1 day 2. Blood glucose is greater than 240 mg/dL, 3. Patient is unable to tolerate food or fluids 4. Ketones are found in urine for more than 24 hours 5. Temperature for 24 hours of 38.6 °C. (101.5 °F.)

Name 2 Vasodilators (inhaled) used for P HTN and give the mechanism of action:

1. Iloprost 2. Treprostinil Dilates systemic and pulmonary vasculature

What are the 2 major complications of increased ICP

1. Inadequate cerebral perfusion 2. Cerebral herniation

3 stages in the process of Metastasis are:

1. Increase in size, penetrate basement membrane 2. Angiogenesis - development of blood supply 3. Hematogenous metastasis

Vital Signs - Cushing's Triad 1. 2. 3. Changes in body temperature may also be noted because of increased ICP impacting the hypothalamus

1. Increasing systolic blood pressure causing a widening pulse pressure 2. Bradycardia 3. Irregular Respiratory Pattern

Airway suctioning is a routine part of Jim's care, since he has assisted ventilation and is intubated. What is the appropriate way to suction a patient with a basilar skull fracture?

1. Jim should be hyperoxygenated but not hyperventilated before and after suctioning 2. Jim should be suctioned for no longer than 10 seconds at a time

2 Medications for Cushing Syndrome are _?_ and _?_.. Both are essentially a medical _?_ . What is their method of action?

1. Ketoconazole (Nizoral) 2. Aminoglutethimide (Cytadren) adrenalectomy: both inhibit corticosteroid synthesis

5 stages of CKD

1. Kidney damage with normal GFR(slow progression) 2. Kidney damage with slightly ↓ GFR (60-89)- watch progression carefully 3. Moderate ↓ in GFR (30-59)—evaluation and treating complications 4. Severe ↓in GFR (15-29)- prepare for kidney replacement therapy 5. Kidney failure GFR (< 15)-- dialyze

3 Risk factors for DKA

1. Lack of sufficient insulin r/t undiagnosed/ untreated type 1 diabetes mellitus or nonadherence to a diabetic regimen; 2. Reduced or missed dose of insulin(insufficient or error in dosing; 3. Any condition that increases carbohydrate metabolism: physical/emotional stress, illness/infection (#1 cause of DKA), surgery, or trauma that requires an increased need for insulin

The following cardiac factors, along with adequate urine output, let us know how our patient is doing: 1. MAP _?_ 2. BP _?_ 3. HR _?_ How are MAP and BP best measured?

1. MAP > 65 2. BP (systolic) > 90 3. HR < 120 bpm Art line

What are the 5 drug types used with IICP?

1. Mannitol 2. corticosteroids (if d/t tumor/abscess) 3. barbiturates 4. antiseizure 5. antibiotics

Treatment for HHS

1. Medical emergency with High mortality rate 2. Therapy similar to DKA except HHS requires greater fluid replacement 3. Patient-centered care - 4. Nursing care - Monitor blood glucose hourly. Blood glucose of less than 200 mg/dL is the goal for resolution. 6. Monitor serum potassium levels. Potassium levels will initially be increased with insulin therapy, but potassium will shift into cells and the client will need to be monitored for hypokalemia. 7. Administer sodium bicarbonate by slow IV infusion for severe acidosis (pH of less than 7.0). Monitor potassium levels because correcting acidosis too quickly may lead to hypokalemia.

Treatment for HHS: 1. Medical emergency with High mortality rate 2. Therapy similar to DKA except HHS requires greater_?_ 3. Patient-centered care - 4. Nursing care - Monitor blood glucose hourly. Blood glucose of less than_?_ is the goal for resolution. 6. Monitor serum _?_ levels (will initially be increased with insulin therapy, but will shift into cells and the client will need to be monitored for _?_ _ 7. Administer _?_ by slow IV infusion for severe acidosis (pH of less than 7.0). Monitor potassium levels because correcting acidosis too quickly may lead to hypokalemia.

1. Medical emergency with High mortality rate 2. Therapy similar to DKA except HHS requires greater fluid replacement 3. Patient-centered care - 4. Nursing care - Monitor blood glucose hourly. Blood glucose of less than 200 mg/dL is the goal for resolution. 6. Monitor serum potassium levels. Potassium levels will initially be increased with insulin therapy, but potassium will shift into cells and the client will need to be monitored for hypokalemia. 7. Administer sodium bicarbonate by slow IV infusion for severe acidosis (pH of less than 7.0). Monitor potassium levels because correcting acidosis too quickly may lead to hypokalemia.

What are the 3 components of treatment for ileus/paralytic ileus:

1. NG tube 2. Reglan to speed up peristalsis 3. f/e replacement

3 types of Diabetes Insipidus (DI):

1. Neurogenic DI (aka central) 2. Nephrogenic DI, 3. Psychogenic DI (aka primary)

What are 5 respiratory complications associated with ARDS?

1. O2 toxicity 2. Pulmonary barotrauma such as pneumothorax 3. Pulmonary emboli 4. pulmonary fibrosis 5. VAP

6 Risk Factors for HHS

1. Older adults who have inadequate fluid intake and become dehydrated experience osmotic diuresis due to high blood glucose. 2. Older adults who have decreased kidney function are unable to excrete the excess glucose into urine, with resulting high blood glucose 3. Medical conditions such as myocardial infarction, cerebral vascular injury, or sepsis. 4. Certain medications- glucocorticoids, diuretics, phenytoin, propranolol, and calcium channel blockers. 5. Infection 6. stress

3 Phases which mark the clinical course of intrarenal AkI

1. Oliguric phase 2. Diuretic phase 3. Recovery phase

Collaborative intervention #3 -- (flail chest) ______________________________ ______________________________ ______________________________ _______________________________

1. Oxygen first! 2. monitor ABC's 3. May require mechanical vent 4. pain control (PCA pump, epideral, nerve block)

What are the 4 phases of burn management?

1. Prehospital care 2. Emergent/resuscitation phase 3. Acute phase 4. rehabilitation phase

Name 5 hemodynamic measurements that relate to preload

1. RAP 2. CVP 3. PAWP 4. LAP 5. PAD

What are some specific interventions that should be implemented on intubated ARDS patients?

1. Rotational/prone positioning therapy 2. Changes of vent modes 3. Pain/abx/sedation meds 4. HOB 30-45 degrees

When planning a community education program related to prevention of the cancer with the highest death rates in both women and men, the nurse will stress the importance of: 1. Smoking cessation. 2. Screening with colonoscopy. 3. Regular examination of reproductive organs. 4. Use of sunscreen and protection from ultraviolet light

1. Smoking cessation.

Classification of Burn -- Depth: 1. _?_-thickness burn; Involves the epidermis 2. _?_ -thickness burn; Involves the dermis 3. _?_ - Involves fat, muscle, bone

1. Superficial partial -thickness burn; Involves the epidermis 2.Deep partial-thickness burn; Involves the dermis 3. Full-thickness burn - Involves fat, muscle, bone

5 components of Treatment of hypocalcemia, hypoparathyroid:

1. Treat tetany 2. Rebreathing (paper bag) 3. EKG monitoring; 4. Lifelong treatment with Ca and Vit D; 5. High calcium diet

3 components of treatment of Addisonian Crisis

1. Treat the Cause: Treat with vasopressors, 2. corticosteroids (main is hydrocortisone), 3. IVF (NS or/and D5W)

Besides monitoring intracranial pressure (ICP), you maintain the integrity of Jim's ICP monitoring system. List 2 interventions that help ensure this:

1. Using aseptic technique when handling equipment 2. Maintaining a closed system

What are the 4 main causes of hypoxemia respiratory failure?

1. V/Q mismatch 2. intrapulmonary shunting (ex: ARDS) 3. Diffusion limitation 4. Alveolar hypoventilation

What are 4 causes of hypoxemia?

1. V/Q mismatch 2. shunt 3.. diffusion limitation 4. alveolar hypoventilation

Name at least 4 complications of ARDS due to TREATMENT

1. VAP 2. Barotrauma 3. Infections (Central line/foley) 4. Injury to trachea/pharynx/vocal cords

What are the 4 catergories of diseases that limit ventilation supply?

1. abnormalities of airway/alveoli 2. abnormalities of CNS 3. abnormalities of chest wall 4. neuromuscular conditions

Brain death (neurologic death) involves clinical confirmation of: (4 things)

1. absence of brainstem reflexes 2. absence of motor responses to pain . 3. unresponsiveness 4. apnea

6 complications of thoracic surgery:

1. acute resp failure 2. Prolonged air leak 3. Hemorrhage 4. cardiovascular disturbances 5. tension pneumo 6. infection

List 6 possible complications after thoracic surgery:

1. acute respiratory failure 2. prolonged air leak 3. hemorrhage 4. cardiovascular disturbances 5. tension pneumothorax and tracheal shift 6. infection

Besides administration of too many steroids and ACTH-secreting pituitary tumors, what are 3 other types of tumors which may cause Cushing Syndrome?

1. adrenal tumors 2. lung tumors 3. pancreas tumors

6 types of cell cycle phase - nonspecific chemo agents

1. alkylating 2. Anti-tumor antibiotics 3. nitrosoureas 4. platinum drugs 5. hormone therapy 6. corticosteroids

What are the 3 ways in which cancer is classified?

1. anatomic site 2. histologic grading 3. extend of disease (staging)

What are 5 classes of drugs used with post-op CABG patients?

1. antidysrhythmics 2. Positive or negative inotropes 3. Sympathomimetics 4. vasodilators 5. vasopressures

Name 3 types of cell cycle phase-specific chemo drugs

1. antimetabolites 2. mitotic inhibitors 3. topoisomerase inhibitors

List 7 causes of hypercapnic respiratory failure:

1. asthma 2. brain/spinal cord injury 3. COPD 4. Cystic fibrosis 5. Guillan-Barre 6. Myasthemia gravis 7. Thoracic trauma

List 5 causes of V/Q mismatch:

1. atelectasis 2. bronchiospasm 3. pain 4. pulmonary embolism 5. increased secretions

4 possible purposes of cancer surgery:

1. cure 2. control 3. palliative 4. rehab/reconstruction

What are 3 CNS and Psychologic Complications associated with ARDS?

1. delirium 2. Sleep deprivation 3. PTSD

What is the "three bottle" design of a chest tube

1. drainage bottle (attached to patient's chest tube) 2. water seal 3. suction (depth of water determines amount of suction)

List 8 s/s of hypoxemia

1. dyspnea 2. I:E of 1:3 or 1:4 3. Nasal flare 4. retractions 5. Paradoxic chest 6. cyanosis 7. tachypnea 8. decreased SpO2

Name 3 Vasodilators (parenteral) and give their mechanism of action

1. epoprostenol 2. treprostinil 3. adenosine Promote vasodilation and reduce resistance

List 4 conditions/situations in which the risk for biliary disorders increases

1. estrogen therapy 2. sedentary 3. family history 4. obesity

3 stages of liver damage

1. fatty liver 2. liver fibrosis 3. cirrhosis

What are the 5 major mechanisms of spinal cord injury?

1. flexion 2. hyperextension 3. flexion-rotation 4. extension-rotation 5. compression

What are 3 ways in which the underlying cause may be treated in SIADH?

1. fluid restriction 2. treat hyponatremia 3. diuretics

What do you do if post-CABG your patient develops cardiac tamponade?

1. get chest tube patent 2. back to sx to correct

What are 4 post-operative complications of head/neck surgery?

1. hemorrhage 2 Carotid artery rupture 3. fistulas 4. airway obstruction

Besides an Ef less than 30% and ventricular dysrhythmias following MI, what are 4 other predictors of sudden cardiac death (SCD)

1. history of syncope 2. aortic stenosis 3. males 4. family hx of Ventricular dysrhythmias

After subarachnoid hemorrhage (SAH) due to bleeding from a cerebral aneurysm, patients like Mrs. Thorne are at high risk for 3 complications. Besides rebleeding, complications include:

1. hydrocephalus 2. vasospasm 3. seizure

Diabetic Ketoacidosis (DKA) Acute, life-threatening condition. Characterized by 5 signs and symptoms:

1. hyperglycemia (greater than 300 mg/dL) 2. ketosis 3. acidosis 4. dehydration 5. Rapid onset

What are 3 major complications with intra-aortic balloon pump (IABP)?

1. if balloon ruptures, clot can form inside 2. if stopped for more than 30 minutes, clots can form on outside 3. if balloon migrates distally - bad for kidneys 4. if balloon migrates proximally - occlusion of L. subclavian

3 results of aortic stenosis:

1. left ventricular hypertrophy 2. ↑ myocardial oxygen consumption due to ↑ myocardial mass 3. Leads to ↓ CO and pulmonary hypertension

What are the 3 manners in which SCI are classified?

1. mechanism of injury 2. level of injury 3. degree of injury

Two possible causes of GI ischemia that may lead to necrosis are:

1. mesenteric clot 2. hypotension (decreased perfusion to bowel)

Smoke and inhalation Breathing hot air or noxious chemicals - affects respiratory tract. Three types of smoke and inhalation burn injuries: 1. 2. 3. Needs _?_ , as is major predictor of mortality.

1. metabolic asphyxiation 2. upper airway injury 3. lower airway injury. rapid assessment

Rhabdomyolysis: Clinical Manifestations Course of rhabdomyolysis varies, depending on cause.; Symptoms can occur in one area or affect the whole body.; Complications : occur in early and later stages; The "classic triad" includes _?_, _?_, and _?_

1. muscle pain in the shoulders, thighs, or lower back; 2. muscle weakness or trouble moving arms/legs; 3. dark red or brown urine or decreased urination

Name 4 complications of ascites:

1. pain 2. peritonitis 3. breathing problems 4. decreased CO

What are 4 GI complications associated with ARDS?

1. paralytic ileus 2. Pneumoperitoneum 3. Stress ulceration/hemorrhage 4. Hypermetabolic state

Immediate post-spinal cord injury goals(4)

1. patent airway 2. adequate ventilation 3. adequate circulating blood volume 4. prevention of secondary injury

Collaborative interventions: #5 --cardiac tamponade 1. 2.

1. pericardial thorecentesis 2. pericardial window, if recurrent

4 Causes of Hypopituitarism:

1. pituitary tumor (most common) 2. autoimmune disorders 3. infections 4. pituitary infarction

6 conditions which may require a chest tube:

1. pneumothorax 2. hemothorax 3. pleural effusion 4. closed pneumothorax 5. open pneumothorax 6. tension pneumothorax

Name 6 signs and symptoms of Diabetes Insipidus:

1. polyuria (5-20 L/da) 2. low specific gravity of urine 3. high serum osmolality 4. dehydration symptoms 5. increased sodium 6. mental dullness to coma

SCI's are divided into 2 main categories: _?_ injuries have actual physical disruptions of axons, while _?_ injuries do not.

1. primary 2. secondary

What are the 5 types of adrenocortical insufficiency?

1. primary 2. secondary 3. pituitary disease 4. Tumor 5. abrupt withdrawal of corticosteroid medications

The nurse is preparing to care for a burn client scheduled for an escharotomy being performed for a third-degree circumferential arm burn. The nurse understands that the anticipated therapeutic outcome of the escharotomy is: 1. return of distal pulses 2. brisk bleeding from the site 3. decreasing edema formation 4. formation of granulation tissue

1. return of distal pulses

What are 3 things that must be involved in a diagnostic work-up of SIADH?

1. serum sodium 2. measured (not calculated) serum osmolality 3. urine specific gravity

Name 2 Phosphodiesterase (Type 5) enzyme inhibitors used for P HTN. What is their mechanism of action?

1. sidenafil 2. tadalafil Promote smooth muscle relaxation

Clinically Important Bleeding is defined as: Overt bleeding (hematemesis, hematochezia = maroon colored stools or melena) complicated by one of following within 24 hrs after onset of bleeding if no other causes for these symptoms: 1. spontaneous decrease in SBP of more than _?_ 2. decrease of more than _?_ in SBP with pt sitting up 3. increase of more than _?_ beats/min in HR 4. decrease in hemoglobin level of more than _?_ g/dl

1. spontaneous decrease in SBP of more than 20mmHG 2. decrease of more than 10mmHG in SBP with pt sitting up 3. increase of more than 20 beats/min in HR 4. decrease in hemoglobin level of more than 2g/dl

3 Considerations for older adult patients with HHS:

1. teach older adult patients to check blood sugar every 1-4 hours when ill; 2. emphasize the importance of not skipping an insulin dose when ill; 3. maintain hydration because older adult patients may have a diminished thirst sensation

Persons who experience SCD because of CAD fall into 2 groups:

1. those who did not have an acute MI 2. those who did have an acute MI

Which 6 populations are recommended for CABG?

1. those who failed medical management 2. those who have L main or 3 vessel disease 3. those who can't have PCI 4. failed PCI with chest pain 5. DM 6. those expected to have longer term benefits with CABG than PCI

Name 3 types of "stress" that may send someone into an Addisonian crisis:

1. trauma 2. infection 3. surgery

Chest Tube: Monitor for fluctuation of fluid in water seal chamber If fluctuations stop...question if 1. 2. 3. 4.

1. tube occluded 2. dependent loop 3. suction not working 4. lung re-expanded

The TNM classification system uses three parameters. They are:

1. tumor size and invasiveness (T) 2. presence or absence of regional spread to the lymph nodes (N) 3. metastasis to distant organ sites (M).

Name 4 regions in which ICP can be measured:

1. ventricles 2. Subarachnoid space 3. Epidural space 4. Brain tissue

What 3 things decrease tidaling in the chest tube system?

1. when a large air leak is present, 2. when the lung has reexpanded, or 3. when positive end expiratory pressure (PEEP) is used with a ventilated patient.

When is intra-aortic balloon pump (IABP) weaned? (2)

1. when patient is hemodynamically stable 2. when patient requires little (or no) pharmacological support)

Oliguria of intrarenal failure: sp gr of urine _?_ ; urine osmolality of __?_; and the presences of 4 things:

1.010 and 300 (the same as plasma, indicating tubular damage) with high sodium, RBC, WBC, & casts (of RBC or epithelial cells - indicate nephron damage)

Burn Center Referral 1. Partial thickness burns greater than _?_total body surface area (TBSA). 2. Burns that involve the face, hands, feet, _?_, perineum, or major joints. 3_?_ burns in any age group. 4. _?_, including lightning injury. 5. Chemical burns. 6. Inhalation injury. 7. Burn injury in patients with preexisting medical disorders that could complicate management, prolong recovery, or affect mortality. 8. Any patient with burns and _?_(such as fractures) in which the burn injury poses the greatest risk of morbidity or mortality. 9. Burned children in hospitals without qualified personnel or equipment for the care of children. 10. Burn injury in patients who will require special social, emotional, or rehabilitative intervention and should be referred to a burn center

10% genitalia .Third degree Electrical burns concomitant trauma

GI care post esophageal cancer surgery includes: starting post-pyloric feeding on day 2 post op at a rate of _?_ to _?_ ml/hr. _?_ sets the rate, and _?_ orders the increase.

10-20 dietitian MD

The qSOFA score (also known as quickSOFA) is a bedside prompt that may identify patients with suspected infection who are at greater risk for a poor outcome outside the intensive care unit (ICU). It uses three criteria, assigning one point for low blood pressure (SBP≤_?_ mmHg), high respiratory rate (≥_?_ breaths per min), or altered mentation (Glasgow coma scale<_?_).

100 22 15

In cancer patients, a fever of _?_ or above is a medical emergency

100.4 (38)

During a thoracentesis, how much fluid should be removed at one time? Why?

1000-1200 mL; more may result in hypotension

What is normal respiratory rate

12-20

Normal GFR is about _?_ ml/min., and approximates the urine creatinine clearance.

125

What is a normal GFR

125

Keep hot water heaters set at / below_?_ Caution with steam heaters Avoid cooking hazards Avoid sunburns Caution with microwave foods Beware fast food drive thru and hot drinks

125 degrees F.

Fluid Therapy after burns: IV access - 2 large bore IV for all with >_?_ TBSA burns. For those with >_?_ consider central line, as well as arterial line. Use crystalloids, colloids, or a combination.

15%; 30%,

End-stage renal disease (ESRD), or stage V, occurs when the GFR is < _?_ ml/min. (at this stage _?_ or _?_ is required, if patient is uremic and desires a transplant)

15; dialysis or transplant

What needle sizes count as "large bore"?

18, 16 and 14 gauge

Surviving Sepsis Campaign: Glucose: use protocol to manage blood glucose management in ICU patients with sepsis Start when 2 consecutive blood glucose levels are >_?_ mg/dL. blood glucose values be monitored every 1 to 2 hrs until glucose values and insulin infusion rates stable, then every 4 hrs in patients receiving insulin infusions Nutrition: start enteral feedings early if gut working

180

The normal over-all V/Q is _?_ however some areas of the lungs get more, some less

1:1

Patient has Upper GI Bleed: Endoscopy: Coagulate or thrombose with heat or lasers or may inject with epinephrine _?_ If can't do an endoscopy or bleeding persists after endoscopy may do _?_

1:10,000 angiography

When WBC count is below _?_, stop chemo

2

What is normal CVP? What other measurement corresponds to CVP?

2-8 RAP

A patient is just admitted to the hospital following a spinal cord injury at the level of T4. A priority of nursing care for the patient is monitoring for: 1. Return of reflexes. 2. Bradycardia with hypoxemia. 3. Effects of sensory deprivation. 4. Fluctuations in body temperature.

2. Bradycardia with hypoxemia.

A young adult is hospitalized after an accident that resulted in a complete transection of the spinal cord at the level of C7. The nurse informs the patient that after rehabilitation, the level of function that is most likely to occur is the ability to: 1. Breathe with respiratory support. 2. Drive a vehicle with hand controls. 3. Ambulate with long-leg braces and crutches. 4. Use a powered device to handle eating utensils.

2. Drive a vehicle with hand controls.

To prevent the debilitating cycle of fatigue-depression-fatigue that can occur in patients with cancer, an appropriate nursing intervention is to: 1. Have the patient rest after any major energy expenditure. 2. Encourage the patient to implement a daily walking program. 3. Teach the patient to ignore the fatigue to maintain normal daily activities. 4. Prevent the development of depression by informing the patient to expect fatigue during cancer treatment.

2. Encourage the patient to implement a daily walking program.

When teaching a patient with chronic kidney disease about prevention of complications, the nurse instructs the patient to: 1. Monitor for proteinuria daily with a urine dipstick. 2. Weigh daily and report a gain of greater than 4 pounds. 3. Take calcium-based phosphate binders on an empty stomach. 4. Perform self-catheterization every 4 hours to accurately measure I & O.

2. Weigh daily and report a gain of greater than 4 pounds.

Nurse caring for a client with a chest tube turns the client and the chest tube accidentally disconnects from the water seal chamber. Which initial nursing action should the nurse take? 1. call the health care provider (PCP) 2. place the tube in a bottle of sterile H20 3. immediately replace the chest tube system 4. Place a sterile dressing over the disconnection site

2. place the tube in a bottle of sterile H20

Most corticosteroids are given as follows: _?_ in the morning _?_ in the late afternoon (provided the patient has a "normal" wake/sleep cycle)

2/3 in morning 1/3

Diagnostic Studies-- Adrenal Disease (Cushing): 24-hour urine for _?_; _?_ suppression test - used if 24 hour test (described above) is borderline diagnostic ; Plasma _?_levels (may ↑ due to loss of diurnal variation);

24-hour urine for free cortisol; Dexamethasone suppression test - used if 24 hour free cortisol is borderline diagnostic ; Plasma cortisol levels (may ↑ due to loss of diurnal variation);

Criteria for Pulm HTN is mean pulmonary artery pressure of _?_ at rest or _?_ with exercise. (Normal is _?_ to _?_)

25 at rest 30 with exercise normal 12-16

Pulmonary pressures are elevated mean pulmonary artery pressure > _?_ at rest (normal is 12-16 mm Hg) or >_?_ with exercise S/S: shortness of breath, fatigue

25mm Hg 30 mm Hg

normal osmolality =

278-300mOsm/kg approximately depending on lab parameters

Follow manufacturer's instructions for adding water to the _?_ cm level in the water seal chamber, and to the _?_cm level in the suction control chamber (unless different level is ordered) Connect the 6 foot patient tube to the _?_. Connect the drain to _?_ , and slowly increase suction until gentle bubbling appears in the suction control chamber

2; 20 thoracic catheter; vacuum

GI care post esophageal cancer surgery: NPO for _?_ to _?_ - prevent anastomotic leak or fistula formation No oral meds - crush & give via J tube Freq oral care (at least q2hrs)- improves comfort, reduces risk of infection (while intubated & NPO) HOB >/= _?_at all times to prevent aspiration/reflux _?_feeding tube - placed during surgery Clamped immediately post op - flush with 10-20 ml sterile H2O/NS q shift or 30 ml q6hrs depending on MD Bowel sound need not be present as tube feeds post pyloric.

5-7 days 45 degrees Jejunostomy

Blood loss of _?_ is expected with a fractured femur.

500 to 3000 mL

Surviving Sepsis Campaign: Prior to 2003, 11 international organizations came together to increase awareness and improve outcomes for patients in severe sepsis. Produced (2003, original) recommendations based on evidence for resuscitating the septic patient during the first _?_hours after recognition of the diagnosis

6

How many types of hepatitis are there? What are they?

6 viral; A, B, C, D, E and G also there are bacterial causes of hepatitis

What is normal PAWP? What other measurement corresponds to PAWP?

6-12 LAP

What is normal LAP? What other measurement corresponds to LAP?

6-12; PAWP

Etiology/Patho of Bowel Obstruction: _?_ Liters of fluid enters the small bowel every day Most absorbed before it reaches the _?_ 75% of intestinal gas is swallowed air With obstruction: _?_ and __?_ accumulate above the obstruction causing distention or collapse of the distal bowel.

6-8 colon gas & fluid

What is normal CPP (cerebral perfusion pressure)? When should we begin to worry? What measurement is incompatible with life?

60-100 < 50 <30

What is normal CPP (Cerebral Perfusion Pressure)

60-100 mm Hg (worry if < 50)

Surviving Sepsis Campaign: Initial target MAP of _?_ mmHg in patients with septic shock requiring vasopressors. _?_as the first choice vasopressor

65 Norepinephrine

What is normal SvO2

65-75

What is normal MAP

70-105

Phases of Burn Management: Emergent phase, or resuscitative phase time required to resolve the immediate, life-threatening problems that result from the burn --- Usually lasts up to _?_ from the time the burn occurred. Ends when _?_ and _?_ begin. Primary concerns- onset of _?_ and edema formation.

72 hours fluid mobilization and diuresis hypovolemic shock

Surviving Sepsis Campaign: Goals of care/prognosis discussed with patients and families as soon as possible but at least with in _?_ of ICU admission Goals of care be incorporated into treatment and end-of-life care planning and use palliative care if appropriate.

72 hrs

If a patient has suffered an electrical burn, __?__ ml/hr is the goal for urine output if the patient has hemoglobinuria or myoglobinuria

75-100

Planning Goals for patients with head injury: Maintain a patent airway with RR of _?_ resp regular, deep and unlabored, and O2 sat _?_ Maintain adequate cerebral perfusion and oxygenation Remain normothermic Be free from pain, discomfort, or infection Attain maximal cognitive, motor, and sensory function

8-20/min,; >/= to 92%

What is a normal Ca level? Is an increase in Ca ever "normal"?

8.4-10.2 no

What is normal SVR

800-1200

The nurse is assisting in the care of several patients in the critical care unit. Which patient is at greatest risk for developing multiple organ dysfunction syndrome (MODS)? 22-year-old patient with systemic lupus erythematosus who is admitted with a pelvic fracture after a motor vehicle accident 48-year-old patient with lung cancer who is admitted for syndrome of inappropriate antidiuretic hormone and hyponatremia 65-year-old patient with coronary artery disease, dyslipidemia, and primary hypertension who is admitted for unstable angina 82-year-old patient with type 2 diabetes mellitus and chronic kidney disease who is admitted for peritonitis related to a peritoneal dialysis catheter infection

82-year-old patient with type 2 diabetes mellitus and chronic kidney disease who is admitted for peritonitis related to a peritoneal dialysis catheter infection

What is normal PAP? What two measurements does it consist of?

9-19; PAS/PAD

Manifestations of Addison's Disease are not apparent until _?_% of the adrenal cortex is destroyed

90

After an aneurysm repair, it is necessary to keep your patient at less than _?_ degrees while sitting in a chair and less than _?_ degrees while in bed

90 in chair to limit flexion at graft site 45 in bed to limit risk of occlusion

Post-Op head/neck cancer surgery: Airway maintenance: Monitor O2 sat continuously administer Oxygen therapy (humidified per trach collar) to keep sat > _?_ Breath sounds every 4 hours and prn Respiratory rate q1-4hrs and prn Trach care q shift and prn Suctioning prn HOB elevated _?_ degrees Monitor edema and crepitus q4hrs and prn Humidifier (hospital and at home) continuous _?_ position (decrease edema and limit tension on the suture lines)

92% 30-45 Semi-Fowler's

What is normal O2 sat

95-100

What is normal arterial Hgb saturation

95-100

What is normal PVR

< 250; (160-380)

Oliguric Phase of AKI: (Urine output of_?_). Begins within hours to days of initiating event and typically usually lasts_?_

< 400 ml/24 hours; 10-14 days

CKD is defined by the National Kidney Foundation as the presence of kidney damage or glomerular filtration rate (GFR) _?_ ml/min/1.73m² for _?_ or longer. It is irreversible.

< 60 3 months

Systemic changes occur when overall renal function is _?_ % of normal, or GFR ≤_?_ ml/minute. Manifestations of uremia vary among patients - many are very tolerant

<20-25% <15

Normally, ADH is secreted in response to serum osmolality, which is monitored by osmoreceptors in hypothalamus.If blood is Hypo-osmolar (_?_): then, _?_

<278mOsm/kg ADH ↓and renal water excretion↑

Normally, ADH is secreted in response to serum osmolality, which is monitored by osmoreceptors in hypothalamus.If blood is hyperosmolar(_?_): then, _?_

> 300mOsm/kg ADH increases and renal water is reabsorbed

In Hepatic encephalopathy, INR _?_ Acute Vs. Chronic: chronic is failure lasting longer than_?_ weeks Liver transplant?

>/= 1.5 26

Oliguria of prerenal failure: sp gr of urine _?_ and low _?_ concentration in urine

>1.025 Na

Chest Tube: Monitor and record drainage, usually q1-4hrs, call if_?_ or _?_, or increases suddenly

>100ml or if drainage becomes bright red

What is normal PaO2 to FiO2? (P/F)

>400

To declare brain death: Known etiology of condition No drug intoxication or poisoning Core temperature _?_ _?_ unresponsiveness Absence of _?_ reflexes Pupils fixed and non-reactive No ocular movement (no dolls eyes, no response to cold caloric exam) No _?_ or _?_ reflex

>or= 32 C (90) Cerebral brain stem cough or gag

Drug Therapy for Hepatitis No drug therapies for acute hepatitis _?_infections

A

A patient with a severe pounding headache has been diagnosed with hypertension. However, the hypertension is not responding to traditional treatment. What should the nurse expect as the next step in management of this patient? Administration of β-blocker medications Abdominal palpation to search for a tumor Administration of potassium-sparing diuretics A 24-hour urine collection for fractionated metanephrines

A 24-hour urine collection for fractionated metanephrine

Which patient should be taught preventive measures for CKD by the nurse because this patient is most likely to develop CKD? A 50-year-old white female with hypertension A 61-year-old Native American male with diabetes A 40-year-old Hispanic female with cardiovascular disease A 28-year-old African American female with a urinary tract infection

A 61-year-old Native American male with diabetes

The nurse is caring for a group of patients. Which patient is at highest risk for pancreatic cancer? A 38-year-old Hispanic female who is obese and has hyperinsulinemia A 23-year-old who has cystic fibrosis-related pancreatic enzyme insufficiency A 72-year-old African American male who has smoked cigarettes for 50 years A 19-year-old who has a 5-year history of uncontrolled type 1 diabetes mellitus

A 72-year-old African American male who has smoked cigarettes for 50 years

The nurse is caring for a woman recently diagnosed with viral hepatitis A. Which individual should the nurse refer for an immunoglobin (IG) injection? A caregiver who lives in the same household with the patient A friend who delivers meals to the patient and family each week A relative with a history of hepatitis A who visits the patient daily A child living in the home who received the hepatitis A vaccine 3 months ago

A caregiver who lives in the same household with the patient

The American Spinal Injury Association (ASIA) impairment scale - A is _?_, B is _?_, C is _?_, D is _?_ and E is normal

A is complete - there is no motor or sensory function in S4-S5 B is incomplete - sensory, but not motor function in S4-S5 C is incomplete - motor function is preserved below the neurologic level; more than half of key muscles have muscle grade less than 3 D is incomplete - motor function is preserved be the neurologic level; at least half of key muscles have muscle grade of 3 or more

A patient with intracranial pressure monitoring has a pressure of 8 mmHg. The nurse understands this pressure indicates: A increase in cerebral perfusion pressure A normal balance between brain tissue, blood, & CSF The loss of autoregulatory control in ICP An alteration in the absorption of CSF

A normal balance between brain tissue, blood, & CSF

A 58-year-old woman is being discharged home today after ostomy surgery for colon cancer. The nurse should assign the patient to which staff member? A nursing assistant on the unit who also has hospice experience A licensed practical nurse who has worked on the unit for 10 years A registered nurse with 6 months of experience on the surgical unit A registered nurse who has floated to the surgical unit from pediatrics

A registered nurse with 6 months of experience on the surgical unit

The American Spinal Injury Association (ASIA) impairment scale ranges from _?_ to _?_.

A to E

You're assessing a patient who is post-opt from a chest tube insertion. On assessment, you note there is 50 cc of serosanguinous fluid in the drainage chamber, fluctuation of water in the water seal chamber when the patient breathes in and out, and bubbling in the suction control chamber. Which of the following is the most appropriate nursing intervention? A. Document your findings as normal. B. Assess for an air leak due to bubbling noted in the suction chamber. C. Notify the physician about the drainage. D. Milk the tubing to ensure patency of the tubes.

A. Document your findings as normal.

A patient is recovering from a pneumothorax and has a chest tube present. Which of the following is an appropriate finding when assessing the chest tube drainage system? A. Intermittent bubbling may be noted in the water seal chamber. B. 200 cc of drainage per hour is expected during recovery of a pneumothorax. C. The chest tube is positioned at the patient's chest level to facilitate drainage. D. All of these options are appropriate findings.

A. Intermittent bubbling may be noted in the water seal chamber.

Norepinephrine has been prescribed for a patient who was admitted with dehydration and hypotension. Which patient data indicate that the nurse should consult with the health care provider before starting the infusion? A. The patients CVP is 3 mm Hg. B. The patient is in sinus tachycardia at 120 beats/min. C. The patient is receiving low dose dopamine. D. The patient has had no urine output since

A. The patients CVP is 3 mm Hg.

The most normal functioning method of speech restoration in the patient with a total laryngectomy is A. a voice prosthesis B. esophageal speech C. an electrolarynx held to the neck D. an electrolarynx placed in the mouth

A. a voice prosthesis

A patient is admitted to the emergency department (ED) for shock of unknown etiology. The first action by the nurse should be to A. administer oxygen. B. obtain a 12-lead electrocardiogram. C. obtain the blood pressure. D. check the level of consciousness

A. administer oxygen.

The current standard of care for PCI typically includes dual antiplatelet therapy with oral A. aspirin and Clopidogrel (Plavix). B. aspirin and Abciximab (ReoPro) C. aspirin and Eptifibatide (Integrilin) D. aspirin and Tirofiban (Aggrastat)

A. aspirin and Clopidogrel (Plavix).

In caring for a postoperative cardiovascular patient, the nurse knows that the most frequent cause of a decreased cardiac output is A. reduced preload. B. increased afterload. C. increased contractility. D. bradycardia.

A. reduced preload.

Lower GI Bleed Treatment: Emergency care: _?_ Respiratory status: airway, breathing V/S, peripheral perfusion Abdominal exam History BMP (monitor BUN), CBC, PT/INR, PTT, liver enzymes, ABG, Type and cross match, UA including specific gravity

ABCs

Name several diagnostic studies for acute respiratory failure

ABG's X-ray Pulse oximetry (reveals nothing about ventilation; is more about perfusion) ECG Cultures (sputum, blood) CBC & BMP BNP (b-Type natriuretic peptide) V/Q scan Spiral CT Urinalysis Pulmonary artery catheter (severe cases)

What is the medication treatment for functional (secondary) MR with heart failure? (5)

ACE inhibitors or ARBs, beta blockers, aldosterone antagonist, digoxin and a diuretic.

Diagnostic Studies-- Adrenal Disease (Cushing): _?_ levels (may be normal, or ↑ depending on cause; Alkalosis, _?_ serum K, glucosuria, hypercalciuria,; _?_glucose, _?_ WBC (eos and lymphs), water retention; MRI and CT

ACTH levels (may be normal, or ↑ depending on cause; Alkalosis, ↓ serum K, glucosuria, hypercalciuria,; ↑glucose, ↑WBC (eos and lymphs), water retention; MRI and CT

C6: Tetraplegia - What can this person expect to do? Shoulder & upper back abduction and rotation of shoulder, full biceps to elbow flexion, wrist extension, weak grasp of thumb, decreased resp reserve.

Able to assist with transfer and perform some self care. Feed self w/ hand devices. Push w/c on smooth/flat surface. Drive adapted van from w/c. independent computer use with adaptive equipment. Attendant care 6hr/day

C5: Tetraplegia- What can this person expect to do? Full neck, partial shoulder, back and biceps. Gross elbow, but unable to roll over/use hands. Decreased resp reserve

Able to drive electric w/c with mobile hand support. Indoor mobility w/ manual w/c. Able to feed self w/ setup and adaptive equipment. Attendant care 10hr/day.

C1-3: Tetraplegia - What can this person expect to do? Often fatal. Movement neck & above. Loss of innervation to diaphragm. Absent independent resp function.

Able to drive electric wheelchair that has port ventilator by using chin control or mouth stick. Headrest to stabilize head. Can use computer with mouth stick, head wand, or noise control. Attendant care 24 hrs/day, able to instruct others

C4: Tetraplegia- What can this person expect to do? Sensation and movement in neck and above. Might be able to breath without vent.

Able to drive electric wheelchair that has port ventilator by using chin control or mouth stick. Headrest to stabilize head. Can use computer with mouth stick, head wand, or noise control. Attendant care 24 hrs/day, able to instruct others

C7-8: Tetraplegia - What can this person expect to do? All triceps to elbow extension, finger extensors and flexors, good grasp with some decreased strength, decreased resp reserve

Able to transfer self to w/c. Roll over and sit up in bed. Push self on most surfaces. Perform most self care. Independent use of w/c. Able to drive car w/ powered hand controls (some patients). Attendant care 0-6hrs/day

Hypovolemic Shock _?_ - blood loss, plasma loss, other fluid loss _?_ - fluid volume moves out of the vascular space into extravascular space

Absolute Relative

A patient with cholelithiasis needs to have the gallbladder removed. Which patient assessment is a contraindication for a cholecystectomy? Low-grade fever of 100° F and dehydration Abscess in the right upper quadrant of the abdomen Activated partial thromboplastin time (aPTT) of 54 seconds Multiple obstructions in the cystic and common bile duct

Activated partial thromboplastin time (aPTT) of 54 seconds

Risk factors for _?_ include: Shock Septicemia Hemolytic process (transfusion of mismatched blood) OB conditions (abruptio placentae, amniotic fluid embolism, septic abortion) Malignancies (acute leukemia, lymphoma, tumor lysis syndrome) Tissue damage (burns, trauma, heatstroke, severe head injury, organ transplant rejection, post-operative damage, pulmonary embolus, glomerulonephritis, fulminant hepatitis)

Acute DIC (Disseminated Intravascular Coagulation)

_?_ is a type of lymphocytic leukemia Most common in children Prognosis poorer with increased WBC

Acute lymphocytic leukemia (ALL)

_?_ is a type of myelogenous leukemia. Most common in adults; Prognosis poorer with increased age, WBC, slow response to treatment

Acute myelogenous leukemia (AML)

3 stages make up acute GBS _?_: 1-4 weeks Weakness of lower ext usually peaks about the 14th day. Begins w/ onset of first symptom Ends when no further deterioration occurs

Acute or initial period

_?_is an acute inflammatory process; autodigestion by enzymes

Acute pancreatitis

What are 12 complications that may develop secondary to ARDS?

Acute renal failure Anemia Atelectasis DIC Dysrhythmias MI MODS oxygen toxicity Paralytic ileus PE Stress ulcers/hemorrhage thrombocytopenia

What does ARDS stand for?

Acute respiratory distress syndrome

_?_ is a type of intrarenal injury caused by ischemia, nephrotoxins, or pigments (such as myoglobin); It is the most common cause of _?_ AKI.

Acute tubular necrosis (ATN); intrarenal

Adrenal Cortex Hypofunction - Primary is called _?_. Which corticosteroids are affected?

Addison's disease all 3 (gluco ,mineral, androgen) are all reduced

_?_ is a Life threatening response to acute adrenal insufficiency

Addisonian Crisis

_?_ is a cancer that forms in epithelial cells that produce fluids or mucus. Tissues with this type of epithelial cell are sometimes called glandular tissues. Includes most cancers of the breast, colon, and prostate

Adenocarcinoma

How is an apnea test conducted? Serial ABGs after five minutes until PCO2 >or= 60 mmHg Reconnection of ventilator when test is complete if respiratory movements are absent and arterial PCO2 is _?_ mm Hg the apnea test result is positive (brain dead) If hypotension, arrhythmia, or severe desaturation occurs, reconnect ventilator immediately & consider other test

Adjust vasopressors to a systolic blood pressure ≥100 mm Hg. • Preoxygenate for at least 10 minutes with 100% oxygen to a PaO2 >200 mm Hg. • If pulse oximetry oxygen saturation remains >95%, obtain a baseline blood gas • Disconnect the patient from the ventilator. • Preserve oxygenation • Look for respiratory movements for 8-10 minutes. • Abort if systolic blood pressure decreases to <90 • Abort if oxygen saturation measured by pulse oximetry is <85% for >30 seconds. • If no respiratory drive is observed, repeat blood gas after approximately 8 minutes. • If respiratory movements are absent and arterial PCO2 is ≥60 mm Hg the apnea test result is positive (brain dead).

Risk Factors for Aortic Aneurysm

Age, Male gender, Hypertension, Coronary artery disease (CAD), Family history, High cholesterol , Lower extremity PAD, Carotid artery disease, Previous stroke, Tobacco use , Excess weight or obesity

A patient's localized infection has progressed to the point where septic shock is now suspected. What medication is an appropriate treatment modality for this patient? Insulin infusion IV administration of epinephrine Aggressive IV crystalloid fluid resuscitation Administration of nitrates and β-adrenergic blockers

Aggressive IV crystalloid fluid resuscitation

_?_ provides two levels of continuous positive airway pressure with timed releases and permits spontaneous breathing throughout the respiratory cycle. The clinician selects both pressure high and pressure low along with time high and time low.

Airway Pressure Release Ventilation (APRV)

Phases of Burn Management: Prehospital care Large thermal burns: _?_ Do not immerse in cool water or pack with ice. Remove _?_. Wrap in clean, dry sheet or blanket. Inhalation injury: Observe for signs of respiratory distress or compromise. Treat quickly.

Airway, breathing, and circulation burned clothing

Adrenal Cortex: Synthesizes 3 classes of steroid hormones. 1 class is mineralocorticoids, primary _?

Aldosterone

_?_ is a type of stem cell transplant in which the patient gets stem cells obtained from donor; must have human leukocyte antigen tissue typing and match recipient

Allogeneic

A frail 72-year-old woman with stage 3 chronic kidney disease is cared for at home by her family. The patient has a history of taking many over-the-counter medications. Which over-the-counter medications should the nurse teach the patient to avoid? Aspirin Acetaminophen (Tylenol) Diphenhydramine (Benadryl) Aluminum hydroxide (Amphogel)

Aluminum hydroxide (Amphogel)

Diagnostics for Chronic Pancreatitis Assess signs and symptoms Lab tests _?_ and _?_may be slightly elevated or normal - depends on how much fibrosis May have increased serum _?_and alkaline phosphatase

Amylase and lipase bilirubin

Treatment for _?_ shock: Epinephrine (subQ, IV, nebulizer) diphenhydramine (Benadryl) (IV, po) Maintain patent airway and oxygen Possible need intubation or cricothyroidotomy Identify and remove cause Aggressive fluid resuscitation with colloids Antihistamines (e.g. Diphenhydramine (benadryl) Bronchodilators (e.g., nebulized albuterol) Corticosteroids (e.g., solumedrol IV) if hypotension persists after 1-2 hrs of aggressive therapy Prevention: avoid risk factors

Anaphylactic

A patient with cirrhosis of the liver is admitted to the hospital. What hematologic symptoms might be noted in this patient? Select all that apply. Anemia Leukemia Leukopenia Polycythemia vera Thrombocytopenia

Anemia Leukopenia Thrombocytopenia

3 cell-related adverse effects of chemo:

Anemia, Thrombocytopenia, Neutropenia

Clinical manifestations of aortic stenosis (3)

Angina Syncope Exertional dyspnea

6 types of targeted therapy are:

Angiogenesis inhibitor BCR-ABL trkyosine kinase inhibitors CD20 Monoclonal antibodies epidermal growth factor receptor (EGFR) proteasome inhibitor tyrosine kinase inhibitors

5 gastrointestinal clinical manifestations of CKD

Anorexia nausea vomiting gastointestinal bleeding gastritis

Drug therapy for Duodenal ulcer, gastric ulcer, acute gastritis: _?_ and _?_ H2-receptor blockers, Proton Pump Inhibitors (PPI)

Antacids, Sucralfate (Carafate)

2. _?_- Results in compromised blood flow to anterior spinal cord; Usually from injury causing acute compression of anterior portion of spinal cord; Often a flexion injury S/S: motor paralysis, loss of pain and temp sensation. Preservation of position, vibration, motion, and touch sense (posterior tracts are intact).

Anterior cord syndrome

_?_ bind directly to DNA, inhibits synthesis of DNA, interferes w/ transcription of RNA; can permanently damage the heart if given in high doses. Examples are: bleomycin (Blenoxan) and doxorubicin (Adriamycin) which in high doses these drugs cause heart damage

Anti-tumor antibiotics

Collaborative care: Identify and treat underlying problem/symptoms with acute pericarditis: _?_ for bacterial pericarditis _?_ for pain and inflammation _?_ if pericarditis d/t lupus, pts taking steroids for rheumatic issues or other autoimmune issues, or if don't respond to _?_ Might need hemodynamic support if cardiac tamponade issues prior to pericardiocentesis Pericardiocentesis Pericardial window

Antibiotics NSAIDS steroids NSAIDS

_?_ - given during and after neurosurgery Given prophylactically to prevent seizures Antibiotics - prevention of infection Antipyretics - Treat fevers (> 38 degrees C.) to decrease metabolic demands and potential for increased ICP

Anticonvulsants- Phenytoin

_?_Largest artery; supplies oxygen, nutrients, and blood to all vital organs

Aorta:

_?_: abnormal dilation of arterial wall, caused by localized weakness & stretching in medial layer or wall of aorta

Aortic Aneurysm

_?_- May result from disease of aortic valve leaflets, aortic root, or both

Aortic Valve Regurg

_?_ - Usually discovered in childhood, adolescence, or young adulthood

Aortic Valve Stenosis

A 78-year-old patient has Stage 3 CKD and is being taught about a low potassium diet. The nurse knows the patient understands the diet when the patient selects which foods to eat? Apple, green beans, and a roast beef sandwich Granola made with dried fruits, nuts, and seeds Watermelon and ice cream with chocolate sauce Bran cereal with ½ banana and milk and orange juice

Apple, green beans, and a roast beef sandwich

19-year-old woman is hospitalized for a frontal skull fracture from a blunt force head injury. Clear fluid is draining from the patient's nose. What action by the nurse is most appropriate? Apply a loose gauze pad under the patient's nose. Place the patient in a modified Trendelenburg position. Ask the patient to gently blow the nose to clear the drainage. Gently insert a catheter in the nares and suction the drainage.

Apply a loose gauze pad under the patient's nose.

Normally, the the visceral and parietal pleura are separated only by the lubricating pleural fluid. Fluid reduces friction, allowing the pleura to slide easily during breathing. *Normal pleural fluid quantity: _?_

Approx. 5-15 ml per lung

Hormone therapy drugs are chemo drugs which selectively attach to estrogen receptors, inhibits tumor growth. A specific type is _?_ which inhibit aromatase (enzyme that converts adrenal androgen to estrogen). An example is: letrozole (Femara)

Aromatase Inhibitors

Restoration of Speech- _?_ -placed next to neck and creates speech with the use of sound waves. _?_ -plastic tube in the corner of the roof of mouth to create vibrations. Cooper-Rand device. _?_-swallow air, hold the air in the esophagus, and release to create sound. Pronounces words with use of muscles in the throat and mouth. -?_- surgical puncture between the trachea and esophagus. Blom-Singer device

Artificial larynx Electrolarynx Esophageal speech Voice prosthesis (Tracheoesophageal Puncture)

Collaborative care for _?_ I/O, Daily Weights, Abdomen girths HOB elevated Paracentesis

Ascites

Complications of Cirrhosis: _?_ - I/O, Daily Weights, Abdomen girths; HOB elevated; Paracentesis _?_ - observe for bleeding

Ascites - I/O, Daily Weights, Abdomen girths HOB elevated Paracentesis Esophageal Varices - observe for bleeding

Complications of Acute Liver Failure: _?_ risk of peritonitis (SBP =spontaneous bacterial Peritonitis) _?_ - kidney disease due to interrupted blood flow through the liver restricting blood flow to the kidneys _?_ - brain function affected due to toxins build up Liver cancer, especially if hepatitis B and/or C

Ascites- Hepatorenal syndrome Hepatic encephalopathy

A 52-year-old man with stage 2 chronic kidney disease is scheduled for an outpatient diagnostic procedure using contrast media. Which action should the nurse take? Assess skin turgor to determine hydration status. Insert a urinary catheter for the expected diuresis. Evaluate the patient's lower extremities for edema. Check the patient's urine for the presence of ketones

Assess skin turgor to determine hydration status.

The physician orders intracranial pressure (ICP) readings every hour for a 23-year-old male patient with a traumatic brain injury from a motor vehicle crash. The patient's ICP reading is 21 mm Hg. It is most important for the nurse to take which action? Document the ICP reading in the chart. Determine if the patient has a headache. Assess the patient's level of consciousness. Position the patient with head elevated 60 degrees.

Assess the patient's level of consciousness

_?_ is when the patient initiates a spontaneous breath, the vent delivers a preset Vt

Assist control (AC)

_?_ requires that the rate, Vt, inspiratory time and PEEP be set for the patient. The ventilator sensitivity is also set and when the patient initiates a spontaneous breath, a full-volume breath is delivered.

Assist control (AC) or Assisted Mandatory Ventilation (AMV)

ACV

Assist-Control Ventilation There is a set rate and tidal volume. BUT, the patient is able to initiate an extra breath. The "extra breath" is delivered by the machine, NOT performed by the patient. The delivered breath is a full-tidal volume breath (may be more than the patient needed).

A patient's spontaneous inspiratory efforts (breaths) are assisted when the _?_ is used. A patient's spontaneous inspiratory efforts (breaths) are assisted as needed so he takes a full breath. Also, if a breath does not happen spontaneously, the machine will initiate and assist the breath. There is a preset tidal volume or pressure

Assist-control ventilation

What are 3 diseases which cause abnormalities of the airways and alveoli that lead to a decrease in ventilatory supply?

Asthma Emphysema Cystic fibrosis

Clinical manifestation of Chronic mitral valve regurg:

Asymptomatic for years until development of some degree of left ventricular failure

3 Manifestations of Acromegaly: Cardio

Atherosclerosis, HTN, CHF,

Postoperative care of a patient undergoing coronary artery bypass graft (CABG) surgery includes monitoring for what common complication? Dehydration Paralytic ileus Atrial dysrhythmias Acute respiratory distress syndrome

Atrial dysrhythmias

Surgical treatment for IPAH: _?_ palliative procedure that involves creation of an intra-atrial right to left shunt to decompress the right ventricle (as wait for lung transplant) If severe right-sided HF, and not good response to drugs - _?_ needed. Recurrence does not occur after this surgery

Atrial septostomy (AS); lung transplant

_?_ is a type of stem cell transplant in which the patient gets their own stem cells back after myeloablative chemo

Autologous

_?__: automatic device called a cycler is used to deliver dialysate- allows PD while asleep; cycles 4 or more exchanges per night, although must have a couple exchanges during the daytime

Automated Peritoneal Dialysis (APD)

_?_ is an uncompensated cardiovascular reaction caused by the sympathetic nervous system. Return of reflexes after resolution of spinal shock means patient with injury level at _?_or ↑ may develop Occurs in response to visceral stimulation Life-threatening - stroke, MI, death, status epilepticus Most common precipitating factor is _?_ or _?_

Autonomic Hyperreflexia. T6 distended bladder or rectum

A patient is admitted to the ICU after she develops disseminated intravascular coagulation (DIC). DIC is known to occur in patients with retained placental fragments. What is the result of DIC? A. Hypersensitive response to an antigen, resulting in anaphylaxis B. Depletion of clotting factors and excessive fibrinolysis, resulting in simultaneous microvascular clotting and hemorrhage C. Vasodilation, resulting in hypotension D. Septic shock, resulting in vasodilation and decreased perfusion.

B. Depletion of clotting factors and excessive fibrinolysis, resulting in simultaneous microvascular clotting and hemorrhage

Which of the following mechanisms is responsible for the augmentation of coronary arterial blood flow and increased myocardial oxygen supply seen with the intra-aortic balloon pump? A. The vacuum created in the aorta as a result of balloon deflation B. Diastolic inflation with retrograde perfusion C. Forward flow to the peripheral circulation D. Inflation during systole to augment blood pressure

B. Diastolic inflation with retrograde perfusion

What type of chest tube system does this statement describe? This chest drainage system has no water column to control suction but uses a suction monitor bellow that balances the wall suction and you can adjust water suction pressure using the rotary suction dial on the side of the system. It allows for higher suction pressure levels, has no bubbling sounds, and water does not evaporate from it as with other systems. A. Mediastinal chest tube system B. Dry suction chest tube system C. Wet suction chest tube system D. Dry-Wet suction chest tube system

B. Dry suction chest tube system

A patient is about to have their chest tube removed by the physician. As the nurse assisting with the removal, which of the following actions will you perform? Select-all-that-apply: A. Educate the patient how to take a deep breath out and inhale rapidly while the tube in being removed. B. Gather supplies needed which will include petroleum gauze dressing per physician preference. C. Place the patient in Semi-Fowler's position. D. Have the patient take a deep breath, exhale, and bear down during removal of the tube. E. Pre-medicate prior to removal as ordered by the physician. F. Place the patient is prone position after removal.

B. Gather supplies needed which will include petroleum gauze dressing per physician preference. C. Place the patient in Semi-Fowler's position. D. Have the patient take a deep breath, exhale, and bear down during removal of the tube. E. Pre-medicate prior to removal as ordered by the physician.

While helping a patient with a chest tube reposition in the bed, the chest tube becomes dislodged. What is your immediate nursing intervention? A. Stay with the patient and monitor their vital signs while another nurse notifies the physician. B. Place a sterile dressing over the site and tape it on three sides and notify the physician. C. Attempt to re-insert the tube. D. Keep the site open to air and notify the physician.

B. Place a sterile dressing over the site and tape it on three sides and notify the physician.

The nurse is caring for a newly admitted trauma patient. Which assessment finding is most important for the nurse to report to the health care provider? A. BP of 92/56 mm Hg B. Skin cool and clammy C. Oxygen saturation 92% D. Heart Rate 118 beats/min

B. Skin cool and clammy

A patient with a chest tube has no fluctuation of water in the water seal chamber. What could be the cause of this? A. This is an expected finding. B. The lung may have re-expanded or there is a kink in the system. C. The system is broken and needs to be replaced. D. There is an air leak in the tubing.

B. The lung may have re-expanded or there is a kink in the system.

Which finding is the best indicator that the fluid resuscitation for a patient with hypovolemic shock has been effective? A. Hgb is within normal limits B. Urine output is 60 mL over the last hour C. CVP is normal D. Mean Arterial Pressure is 72 mm Hg

B. Urine output is 60 mL over the last hour

When an intra-aortic balloon is in place, it is essential for the nurse to frequently assess A. for a pulse deficit. B. peripheral pulses distal to the catheter insertion site. C. bilateral blood pressures. D. coronary artery perfusion

B. peripheral pulses distal to the catheter insertion site.

_?_ decrease myocardial oxygen demand by reducing HR, BP and contractility.

BB

A medication that may be helpful with mitral valve prolapse is _?_. Also, patients should stay _?_ and avoid _?_

BB well-hydrated avoid caffeine

What is the medical management of aortic stenosis: (4 medications)

BB, digoxin, diuretics, ACE

Once a hypertensive, hypoxic patient is intubated, what may happen to their BP? How would you treat it? (2)

BP may drop suddenly; fluids Positive inotrope

__?__ is Acute inflammation of meningeal tissues Infection of arachnoid matter and CSF Organisms Streptococcus pneumoniae & Nesisseria meningitidis Signs and symptoms Fever, severe headache, N/V, nuchal rigidity, photophobia, decreased LOC, seizures, increase ICP

Bacterial meningitis

13 Conditions predisposing a patient to ARDS:

Bacterial/viral PNA Sepsis Chest Trauma/Trauma in general Embolus (air, fat, etc) Aspiration of gastric contents Toxic substances O2 toxicity Near-drowning Radiation pneumonitis Acute pancreatitis Bi-pass machines Shock states transfusion related acute lung injury

_?_is used for mitral, tricuspid, and pulmonic stenosis, and less often for aortic stenosis. The _?_ procedure is generally indicated for older adults and for those who are poor surgery candidates. It has fewer complications than valve replacement. The long-term results are similar to those of surgical commissurotomy.

Balloon valvuloplasty Percutaneous transluminal balloon valvuloplasty (PTBV)

Sarah, a hospice nurse visits a client dying of ovarian cancer. During the visit, the client expresses that "If I can just live long enough to attend my daughter's graduation, I'll be ready to die." Which phrase of coping is this client experiencing? Anger Denial Bargaining Depression

Bargaining

_?_ is a cancer that begins in the lower or basal (base) layer of the epidermis (outer layer of skin).

Basal cell carcinoma

The location of the skull fracture determines the clinical manifestations the patient presents with. Conjugate deviation of gaze: _?_ skull fractures. Eyes are not working together. Periorbital edema & ecchymosis (raccoon eyes): _?_ fractures Rhinorrhea:_?_ or _?_ skull fractures

Basilar; Orbital; Frontal & Basilar

The location of the skull fracture determines the clinical manifestations the patient presents with. Different types of skulls fractures include: frontal, orbital, temporal, parietal, posterior fossa, and basilar skull fractures Symptoms of Skull Fractures: Facial paralysis: _?_skull fractures Battle's sign: _?_ or _?_ skull fractures. Battle's sign is an oval shaped bruise behind the ear in the mastoid region aka _?_

Basilar; Parietal or basilar; . - retroauricular ecchymosis

Spinal Nerve Roots Involved in testing are the following: Biceps tests for spinal nerve roots _?_ and _?_ Brachioradialis tests for spinal nerve roots _?_ Triceps tests for spinal nerve roots _?_ Patellar tests for spinal nerve roots _?_ Achilles Tendon tests for spinal nerve roots _?_

Biceps tests for spinal nerve roots C5 and C6 Brachioradialis tests for spinal nerve roots C6 Triceps tests for spinal nerve roots C7 Patellar tests for spinal nerve roots L4 Achilles Tendon tests for spinal nerve roots S1

Primary etiologies of acute pancreatitis cause autodigestion. Name 9

Biliary tract disease Alcoholism trauma (including surgery) viral infections (mumps, HIV) penetrating duodenal ulcer Abscesses cystic fibrosis metabolic disorders (hperparathyroidism, renal failure) Meds (corticosteroids, thiazide diuretics, oral contraceptives, sulfonamides, NSAIDS)

The following are ways in which _?_ are useful in cancer treatment: 1. direct anti-tumor effects 2. restore, augment, or modulate host immune system mechanisms 3. interfere w/ cancer cells' ability to metastasize or differentiate

Biologics

_?_ are cancers therapies which affect host-tumor response in 3 ways: direct anti-tumor effects; restore, augment, or modulate host immune system mechanisms; interfere w/ cancer cells' ability to metastasize or differentiate

Biologics

_?_ is a type of cancer therapy which modifies the relationship between the host and tumor cells

Biologics

_?_ is a 3 lumen tube used for the tamponade of gastro-oesophageal bleeding that is unresponsive to medical and endoscopic treatment

Blakemore Tube

The nurse should monitor for increases in which laboratory value for the patient as a result of being treated with dexamethasone (Decadron)? Sodium Calcium Potassium Blood glucose

Blood glucose

Which of the following assessments are important with hypovolemic shock? Select all that apply (there are 6 correct answers). Blood pressure Bowel sounds Heart rate Respiratory rate Skin color Urine output Mental status

Blood pressure Heart rate Respiratory rate Skin color Urine output Mental status

A patient with a history of end-stage kidney disease secondary to diabetes mellitus has presented to the outpatient dialysis unit for his scheduled hemodialysis. Which assessments should the nurse prioritize before, during, and after his treatment? Level of consciousness Blood pressure and fluid balance Temperature, heart rate, and blood pressure Assessment for signs and symptoms of infection

Blood pressure and fluid balance

_?_ is when body struck by blunt object

Blunt trauma:

The following is the process for _?_ done in OR, anesthesia given, cells removed from Iliac crest or sternum; takes about 1-2hrs, pt can go home after recovery takes 2 weeks to see if it was effective

Bone Marrow Transplant (a type of HSCT (hematopoietic stem cell transplant))

Interventions for _?_ Placement of NG or intestinal tubes Surgery: strangulated obstruction is a surgical emergency! May use laporoscopy for repair. Nonsurgical: colonoscopy to remove polyps, dilate strictures, and remove or destroy tumors with laser.

Bowel obstruction

The nurse is caring for a patient admitted with a subdural hematoma following a motor vehicle accident. Which change in vital signs would the nurse interpret as a manifestation of increased intracranial pressure (ICP)? Tachypnea Bradycardia Hypotension Narrowing pulse pressure

Bradycardia

Thoracic and lumbar vertebrae injuries Systemic support less intense than cervical injury Respiratory compromise not as severe _?_ is not a problem. Specific problems treated symptomatically

Bradycardia

Which clinical manifestation would the nurse interpret as a manifestation of neurogenic shock in a patient with acute spinal cord injury? Bradycardia Hypertension Neurogenic spasticity Bounding pedal pulses

Bradycardia

Nursing Care & Interventions for _?_ Antimicrobial therapy Treatment of symptoms Increased ICP management Headache Fever Surgical Intervention Abscess may need to be drained or removed Goal after cranial surgery is prevention of increased ICP

Brain abscess

_?_ different types of brain and spinal cord tumors, named based on the type of cell in which they formed and where the tumor first formed in the central nervous system. For example, an astrocytic tumor begins in star-shaped brain cells called astrocytes, which help keep nerve cells healthy. Brain tumors can be benign (not cancer) or malignant (cancer).

Brain and Spinal Cord Tumors:

Surviving Sepsis Campaign: _?_ therapy to cover all likely pathogens, given as soon as possible after recognition and within 1 h for both sepsis and septic shock. Appropriate cultures obtained prior to antibiotic use if no substantial delay in starting antibiotics

Broad-spectrum IV antibiotic

_?_ - Result of damage to one half of spinal cord Characterized by a loss of motor function, position,vibration sense, and vasomotor paralysis on same side as lesion Opposite side has loss of pain and temperature sensation below level of lesion Usually from penetrating wound to spinal cord

Brown-Sequard syndrome

Proper oral care of a ventilated patient

Brush their teeth 2x / day. Apply moisturizing agent to lips q4h.

Nursing and collaborative management of burns: Determine if need to be transferred to _?_ quickly. Nursing and collaborative management consists of 3 things. What are they?

Burn Unit; airway management fluid management wound care.

_?_- is an opening into cranium using drill, used to drain/remove localized fluid or blood beneath the dura

Burr-hole

How do cancer cells evade the immune system?

By secreting substances that mute the T-cells and/or by having weak TAA's that evade detection

What does "CAUTION" stand for regarding cancer?

C - change in bowel/bladder habits A - a sore that doesn't heal U - unusual bleeding/discharge T - thickening/lump I - Indigestion/trouble swallowing O - obvious change in mole N - nagging cough

A patient is receiving positive pressure mechanical ventilation and has a chest tube. When assessing the water seal chamber what do you expect to find? A. The water in the chamber will increase during inspiration and decrease during expiration. B. There will be continuous bubbling noted in the chamber. C. The water in the chamber will decrease during inspiration and increase during expiration. D. The water in the chamber will not move.

C. The water in the chamber will decrease during inspiration and increase during expiration.

Tetraplegia (total or partial) involves a spinal cord injury in areas __?_ through _?_?

C1 - C8

Cervical injuries above the level of _?_ present special problems because of the total loss of respiratory muscle function

C4

Respiratory complications - level of injury Cervical injury - Above level of _?_ = Total loss of respiratory muscle function; Mechanical ventilation is required to keep patient alive Cervical injury - Below level of _?_= Diaphragmatic breathing if phrenic nerve is functioning; Spinal cord edema/hemorrhage affects function of _?_, can cause respiratory insufficiency; _?_ usually occurs with diaphragmatic breathing

C4 C4 phrenic nerve Hypoventilation

Examples of oncofetal antigens: 1. _?_ - pancreatic and gallbladder 2. _?_ - prostate 3. _?_ and _?_ breast cancer

CA19-9 PSA CA-15-3 and CA 27-29 -

The following are indicators for _?_" Significant Left Main Coronary Occlusion Triple vessel CAD Unstable angina L. ventricular failure Failed medical management Those who are not candidates for PCI DM Failed PCI

CABG

_?_ consists of placement of conduits to transport blood between the aorta (or other major arteries) and the myocardium

CABG

CABG is a palliative treatment for _?_, not a cure

CAD

_?_ is the #1 cause of death in transplant patients. This is due a side effect of medication: _?_

CAD hyperlipidemia

There are 7 warning signs of cancer (plus 4 others!?!). What is the acronym that helps you remember the first 7?

CAUTION

Lab test for Gallbladder disease : _?_ (will show elevated _?_ d/t inflammation) direct and indirect _?_ (both elevated if obstruction present); alkaline phosphatase, ALT, AST may be elevated; serum amylase and lipase increased if pancreas involved

CBC (WBC elevated d/t inflammation); bilirubin

What three measurements tell us how well the brain is perfused?

CBF (cerebral blood flow) CPP(cerebral perfusion pressure) and ICP (intracranial pressure)

Vasodilators (parenteral) are given IV to patients who do not respond to _?_ or have Class III or IV right-sided heart failure

CCB

Rhabdomyolysis: Collaborative Care Lab work including chemistries and _?_ levels, urine __?_, serum _?_;

CK myoglobin myoglobin

_?_ is a progressive reduction of functioning renal tissue such that the remaining kidney mass can no longer maintain the body's internal environment.

CKD

Corneal reflexes evaluates _?_ and _?_ at same time. Not normally tested in awake and alert pts. Involves applying cotton wisp strand to cornea. The Sensory component of this reflex is stimulated by the ophthalmic division of _?_. Motor component (blinking of eye) is stimulated by the facial nerve _?_.

CN V and CN VII CN V CN VII

In sudden cardiac death (SCD), there is a sudden disruption in cardiac function which produces an abrupt loss of _?_ and _?_

CO cerebral blood flow

How is CI calculated?

CO/BSA(body surface area)

Normal Lab values for the following are: CO2: BUN: Creatinine: Phosphorous:

CO2: (normal 23-30 mEq/L) BUN: (normal 6-20 mg/dl) Creatinine: (normal 0.6-1.3 mg/dl) Phosphorous: (normal 2.4-4.4 mg/dl)

Inflammation and healing- Burn injury to tissues and vessels causes _?_ Immunologic changes -skin barrier is destroyed, bone marrow depression occurs, and circulating levels of immunoglobulins are decreased. WBC's function is decreased. The inflammatory cytokine cascade, triggered by tissue damage, impairs the function of lymphocytes, monocytes, and neutrophils. This causes the patient to be at a greater risk for ________________.

COAGULATION NECROSIS. infection

_?_ restores functional residual capacity (FRC). This pressure is continuous during spontaneous breathing; no positive pressure breaths are present

CPAP

_?_. _?_ and _?_ have greatly improved long-term survival rates after witnessed cardiac arrest

CPR defibrillation with AED early advanced cardiac life support

RRT also includes _?_ and _?_ because it is any way in which the blood is filtered

CRRT and transplant

_?_is the best diagnostic test to evaluate for head trauma due to rapid diagnosis and intervention MRI and PET may also be used. _?_ are more sensitive at detecting small lesions . _?_ studies allow for measurement of CBF velocity

CT ; MRIs; Transcranial Doppler

_?_(gold standard)- assess the stability of the injury, the location and degree of bony injury, and degree of spinal canal compromise Complete spine films (Xray) MRI - used to assess soft tissue Neurological exam - comprehensive along with assessment of head, chest, abdomen for additional injuries or trauma Vertebral angiography to r/o vertebral artery damage in pt with cervical injury Labs: ABGs, CMP, coag profile, CBC, UA

CT Scan

A _?_ may be used to rule out vertebral artery damage

CT angiogram

Lower GI Bleed Treatment: May not require medical treatments except monitoring If continue to bleed: _?_ to localize (see) area bleeding; can use embolization to stop bleeding Need _?_ blood loss Tagged RBC scan (nuclear med) Can see bleeding if > 0.1ml/min blood loss Unreliable localization; High false positive rate

CT angiography > 0.5ml/min

If EVAR: regular follow ups, routine _?_ or _?_

CT or MRI

_?_ is the preferred way in which to diagnose the location and degree of injury and the degree of spinal canal compromise

CT scan

What would the following values look like with cardiogenic shock? CVP PAWP SVR SvO2 CO JVD present/absent

CVP = increased PAWP= increased SVR = increased SvO2 = decreased CO = decreased JVD present/absent = present

What do the following values look like if your patient develops cardiac tamponade post-CABG: CVP and PAWP = Heart Size on X-ray = Heart sounds = BP = Urine Output = Mediastinal Chest Tube drainage = Presence/Absence of JVD =

CVP and PAWP = equalized and elevated Heart Size on X-ray = enlarged Heart sounds = muffled BP = decreased Urine Output = decreased Mediastinal Chest Tube drainage = decreased Presence/Absence of JVD = present

Diagnostic Studies for aneurysm:

CXR: to reveal widening of the thoracic aorta , Abdominal x-ray: may show calcification in aortic wall, ECG: rule out MI (pain may mimic angina) , Echocardiogram: assess function of the aortic valve, Ultrasound: useful for aneurysm screening, monitor aneurysm size , CT: most accurate to determine length, cross-sectional diameter, & if thrombus in aneurysm, MRI: used to diagnose, assess location/severity of aneurysms. Angiography: provides helpful information, uses contrast imaging to map entire aortic system

5 Criteria for hyperparathyroidism surgery:

Ca > 12, < 50 years old, hypercalciuria, ↓ bone density, kidney stones

Name 2 types immunosuppressive agents: _?_ such as: cyclosporine (Sandimmune, Neoral, Gengraf) and tacrolimus (Prograf, FK506). _?_ is the most widely used _?_ such as: (prednisone, methyprednisolone [Solu-Medrol]IV)

Calcineurin Inhibitors tacrolimus Corticosteroids

Normal values of the following labs are: Calcium: Magnesium: Hematocrit: Hemaglobin:

Calcium: (normal 8.6-10.2 mg/dl) Magnesium: (normal 1.5-2.5 mEq/L) Hematocrit: (normal 35-50%) Hemaglobin: (normal 11.7-17.3 g/dl)

The patient has had type 1 diabetes mellitus for 25 years and is now reporting fatigue, edema, and an irregular heartbeat. On assessment, the nurse finds that the patient has newly developed hypertension and difficulty with blood glucose control. The nurse should know that which diagnostic study will be most indicative of chronic kidney disease (CKD) in this patient? Serum creatinine Serum potassium Microalbuminuria Calculated glomerular filtration rate (GFR)

Calculated glomerular filtration rate (GFR)

Hypo- and Hyperventilation of alveoli

Can lead to abnormal PaO2 and PaCO2 levels --> pH alterations --> acidosis / alkalosis --> etc.

The critical care nurse is caring for a 55-year-old man who has a catheter in the right radial artery that is being used for continuous arterial blood pressure monitoring following his abdominal aortic aneurysm surgery. Which observation by the nurse would require an emergency intervention? Calculated mean arterial pressure is 74 mm Hg. Patient's head of bed elevation is at 30 degrees. Capillary refill time in the right hand is 5 seconds. Pressure bag attached to the arterial line is inflated to 270 mm Hg.

Capillary refill time in the right hand is 5 seconds.

_?_- found in fetal gut, liver, pancreas. Found in colorectal cancer cells as well as cirrhosis of liver, ulcerative colitis, heavy smoking i. Useful as tumor markers to monitor effectiveness of therapy or indicate tumor recurrence 1. If elevated post-surgery, not all tumor has been removed 2. If elevated post-chemo, return/spread of cancer

Carcinoembryonic antigen (CEA)

Name 2 oncogenes

Carcinoembryonic antigen (CEA) Alpha-fetoprotein (AFP)

Examples of oncofetal antigens: a. _?_ - found in fetal gut, liver, pancreas. Found in colorectal cancer cells as well as cirrhosis of liver, ulcerative colitis, heavy smoking b. _?_- produced by malignant and fetal liver cells; elevated in testicular carcinoma, viral hep, liver disorders c. _?_ - ovarian

Carcinoembryonic antigen (CEA) α-fetoprotein (AFP) CA-125

_?_ type of neuroendocrine tumor. Slow-growing tumors, usually found in GI system (most often in rectum/small intestine); may spread to the liver or other sites in the body, and they may secrete substances such as serotonin or prostaglandins, causing carcinoid syndrome.

Carcinoid Tumors:

_?_ cancers often have a column-like shape when viewed under a microscope.

Carcinomas;

Signs of _?_ included elevated and equalized filling pressures.

Cardiac tamponade

What complication may occur post-CABG if the chest tube isn't patent?

Cardiac tamponade

_?_ is a type of infiltrative oncologic emergency. It is usually secondary to radiation therapy. S/S are distant heart tones, SOB, decrease LOC, decreased BP, Increased HR, decreased urine, decreased CO.

Cardiac tamponade

_?_ is rapid accumulation of blood in pericardial sac or medistinal cavity (like after open-heart sx); Compresses the heart (pericardium does not stretch); Prevents heart from pumping effectively; decreases CO, decreases myocardial perfusion resulting in chest pain

Cardiac tamponade

2 infiltrative oncologic emergencies include:

Cardiac tamponade Carotid artery rupture

Causes of _?_ Shock include: ACS, cardiomyopathy, cardiac tamponade, arrhythmias, structural abnormalities, MI, Severe systemic or pulmonary hypertension

Cardiogenic

_?_ - A group of diseases that directly affect the structural or functional ability of the myocardium.

Cardiomyopathy

three major organ systems most susceptible to complications during emergent burn phase are:

Cardiovascular Respiratory Urinary systems.

Care of NG/ Intestinal Tubes includes 4 things:

Care of NG/ Intestinal Tubes Mouth care/ Nose care Placement/flush Patency Clamping schedule

_?_ is a later post-operative complication of head/neck surgery related to: Infection Radiation side effects

Carotid Artery Rupture-

_?_ is a type of infiltrative oncologic emergency. It occurs when a tumor invades the arterial wall or as a complication to radiation treatment. Treatment is surgery. May use IV and blood products to stabilize prior to surgery

Carotid artery rupture

What are 2 infections which may be complications associated with ARDS?

Catheter related (foley, central/peripheral IV); sepsis

A 68-year-old man with suspected bacterial meningitis has just had a lumbar puncture in which cerebrospinal fluid was obtained for culture. Which medication should the nurse administer first? Codeine Phenytoin (Dilantin) Ceftriaxone (Rocephin) Acetaminophen (Tylenol)

Ceftriaxone (Rocephin)

_?_ is a medication which is a lymphocyte-specific inhibitor of purine synthesis with suppression of both B and T cells

Cell Cept

Chemotherapeutic drugs are classified by the specific types of action they exert in the cancer cell. The two major categories of chemotherapeutics are _?_ and _?_: Many times therapies are given in combination to maximize effectiveness

Cell Cycle Phase-Nonspecific Agents Cell Cycle Phase-Specific Agents

_?_ chemo has an effect on the cells during all phases of the cell cycle (cellular replication, proliferation, resting

Cell Cycle Phase-Nonspecific Agents:

_?_ chemo has the most significant effects during specific phases of the cell cycle.(ex: when the cells are in the process of replication or proliferation)

Cell Cycle Phase-Specific Agents:

Side effects of _?_include diarrhea, neutropenia, thrombocytopenia

CellCept

Testing for Diabetes Insipidus involves: 1. withhold fluids for 8-16 hours & assess B/P, weight, and urine osmolality hourly 2. then vasopressin administered and urine osmo measured in one hour _?_= rise in urine osmo _?_ = no response

Central DI ; Nephrogenic DI

1. _?_ Damage to central spinal cord Occurs most commonly in cervical cord region More common in older adults Motor weakness and sensory loss are present in both upper and lower extremities Upper extremities more affected than lower extremities

Central cord syndrome

The post-anesthesia care unit (PACU) has several patients with endotracheal tubes. Which patient should receive the least amount of endotracheal suctioning? Transplantation of a kidney Replacement of aortic valve Cerebral aneurysm resection Formation of an ileal conduit

Cerebral aneurysm resection The nurse should avoid suctioning the patient after a craniotomy until it is necessary because suctioning will increase this patient's intracranial pressure. The patients with a kidney transplantation, aortic valve replacement, or formation of an ileal conduit will not be negatively affected by suctioning, although it should only be done when needed, not routinely.

What are 4 causes of the neurological changes in the AKI patient?

Change in electrolytes, build up of waste products (BUN, Cr), hypotension (decreased kidney perfusion), excessive fluid (increases ICP)

Nursing interventions for Autonomic Hyperreflexia: 1. 2. 3. Assess for cause Immediate catheterization may be needed if bladder distention is cause (Lidocaine jelly should be instilled in urethra before cath to decrease stimulation) Digital exam of rectum to check for bowel impaction (only after anesthetic ointment used to decrease rectal stimulation and to prevent increase in symptoms) Remove restrictive clothing/tight shoes to decrease skin stimulation Teach patient and family causes, symptoms, and treatment May need alpha adrenergic blocker or arteriolar vasodilator

Check BP Elevate head of bed at 45 degrees or sit patient upright Notify physician

_?_ carcinogens - ex: alkylating agents such as Cytoxan

Chemical

Causes of hemothorax (5)

Chest trauma Lung malignancy Anticoagulant therapy Pulmonary embolus Tearing of pleural adhesions

Complications and Management after Thoracic Surgery: Prolonged air leak from _?_; requires _?_

Chest tube; requires chest tube to stay inlonger

What are some important diagnostic studies for patients with valve stenosis? Patient's history/physical exam _?_ ECG _?_

Chest x-ray Cardiac catheterization

The following are clinical manifestations of _?_ Indigestion to moderate to severe pain Fever Jaundice Tenderness in rt upper quadrant - referred pain to rt shoulder and scapula N/V Restlessness Diaphoresis If chronic in nature: intolerance to fatty foods, dyspepsia, heartburn, flatulence

Cholecystitis

The following are meds for _?_: Interferon (Intron A) Pegylated interferon (PegIntron, Pegasys) Nucleoside and nucleotide analogs Lamivudine (Epivir) Adefovir (Hepsera) Entecavir (Baraclude) Telbivudine (Tyzeka) Tenofovir (Viread)

Chronic HBV

The following are meds for _?_ Pegylated interferon Ribavirin (Rebetol, Copegus) Protease inhibitors Telaprevir (Incivek) Boceprevir (Victrelis)

Chronic HCV

_?_ - Develops over weeks or months after a seemingly minor head injury Peak incidence in ages 60's to 70's Presenting complaints often focal symptoms, not signs of ICP Delays in diagnosis for older adults because symptoms mimic those of vascular disease and dementia

Chronic Subdural Hematoma

_?_ is a type of lymphocytic leukemia Common in adults after age 40 Treatments for chronic types are delayed until symptoms develop

Chronic lymphocytic leukemia (CLL)

_?_ is a type of myelogenous leukemia Presence of Philadelphia chromosome / development of too many neurtophils Common adults age 40-50

Chronic myelogenous leukemia (CML)

_?_ is a continuous, prolonged, inflammatory, fibrosing process; progressive destruction as replaced by fibrotic tissue; may see remissions and exacerbations.

Chronic pancreatitis

_?_ lymphatic fluid in pleural space due to leak in the thoracic duct

Chylothorax:

Classification of Burn -- Location _?_ burns of the extremities can cause circulatory compromise. Patients may also develop _?_

Circumferential compartment syndrome.

New York Heart Association (NYHA) classifies Heart Disease as Classes I through IV. Class 1 is characterized by _?_, Class II is characterized by _?_, Class III is classified by _?_ while Class IV is characterized by _?_. What are the symptoms?

Class 1 - no limitation of physical activity Class II - Slight symptoms with activity Class III - less than ordinary physical activity causes symptoms Class IV - inability to carry on any physical activity without discomfort Fatigue, dyspnea, angina, palpitations

Skull fracture severity depend on the velocity (speed), momentum, and shape (blunt vs sharp) of the injuring agent and the site of impact to the skull. Skull fractures can be described as: _?_ or _?_ dependent on the presence of a scalp laceration or extension of the fracture into the air sinuses or dura A _?_ is a break in continuity of bone without altering relationship of parts. This means there is no shifting of bones of the skull. They are caused by blunt force trauma where the impact energy is transferred over a wide area of the skull. They are the most common type of skull fracture.

Closed or open; Linear fracture

_?_ is when air enters the intrapleural space through the lung, causing partial or total collapse of the lung.

Closed pneumothorax:

2 parts of the Lower GI

Colon Rectum

11 S&S of Addisonian Crisis

Coma and life threatening!, Hypotension, Tachycardia, Dehydration, Hyponatremia, Hyperkalemia, Hypoglycemia, Fever, weakness, confusion, N/V

9 s/s of Hypoxia

Combative behavior Dyspnea Decreased SpO2 (<80%) Dysrhythmias Cyanosis Hypertension Accessory muscle use Restlessness - 1st sign!!! Tachycardia

_?_skull fractures involve multiple linear fractures with fragmentation of bone into may pieces. Caused by direct, high momentum impact _?_ skull fractures involve inward indentation of skull into the brain caused by a powerful blow - getting struck by an object.

Communicated; Depressed

S/S of the _?_ stage of shock: Restless, apprehensive, change in LOC Increased heart rate Decreased B/P but adequate to perfuse vital organs Hypoactive bowel sounds Decreased urine Skin pale and cool except early septic shock Increased glucose Decreased SaO2

Compensatory

Stages of Shock _?_ Stage: Body attempts to overcome the increasing consequences of anaerobic metabolism and to maintain homeostasis If the cause of the shock is corrected, the patient will recover with little or no residual aftereffects If deficit not corrected, patient enters _?_ stage

Compensatory progressive

L3-4: Paraplegia - What can this person expect to do? Quadriceps and hip flexors, absence of hamstring function, flail ankles

Complete independent ambulation w/ short leg braces and canes. Unable to stand for long periods.

Classification of Spinal Injury: _?_-Results in total loss of sensory and motor function below level of lesion (injury) _?_- Results in a mixed loss of voluntary motor activity and sensation and leaves some tracts intact

Complete injury; Incomplete injury -(partial transection)

_?_ fractures occur when there is a laceration over a depressed skull fracture, which causes the internal cranial cavity to be exposed to the outside environment. These are caused by severe head injury and have a high risk of infection _?_ are either linear or depressed skull fractures without fragmentation or communicating lacerations. These are caused by low to moderate impact objects

Compound; Simple skull fractures

_?_- vertical compression Extension-rotation injury-

Compression injury (axial loading)

Nursing implementation for patients with IE: __?__ with immobility ROM TCDB Teach signs and symptoms of _?_. Teach reduction measures for risk for infection. Stress _?_ care.

Compression stockings infection follow-up

2 Diffuse injuries include:

Concussions Diffuse axonal injuries

Diagnosis of Pheochromocytoma: Consider this diagnosis when patient doesn't respond to _?_. Do not allow consumption of _?_

Consider this diagnosis when patient does not respond to usual anti-hypertensive drugs; Don't give anything that ↑B/P (like caffeine!)

A patient is diagnosed with acute pancreatitis. When providing dietary teaching, what are the points that a nurse should tell the patient? Select all that apply. Consume a low-fat diet. Avoid consuming alcohol. Limit fluid intake. Consume a high-carbohydrate diet. Consume a low-carbohydrate diet.

Consume a low-fat diet. Avoid consuming alcohol. Consume a high-carbohydrate diet.

A patient has been admitted to the neuro ICU following a craniotomy. To achieve a goal to maintain intracranial pressure within normal limits, the nurse should do the following: Select all that apply. Contact the physician if ICP measurement is greater than 20mmHg Perform active range of motion exercises on the patient Elevate the head of bed 30 degrees if CPP tolerates Use the Glasgow Coma Scale to monitor the patient's neurological status and trends Encourage coughing and deep breathing

Contact the physician if ICP measurement is greater than 20mmHg Elevate the head of bed 30 degrees if CPP tolerates Use the Glasgow Coma Scale to monitor the patient's neurological status and trends

Following a gastric resection, on post-op day 3, the patient's nasogastric tube is draining bile-colored liquid containing coffee-ground material. What is the best nursing action? Continue to monitor the amount of drainage and correlate it with any changes in vital signs. Reposition the NG tube and irrigate the tube with normal saline. Call the MD and discuss the possibility that the patient is bleeding. Irrigate the NG tube with iced saline solution and attach the tube to gravity suction.

Continue to monitor the amount of drainage and correlate it with any changes in vital signs.

_?_:manual exchanges 3-4 times a day - more labor intensive

Continuous Ambulatory Peritoneal Dialysis (CAPD)

_?_ provides a method to slowly remove solutes and fluids continuously in the hemodynamically unstable patient Only for those who do not need rapid resolution A more physiologic way to remove fluid over 24 hours

Continuous renal replacement therapy (CRRT)

The physician has decided to use renal replacement therapy to remove large volumes of fluid from a patient who is hemodynamically unstable in the intensive care unit. The nurse should expect which treatment to be used for this patient? Hemodialysis (HD) 3 times per week Automated peritoneal dialysis (APD) Continuous venovenous hemofiltration (CVVH) Continuous ambulatory peritoneal dialysis (CAPD)

Continuous venovenous hemofiltration (CVVH)

The patient with right upper quadrant abdominal pain has an abdominal ultrasound that reveals cholelithiasis. What should the nurse expect to do for this patient? Prevent all oral intake. Control abdominal pain. Provide enteral feedings. Avoid dietary cholesterol.

Control abdominal pain.

CMV

Controlled Mandatory Ventilation Ventilator does all the work. Set rate and tidal volume that is delivered with each breath. No accommodation for patient-initiated breaths. Administered to patient who are paralyzed or heavily sedated.

5. & 6. _?_ and _?_ Injury to these areas produces flaccid paralysis of lower limbs and areflexic (flaccid) bladder and bowel (Neurogenic bladder and bowels results.)

Conus Medullaris Syndrome and Cauda Equina Syndrome

_?_ is a spinal cord syndrome which causes flaccid paralysis of the lower limbs and areflexic bladder and bowel

Conus Medullaris syndrome; cauda equina

_?_ is enlargement of the RV caused by a primary disorder of the respiratory system Can be present with or without _?_ Most common cause is _?_

Cor Pulmonale cardiac failure COPD

Treatment for head/neck cancer may include: _?_-involves partial removal of 1 vocal cord d/t superficial tumor involving only one cord.

Cordectomy

_?_ is located on posterior and lateral side of the lower chest wall where diaphragm meets lower rib cage

Costophrenic angle

Chest tube care: Make sure connections secure (tape them or band them) _?_ q2 hrs and prn _?_at least q2hrs...helps with drainage and ventilation Don't strip or milk tubes unless MD orders for Must have clamp at bedside; occlusive dressings Don't clamp CT without order

Cough, deep breath Turn/reposition

_?_ - repair of cranial defect resulting from trauma, malformation, previous surgery....artificial material used to replace damaged or lost bone.

Cranioplasty

_?_ is an opening into cranium with removal of bone flap and opening the dura to remove lesion, repair damaged area, drain blood or relieve increased ICP

Craniotomy

The nurse reviews lab tests that have been prescribed for a patient in acute renal failure. Which is the best indicator of renal function? Potassium Creatinine BUN (blood urea nitrogen) ALT (alanine aminotransferase)

Creatinine

S/S of Severe Sepsis include: Creatinine > _?_ mg/dL Bilirubin > _?_ mg/dL Platelet count < _?_ μL Coagulopathy with INR > _?_ Abrupt change in _?_ status

Creatinine > 2.0 mg/dL Bilirubin > 2 mg/dL Platelet count < 100,000 μL Coagulopathy with INR > 1.5 Abrupt change in mental status

_?_ - # of cells needed to become detectable

Critical mass

_?_ - uses serum from the recipient mixed with donor lymphocytes to test for any preformed anti-_?_ antibodies to the potential donor organ; may be used as a screening tool when multiple possible living donors are being considered, or once a cadaver is selected.

Crossmatch HLA

4 Traumatic Causes of rhabdomyolysis:

Crush injury (auto accident, fall, building collapse), Prolonged immobilization (after fall, lying on hard surface during illness or under influence), Electrical shock injury, lightning strike, burn, Venom from snake or insect bite

_?_ means the cancer has been eradicated while _?_ means the cancer is being treated. _?_ is to control s/s to improve the quality of life

Cure Control Palliation

Thinning of hair, red cheeks, buffalo hump, muscle atrophy, poor wound healing, acne, moon face, purple striae, pendulous abdomen, and ecchymosis are all characteristics of someone with _?_

Cushing Syndrome

_?_ refers to the signs and symptoms associated with excess corticosteroids (especially glucocorticoids) in the body, regardless of the cause.

Cushing syndrome

Integrementary manifestations of DIC include 2 things

Cyanosis, ischemic tissue necrosis

_?_ - kill tumor cells; produce cytokines

Cytotoxic T Cells

Name 3 immunosuppressive agents: _?_ such as: Mycophenolate mofetil (CellCept), Sirolimus (Rapumune), azothioprine (Imuran), & cyclophosphamide (Cytoxan, Neosar) _?_ such as: muromonab-CD3 (Orthoclone OKT3), daclizumab (Zenapax), basiliximab (Simulect) _?_ such as : Lymphocyte immune globulin (Atgam)

Cytotoxic agents: Monoclonal Antibodies Polyclonal Antibody

Side effects of _?_ include neutropenia, hemorrhagic cystitis

Cytoxan:

_?_ is a medication which crosslinks DNA, leading to cell injury

Cytoxan; Neosar (cyclosphamide)

A patient is admitted to the ICU after a positive exercise treadmill test with a diagnosis of CAD and stable angina. Radiographic test show that the patient has blockage in the left main coronary artery and four other vessels. The nurse anticipates that the patient will be scheduled for A. medical therapy antianginal medications. B. PCI. C. TAVR. D. CABG.

D. CABG.

1. You are providing care to a patient with a chest tube. On assessment of the drainage system, you note continuous bubbling in the water seal chamber and oscillation. Which of the following is the CORRECT nursing intervention for this type of finding? A. Reposition the patient because the tubing is kinked. B. Continue to monitor the drainage system. C. Increase the suction to the drainage system until the bubbling stops. D. Check the drainage system for an air leak.

D. Check the drainage system for an air leak.

Which of the following is a late symptom of head and neck cancer? A. Hoarseness B. Change in the fit of dentures C. Mouth ulcers that do not heal D. Decreased mobility of the tongue

D. Decreased mobility of the tongue

The patient in room 2569 calls on the call light to tell you something is wrong with his chest tube. When you arrive to the room you note that the drainage system has fallen on its side and is leaking drainage onto the floor from a crack in the system. What is your next PRIORITY? A. Place the patient in supine position and clamp the tubing. B. Notify the physician immediately. C. Disconnect the drainage system and get a new one. D. Disconnect the tubing from the drainage system and insert the tubing 1 inch into a bottle of sterile water and obtain a new system.

D. Disconnect the tubing from the drainage system and insert the tubing 1 inch into a bottle of sterile water and obtain a new system.

A patient is admitted with left-sided heart failure and a blood pressure of 220/118 mm Hg. Which of the following drugs will be most effective in decreasing the blood pressure and reducing afterload? A. Dopamine (Intropin) B. Verapamil (Calan) C. Propranolol (Inderal) D. Sodium nitroprusside (Nipride)

D. Sodium nitroprusside (Nipride)

When assessing the patient upon return to the surgical unit following a total laryngectomy and radical neck dissection, the nurse would expect to find A. a closed wound drainage system. B. a nasal tracheal tube in place. C. an endotracheal tube and mechanical ventilation. D. placement of a nasogastric tube with orders for tube feedings.

D. placement of a nasogastric tube with orders for tube feedings.

Underproduction/undersecretion of ADH by the pituitary gland causes _?_

DI (diabetes insipidus)

Collaborative care for _?_ Early diagnosis Stabilize patient if needed Treat underlying cause Control thrombosis with _?_ or _?_ Control bleeding (replace blood products)

DIC (heparin or LMWH)

_?_ is a serious bleeding and thrombotic disorder d/t abnormal initiated and accelerated clotting Then decreases in clotting factors and platelets occur Uncontrollable hemorrhaging can occur

DIC (Disseminated Intravascular Coagulation)

_?_ is not a disease. Instead, it is an abnormal response of the normal clotting cascade caused by disease process or disorder.

DIC (Disseminated Intravascular Coagulation)

Ms. Schudde recovered from her AKI. What health promotion activities would you discuss with her prior to discharge? Daily Weights Diet __?_ Hydration Meds _?_ IV dye _?_ Lab Monitoring _?_ Infections _?_ F/U appointments

Daily Weights Diet - low Na, low protein, increased carb and fat Hydration Meds - no nephrotoxic drugs IV dye renal dosing with bicarb Lab Monitoring f/e, BUN, Cr, Urine Output Infections treat immediately F/U appointments

After a subtotal gastrectomy, the nurse should anticipate that NG tube drainage will be what color for about 12-24 hours after surgery? Dark brown Bile green Bright red Cloudy white

Dark brown

Burn Wound Care: _?_ : Removal of necrotic skin Done in tub, bed, surgery May have daily dressing changes Infection may cause further skin injury or lead to _?_ . _?_ IS THE PRIMARY GOAL FOR BURN WOUND CARE! _?_ skin (from skin donor cadavers) is used, although there are newer biosynthetic options.

Debridement; sepsis PERMANENT SKIN COVERAGE Allograft (homograft)

_?_ posturing occurs because of midbrain and brain stem injury.

Decerebrate posturing (extensor)

What is the #1 medication which can be used for SIADH? What is its method of action?

Declomycin; blocks the effect of ADH on the renal tubules which allows for more dilute urine

Treatment for bowel obstruction may include: decompression of the intestine using _?_ Correction and maintenance of _?_ Relief or removal of the obstruction

Decompression with NG tube fluid and electrolyte balance

_?_ : Removal of fibrous membrane from visceral pleura. _?_ : Artificial production of adhesions between parietal and visceral pleura _?_ : Resection of the most damaged portions of lung tissue, allows more normal chest wall configuration. _?_ : Rigid scope with distal lens inserted into the pleura; allows manipulation of instruments through separate, small intercostal incisions

Decortication Pleurodesis Lung volume reduction surgery (LVRS) Video-assisted thoracic surgery (VATS)

During hemodialysis, the patient develops light-headedness and nausea. What should the nurse do for the patient? Administer hypertonic saline. Administer a blood transfusion. Decrease the rate of fluid removal. Administer antiemetic medications.

Decrease the rate of fluid removal.

Problems caused by increased thoracic pressure

Decreased BP --> decreased tissue perfusion. Decreased blood flow back to heart --> INCREASED ICP, decreased myocardial and pulmonary perfusion.

When caring for a critically ill patient who is being mechanically ventilated, the nurse will astutely monitor for which clinical manifestation of multiple organ dysfunction syndrome (MODS)? Increased serum albumin Decreased respiratory compliance Increased gastrointestinal (GI) motility Decreased blood urea nitrogen (BUN)/creatinine ratio

Decreased respiratory compliance

What are 6 LATE s/s of IICP?

Decreasing LOC Pupillary changes Motor function Changes (hemiplegia or hemiparesis) posturing Changes in vital signs Projectile vomiting w/o nausea

What is Being Tested? _?_ may be diminished by abnormalities in muscles, sensory neurons, lower motor neurons, and the neuromuscular junction; acute upper motor neuron lesions; and mechanical factors such as joint disease. Abnormally increased reflexes are associated with upper motor neuron lesions. Note that deep tendon reflexes can be influenced by age, metabolic factors such as thyroid dysfunction or electrolyte abnormalities, and anxiety level of the patient.

Deep tendon reflexes

_?_ problem- especially common first 3 months Signs and symptoms - normal signs of tenderness and pain not present _?_ a leading cause of death

Deep vein thrombosis (DVT); Pulmonary embolism

5 disorders of the anterior pituitary related to underproduction:

Deficiency of TSH Deficiency of ACTH Deficiency of GH Deficiency of FSH and LH Hypothyroidism (deficiency of TSH)

The nurse knows the patient with AKI has entered the diuretic phase when what assessments occur (select all that apply)? Dehydration Hypokalemia Hypernatremia BUN increases Serum creatinine increases

Dehydration Hypokalemia

Patients with IE should receive prophylactic treatment with abx when patients are undergoing 4 types of procedures:

Dental Respiratory tract Gastrointestinal Genitourinary

Last 2 stages of grief and mourning post-injury 3._?_ - sadness, pessimism, anorexia, nightmares, insomnia, agitation, suicidal thoughts, refuses to participate in self-care activities Nursing Interventions: encourage family involvement, plan steps in rehab to show signs of success, give cheerful and will assistance in ADL, avoid sympathy, use firm kindness 4. _?_ - planning for future, active participation in therapy, find personal meaning in experience and continue to grow, return to pre-injury personality Nursing Interventions: remember each patient has own personality, balance support systems to encourage independence, set goals with pt input, emphasize potentials.

Depression Adjustment and Acceptance:

_?_treats vasogenic edema surrounding tumor or abscesses Dosing - 10mg IV, followed by 4mg IM every 6 hours until symptoms of cerebral edema subside Not recommended for head injury patients Side effects - hyperglycemia, risk for infection, GI bleeding Monitor - fluid intake, Na & glucose levels, signs of GI bleeding

Dexamethasone (Decadron) - a corticosteroid

Classification of burn injury: Patient Risk Factors Preexisting comorbidities: cardiovascular, respiratory, or renal disease _?_ or peripheral vascular disease - risk for healing Physical debilitation - chronic disease (alcoholism, drug abuse, or malnutrition.) Patients with other injuries: head injuries, fractures, or other trauma Older adults heal slower than younger adults

Diabetes

ADH insufficiency causes _?_

Diabetes insipidus

7 Major causes of CKD:

Diabetic nephropathy Hypertensive nephrosclerosis Chronic glomerulonephritis Chronic pyelonephritis Systemic lupus erythematosus Polycystic kidney disease Autoimmune process

_??_ - Movement of fluid and molecules across a semipermeable membrane from the blood to the dialysate solution.

Dialysis

_?_ is used to correct fluid and electrolyte imbalances and to remove waste products in renal failure (can also be used for some drug overdoses). Begun when uremia can no longer be treated with conservative medical management (usually GFR< 15)

Dialysis

Nursing Assessment for AA - Monitor for signs of aneurysm rupture, such as_?_, pallor, weakness, _?_, _?_, abdominal-back-groin-or periumbilical pain, changes in LOC, or a pulsating abdominal mass.

Diaphoresis; tachycardia; hypotension

4 components of CKD treatment

Dietary and fluid management Pharmacology Supportive and surgical therapy Dialysis

Clinical signs of a _?_ Decreased level of consciousness Increased intracranial pressure (ICP) Decerebration or decortication (posturing) Global cerebral edema

Diffuse Axonal Injury (DAI)

The following describes _?_ Widespread axonal damage occurring after a mild, moderate, or severe TBI Major cause of unconsciousness and persistent vegetative state after head trauma Develops within 12 to 24 hours Diffuse injury due to damage occurring over a widespread area of the brain versus a specific part (focal injury) Damage occurs around Axons in subcortical white matter of the cerebral hemispheres Basal Ganglia Thalamus Brainstem

Diffuse Axonal Injury (DAI)

Principles of dialysis - uses concentration gradients _?_ - movement of solutes from area of greater to lesser concentration (urea, creatinine, uric acid, and electrolyes move from blood to dialysate

Diffusion

3 Principles of dialysis - uses concentration gradients

Diffusion Osmosis Ultrafiltration

_?_ is when gas exchange is compromised d/t alveolar changes

Diffusion limitation

Collaborative Care for Valve Stenosis: Low Sodium Diet Drug therapy to treat/control HF (name 4) Other drug therapies (name 2)

Digitalis (positive inotropes) Vasodilators Diuretics Beta-Blockers Anticoagulants Antidysrhythmics

Bradycardia and widening of the QRS complex may occur with infusion of _?_ especially if it is rapidly infused. Hypotension may also develop. If any of these adverse effects occur, infusion rate should be slowed or stopped until the physician is consulted.

Dilantin (phenytoin) - an anticonvulsant

_?_ - Most common form of cardiomyopathy; Rapid degeneration of myocardial fibers resulting in ventricular dilation, impairment of systolic function, atrial enlargement, and stasis of blood in the left ventricle

Dilated Cardiomyopathy

What are the 3 major types of cardiomyopathy:

Dilated cardiomyopathy Hypertrophic cardiomyopathy Restrictive cardiomyopathy

_?_ is an increasingly rare syndrome characterized by neurologic symptoms of varying severity that affect dialysis patients, particularly when they are first started on hemodialysis. It is thought to be due primarily to cerebral edema. S/S: confusion, HA, Nausea with or without vomiting, Dizziness, Muscle cramp, Blurred vision, Tremors, Restlessness, Agitation, Alteration of consciousness, Seizure, Coma, Sudden cardiac arrest Treatment is to infuse _?_

Disequilibrium Syndrome Treatment: 3-5 hypersaline

The patient was diagnosed with prerenal AKI. The nurse should know that what is most likely the cause of the patient's diagnosis? IV tobramycin (Nebcin) Incompatible blood transfusion Poststreptococcal glomerulonephritis Dissecting abdominal aortic aneurysm

Dissecting abdominal aortic aneurysm

Signs of a _?_ Small bowel obstruction (SBO): Emesis (orange-brown in color, foul odor like feces, not as rapid onset; multiple areas of fluid on xray; abdominal pain

Distal

Septic shock is also a _?_Shock Before talking about septic shock, need to talk about SIRS, Sepsis, Severe Sepsis.

Distributive

Cor Pulmonale: Collaborative Care - Pharmacological management _?_: helps decrease plasma volume, reduce heart workload _?_ -indicated if obstructive resp disorder. Vasodilator therapy, CCBs and anticoagulants _?_ - help due to weak inotropic effect on the heart _?_- increase cardiac contractility.

Diuretics & low Na diet Bronchodilator therapy Theophylline Digitalis

2 drug therapies for ascites

Diuretics and spironolactone

Bleeding can also occur in the lower GI tract (colon) Name several causes of Lower GI Bleed:

Diverticular bleeding Ischemia Hemorrhoids, fissures Neoplasia Irritable Bowel, infectious colitis, post polypectomy Arteriovenous malformations Unknown causes

What will be done if a patient needs a stress test but can't exercise?

Dobutamine or Adenosine stress test

Reflexes - _?_ - Only do this test if there is no reason to suspect cervical spine damage. Nurse turns pt's head briskly to left or right while holding eyelid open. Normal response is movement of eyes in the opposite direction than the turning. Next the nurse quickly flexes and then extends the neck. Eye movement should be opposite to the direction of head movement: Up when neck is flexed and down when neck is extended.

Doll's Eyes (oculocephalic reflex response)

After administering a dose of promethazine (Phenergan) to a patient with nausea and vomiting, what common temporary adverse effect of the medication does the nurse explain may be experienced? Tinnitus Drowsiness Reduced hearing Sensation of falling

Drowsiness

What are 5 diseases which cause abnormalities of the CNS that lead to a decrease in ventilatory supply?

Drug overdose Brainstem infarction Spinal cord injury Abnormalities of the chest wall Neuromuscular conditions

With drug-eluting stents, _?_ is necessary; it helps prevent stent thrombosis

Dual Anti-platelet (Plavix and Aspirin)

The patient is having a gastroduodenostomy (Billroth I operation) for stomach cancer. What long-term complication is occurring when the patient reports generalized weakness, sweating, palpitations, and dizziness 15 to 30 minutes after eating? Malnutrition Bile reflux gastritis Dumping syndrome Postprandial hypoglycemia

Dumping syndrome

in _?_, the cells look abnormal under a microscope, but are not yet cancerous.

Dysplasia

6 s/s of hypercapnia

Dyspnea Headache Decreased tidal volume Decreased LOC Pursed lip breathing Tripod position

8 Clinical s/s of hemothorax:

Dyspnea Tachycardia Possible chest pain Dullness to percussion over affected area Decreased or absent breath sounds over affected area If large amount of blood: signs of shock, decreased H&H & SaO2

Clinical Manifestations of IPAH: Classic symptoms: _?_ and _?_ Chest pain, dizziness, syncope on exertion

Dyspnea & fatigue on exertion

Clinical manifestations of Mitral Valve Prolapse: _?_ (e.g., premature ventricular contractions, paroxysmal supraventricular tachycardia, ventricular tachycardia) Palpitations Light-headedness Dizziness

Dysrhythmias

Complications and Management after Thoracic Surgery: Cardiovascular disturbances: _?_ d/t hypoxia and pulmonary edema

Dysrhythmias

Objective data for valve stenosis: Abnormal heart sounds (S3 S4) Tachycardia _?_ _?_ _?_ _?_ Peripheral edema Ascites Crackles, wheezes, hoarseness

Dysrhythmias Hypotension Hepatomegaly Diaphoresis

Cardiovascular complications post burn: _?_ and _?_. Circulation to extremities can be severely impaired by deep circumferential burns acting like tourniquets to areas leading to ischemia, paresthesia, and necrosis --- perform escharotomy --- to restore circulation to compromised extremities.

Dysrhythmias hypovolemic shock

_?_ are the most common complication post-MI. _?_ are drugs that may be used to treat this complication

Dysrhythmias; antidysrhythmic drugs

Nursing Implementation for Shock: Neurologic status: orientation and level of consciousness Cardiac status Continuous _?_ VS, capillary refill Hemodynamic parameters (if available): central venous pressure, PA pressures, CO, PAWP Heart sounds: _?_, _?_, _?_

ECG murmurs, S3, S4

Cardiovascular manifestations of DIC include 2 things

EKG changes, venous distention

Diagnostics for Chronic Pancreatitis Usually mild leukocytosis and increased sed rate Secretin stimulation test - used to assess degree of pancreatic function _?_ used to see pancreatic and common bile ducts CT, MRI, ultrasound - show calcifications, ductgal dilation, pseudocysts, pancreatic enlargement

ERCP

Collaborative Care for Gallbladder disease: ERCP w/ _?_ Extracorporeal shock-wave lithotripsy _?_ : Ursodeoxycholic acid (Actigall); chenodeoxycholic acid (chenodiol) _?_ : laparoscopic cholecystectomy; open cholecystectomy

ERCP w/ sphincterectomy Extracorporeal shock-wave lithotripsy Dissolution therapy: Ursodeoxycholic acid (Actigall); chenodeoxycholic acid (chenodiol) Surgical therapy: laparoscopic cholecystectomy; open cholecystectomy

Clinical Manifestations of Spinal Cord Injury: _?_- cause deficit above and below the area of the injury. By -?_, permanent damage may occur because of edema Edema secondary to inflammatory response is harmful because of lack of space for tissue expansion Resultant compression of cord and extension of edema above and below injury increase ischemic damage Extent of injury and prognosis for recovery most accurately determined at least _?_or more after injury

Edema; ≤24 hours 72 hours

A 63 year old female patient is admitted to Cardiac Recovery Unit following coronary artery bypass graft surgery. The patient has a pulmonary artery catheter and the nurse notes the following: pulmonary artery pressure of 45/24 mmHg with a pulmonary artery wedge pressure of 22mmHg and SVR of 1200. The patients cardiac output is 3.5 L/min and her index is 1.9 L/min/m2. What is your assessment of these values: Elevated preload and afterload Elevated preload and low cardiac output Values are normal for this pt. Elevated afterload and low cardiac output

Elevated preload and low cardiac output

Signs of VAP

Elevated temp and WBCs. Purulent sputum. Odorous sputum. Crackles or rhonchi on auscultation. Pulmonary infiltrates on X-ray.

Chest Surgery Pre-Op Care: Baseline Pulmonary Function Tests, chest x-ray, BMP, CBC, EKG, ABG _?_ _?_ to drain lungs of secretions Pre-op teaching: _?_, _?_ and _?_; teach about CT and O2 use

Encourage to stop smoking; CPT (chest physiotherapy) Teach: Cough, Deep Breathe; Incentive Spirometer, splinting, CT and O2

_?_- seepage of blood back into old aneurysm d/t poor seal, tear in graft, leak between overlapping graft segments

Endoleak

10 Complications of Repair of aneurysm repair:

Endoleak, new aneurysm growth, rupture, aortic dissection, bleeding, rental artery occlusion, stent migration, thrombosis, hematoma, infection

A patient is seeking emergency care after choking on a piece of steak. The nursing assessment reveals a history of alcoholism, cigarette smoking, and hemoptysis. Which diagnostic study is most likely to be performed on this patient? Barium swallow Endoscopic biopsy Capsule endoscopy Endoscopic ultrasonography

Endoscopic biopsy

Treatment if esophageal or gastric varcies do begin to bleed: Name 3 or 4 therapies that may be used

Endoscopic therapies Balloon tamponade, transjugular intra-hepatic porto- systemic shunt (TIPS), DSRS (distal splenorenal shunt)

In _?_, Place sutureless aortic graft into abdominal aorta inside aneurysm via femoral artery, Graft supported by multiple flexible wire rings, When graft components in place, released against wall with balloon inflation, Blood goes through graft, prevents expansion of aneurysm, Aneurysm wall shrinks overtime as not experiencing pressure of blood anymore

Endovascular Graft Procedure (EVAR

_?_ allow for safe graft insertions; Minimally invasive; Must meet certain criteria: iliofemoral vessels long and wide enough to support graft

Endovascular Graft Procedure (EVAR);

2 ways to repair aortic dissection:

Endovascular aortic dissection repair, Open Surgical aortic resection and repair

5 Manifestations of Acromegaly: Musculoskeletal

Enlargement of hands and feet, Fingertips club, forehead enlarges, Maxilla lengthens, Coarsening of facial features, Tongue enlarges and voice deepens,

Signs of_?_ Initial period of unconsciousness Brief lucid interval followed by decrease in level of consciousness Headache Nausea vomiting Focal Findings

Epidural Hematomas

_?_ - results from bleeding between the dura and the inner surface of the skull; Neurologic emergency; Origin can be either venous or arterial but is usually _?_

Epidural hematoma; arterial

Name 3 catecholamines released by the adrenal medulla.

Epinephrine(major hormone) Norepinephrine Dopamine;

5 Manifestations of Pheochromocytoma

Episodes of severe hypertension; Severe headache, tachycardia, profuse sweating, unexplained abdominal and/or chest pain;

In _?_ cancer, the cause unknown. Risk factors include: Heavy alcohol use Smoking Central body obesity Diet low in fruits/vegetables Injury to esophageal mucosa Barrett's metaplasia Genetic: overexpression/ mutations of tumor suppressor genes

Esophageal cancer

S/S of _?_ may include: Halitosis Sore throat Choking Hoarseness Hemorrhage if cancer erodes through esophagus into aorta N/V Changes in bowel habits (constipation, diarrhea, bleeding)

Esophageal cancer

S/S of _?_ may include: Progressive dysphagia Painful swallowing Weight Loss/Malnutrition Regurgitation of blood flecked esophageal contents Weakness Pain (Late symptom) - occurs substernal, epigastric or back areas; increases with swallowing May radiate to neck, jaw, ears, shoulders

Esophageal cancer

Treatment of _?_ depends on location of tumor and if metastasis/invasion occurred Poor prognosis due to usually late diagnosis Best results with combination of surgery, endoscopy ablation, chemotherapy and radiation

Esophageal cancer

_?_ Tortuous veins at lower end of esophagus, enlarged & swollen d/t portal hypertension

Esophageal varices

2 Complications of portal hypertension:

Esophageal varices Gastric varices

_?_: remove part or all of esophagous and use dacron graft to replace resected area

Esophagectomy

Subjective data in nursing assessment for patient with IE: Diaphoresis Bloody urine _?_ Generalized weakness Fatigue Cough _?_ Night sweats Chest, back, abdominal pain

Exercise intolerance Dyspnea on exertion

Nursing assessment for Respiratory failure and ARDS: _?_ Immunizations Anorexia Weight gain/loss Diaphoresis Dizziness _?_ wheezing, cough, sputum, palpitations, swollen feet Changes in _?_ Fatigue

Exercise; Dyspnea, sleep pattern

Chronic aortic regurgitation Remains asymptomatic for years and then...(3 s/s)

Exertional dyspnea Orthopnea Paroxysmal nocturnal dyspnea

Clinical manifestations of mitral valve stenosis: _?_ Primary symptom due to reduced lung compliance Palpitations from _?_ Fatigue Accentuated _?_ heart sound Low-pitched, rumbling _?_ murmur Hoarseness Hemoptysis Chest pain Seizures Stroke

Exertional dyspnea atrial fibrillation first diastolic

ECMO

Extracorporeal Membrane Oxygenation. Modification of cardiac bypass. Large-bore catheters are inserted, blood is removed, oxygenated, CO2 is removed, and then returned to body.

Nursing and collaborative Management Emergent cont., other care measures Other care measures---- they are very important, too. Name 6

Eye care Ears Elevate arms; hand splints Keep perineum clean and dry - remove Foley catheter when no longer needed. PT/ROM oral care

How do you assess cranial nerves? Eye movements nerve # _?_ Hearing nerve # _?_ Shrug shoulders & turn head nerve # _?_ Tongue movement nerve # _?_

Eye movements nerve # 3 4 6 Hearing nerve # 7 Shrug shoulders & turn head nerve # 11 Tongue movement nerve #12

The nurse is providing care for a patient who has been admitted to the hospital with a head injury and who requires regular neurologic and vital sign assessment. Which assessments will be components of the patient's score on the Glasgow Coma Scale (GCS) (select all that apply)? Judgment Eye opening Abstract reasoning Best verbal response Best motor response Cranial nerve function

Eye opening Best verbal response Best motor response

The nurse is caring for a 68-year-old woman after a parathyroidectomy related to hyperparathyroidism. The nurse should administer IV calcium gluconate if the patient exhibits which clinical manifestations? Facial muscle spasms or laryngospasms Decreased muscle tone or muscle weakness Tingling in the hands and around the mouth Shortened QT interval on the electrocardiogram

Facial muscle spasms or laryngospasms

4 Cranial nerve manifestations of GBS

Facial weakness Dysphagia Diplopia Difficulty speaking

_?_- Is a disruption of all arterial wall layers with bleeding contained by surrounding anatomic structures

False aneurysm (pseudoaneurysm)

The nurse is caring for a group of older patients in a long-term care setting. Which physical changes in the patients should the nurse investigate as signs of possible endocrine dysfunction? Absent reflexes, diarrhea, and hearing loss Hypoglycemia, delirium, and incontinence Fatigue, constipation, and mental impairment Hypotension, heat intolerance, and bradycardia

Fatigue, constipation, and mental impairment

O2 concentration delivered to patient is called _?_. It may be set between _?_ % and _?_ %.

FiO2; 21% to 100%

State whether the following labs are high or low with DIC: Fibrin Split Products (FSP) or FDP Fibrinogen level Platelet count PTT & PT INR Thrombin time D-dimer

Fibrin Split Products (FSP) or FDP - high Fibrinogen level - low Platelet count - low PTT & PT - high INR - high Thrombin time - high D-dimer - high

_?_ Results from multiple rib fractures causing instability of the chest wal

Flail Chest:

What are 7 abnormalities of the chest wall that may lead to a decrease in ventilatory supply?

Flail chest Kyphoscoliosis Morbid obesity Fracture Mechanical restriction Muscle spasm Elderly

_?_: most unstable of all injuries

Flexion-rotation injury

Signs & Symptoms of Oliguric Phase of AKI Fluid volume _?_ and 4 electrolyte imbalances:

Fluid volume excess, Sodium imbalance, Potassium excess, Calcium Imbalance Phosphorous imbalance

A patient with type 2 diabetes mellitus has been diagnosed with nonalcoholic fatty liver disease (NAFLD). The nursing teaching plan should include which of the following? Having genetic testing done. Eliminating carbohydrates from the diet. Following measures to gain tighter glucose control. Avoiding alcohol until liver enzymes return to normal

Following measures to gain tighter glucose control.

FiO2

Fraction of inspired oxygen delivered to the patient. May be between 21% (normal room air) and 100%.

T1-6: Paraplegia - What can this person expect to do? Full innervation of upper extremities. Back, essential intrinsic muscles of hand; full strength and dexterity of grasp. Decreased trunk stability. Decreased resp reserve.

Full independence in self-care and in w/c. Able to drive care with hand controls (most pts). Independent standing in standing frame.

T6-12: Paraplegia - What can this person expect to do? Full, stable thoracic muscles and upper back. Functional intercostals, results in increased resp reserve.

Full independent use of w/c. Able to stand erect w/ full leg brace, ambulate on crutches w/ swing (has gait difficulty). Not able to climb stairs.

Overall mortality rate for acute hepatitis is < 1%. _?_ viral hepatitis: severe impairment or necrosis of liver cells, hepatic failure Chronic hepatitis - most at risk with Hep _?_ Cirrhosis of the liver Hepatocellular carcinoma

Fulminant C

_?_aneurysm is circumferential, relatively uniform in shape

Fusiform

No single clinical or lab finding confirms _?_ Lumbar Puncture may be done Peripheral blood test Electrophysiologic studies (Eps) Electromyographic test (EMG) and nerve conduction velocity (NCV) MRI or CT ABG

GBS

What are the "Risk" criteria for classification of AKI for both GFR and urine output:

GFR: increased creatinine (1.5 times) or GFR decrease > 25%; Urine: output < 0.5 ml/kg/h x 6 hour

What are the "Injury" criteria for classification of AKI for both GFR and urine output:

GFR: increased creatinine (2 times) or GFR decrease > 50%; Urine: output < 0.5 ml/kg/h x 12 hour

What are the "Failure" criteria for classification of AKI for both GFR and urine output:

GFR: increased creatinine (3 times) or GFR decrease > 75% or creatinine > 4 mg per 100 mL; Urine: output < 0.3 ml/kg/h x 24 hour or anuria x 12 hour

What are the 3 most common deficiencies associated with hypopituitarism?

GH and gonadotropins (LH, FSH)

Cancer pain is usually treated with NSAIDS and narcotics. Complications of NSAID use are _?_ and _?_ 3 complications of narcotic use are:

GI issues/bleeding and renal Respiratory depression, N/v, Constipation

_?_ is complete penetration of any part of the wall of the gastrointestinal tract in which intestinal contents flow into the abdominal cavity Potential for bacterial contamination of the abdominal cavity called _?_

GI perforation Peritonitis

Once organ donation has been authorized, _?_assumes medical care of donor and sends medical information about the donor to _?_ for matching with potential recipients.

GOH's Organ Recovery Coordinator United Network for Organ Sharing (UNOS)

How do you assess cranial nerves? Gag & Cough reflex for nerve # _?_ Corneal reflex for nerve # _?_ Pupillary reflex for nerve # _?_

Gag & Cough reflex for nerve #9 Corneal reflex for nerve # 5 Pupillary reflex for nerve # 3

_?_ Located in upper portion of stomach (fundus); d/t portal hypertension

Gastric varices

What are the 6 categories of Objective Data Collected during a nursing assessment on a patient in acute respiratory failure?

General (restlessness) Integumentary (cool, pale, clammy or warm flushed) Respiratory (shallow with increased RR) Cardiovascular (tachy progressing to brady) GI (ascites Neurologic (Somnolence) Possible Diagnostic Findings (lab values, etc)

_?_: type of tumor that begins in the cells that give rise to sperm or eggs. These tumors can occur almost anywhere in the body and can be either benign or malignant.

Germ Cell Tumors

Federal law requires hospitals to notify _?_ each time a patient dies or is about to die to determine if patient is a potential donor.

Gift of Hope

Adrenal Cortex: Synthesizes 3 classes of steroid hormones, called collectively corticosteroids. They are:

Glucocorticoids Mineralocorticoids Androgens

Diagnostic Labs for DKA: Glucose = _?_ Na _?_ K _?_ BUN _?_ Cr _?_ Urine Ketones _?_ Serum ketones _?_ Serum osmo _?_ pH _?_

Glucose >300 mg/dl Na - high K - low BUN >30 mg/dl Cr >1.5 mg/dl Urine Ketones present Serum ketones - present Serum osmo - increased pH < 7.3

Diagnostic Labs For HHS: Glucose Na K BUN Cr Urine ketones Serum ketones Serum osmo pH

Glucose >600 mg/dl Na - high K - low BUN >30 mg/dl Cr >1.5 mg/dl Urine ketones - negative Serum ketones - negative Serum osmo - > 320 pH > 7.4

4 Metabolic functions of the Liver: Also: Regulates Blood Coagulation Detoxification Drug & Toxin metabolism Bile synthesis & secretion Bile formation Bilirubin excretion

Glucose metabolism Ammonia conversion Protein metabolism Fat metabolism, vitamin & iron storage

L1-2: Paraplegia - What can this person expect to do? Varying control of legs and pelvis, instability of lower back.

Good sitting balance. Full use of w/c. Ambulation w/ long leg braces.

Histologic classification of cancer. Grade I = Grade II = Grade III = Grade IV = Grade X =

Grade I = mild dysplasia Grade II = moderate dysplasia Grade III = severe dysplasia Grade IV =anaplasia/undifferentiated Grade X = grade can't be assessed

__?__ occurs when an immunodeficient patient is transfused with immunocompetent cells. Begins _?_ days post transplant

Graft vs Host Disease (GVHD) 7-30

While ARDS may be the result of a direct lung injury to lung epithelium or may develop as a consequence of Systemic Inflammatory Response Syndrome (SIRS) in which mediators are transported to the lungs, the 2 most common risk factors are: _?_ and _?_

Gram negative sepsis Aspiration of gastric contents

5 hormones secreted by the Anterior pituitary

Growth hormone (GH) Prolactin Thyroid Stimulating Hormone (TSH) Adrenocorticotropic Hormone (ACTH) Follicle-stimulating Hormone (FSH)

_?_ is also known as acute idiopathic polyneuritis or polyradiculoneuropathy

Guillain Barre Syndrome

_?_ is Demyelination of peripheral nerves Progressive motor weakness & sensory abnormalities occur _?_ Paralysis Symptoms in feet/legs & spread to arms/upper body Paralysis intensity can result in almost total immobility May require _?_

Guillain Barre Syndrome; Ascending; mechanical ventilation

Name 3 causes of SIADH that relate to the nervous system:

Guillian Barre, skull fx, subdural hematoma (these are usually self-limiting)

Nursing Assessment for AA - Thorough _?_ assessment , Look for signs of coexisting cardiac, pulmonary, cerebral, and lower extremity vascular problems

H & P

6 Diagnostic Studies of Aortic Dissection

H and P, ECG CXR, CT, TEE, MRI

What medications can be expected to help prevent aspiration?

H2 receptor blockers (Zantac) and proton pump inhibitors (Nexium, Protonix). Decrease gastric acid secretions.

Treatment for _?_: Stop heparin, switch to direct thrombin inhibitor (argatroban=Acova) or Factor Xa inhibitor (rivaroxaban = Xarelto; apixaban = Eliquis) NS flush for IVs If clotting severe, plasmapheresis, protamine to counteract circulating Heparin, thrombolytic agents/surgery for clots No platelet transfusion: could make it worse NO LMWH; NO HEPARIN....never, never, never, never again

HIT

S/S of _?_ Bleeding Purpura Petichiae Superficial ecchymosis Hemorrhage (spontaneous ICB may occur w/plt < _?_)

HIT 20,000

_?_ - suspect if 50% or more reduction in platelets OR plt count < 150,000 Results in venous &/or arterial thrombosis

HIT: (heparin included thrombocytopenia)

Nursing interventions to prevent VAP

HOB elevated to 30 - 45 degrees. Use of an ET-tube with a suction port above the cuff. Proper and regular oral care. Oral suctioning prn. HAND WASHING and proper aseptic/clean technique as indicated.

Prevention of intraabdominal hypertension and compartment syndrome (4):

HOB elevation, hemodynamic stability, pain and anxiety control

Conservative management of Aortic Dissection: _?_ and _?_ control

HR and BP (controls aortic wall stress by decreasing systolic PB and myocardial contractility)

Complications of _?_ Bacterial, viral, and fungal infections are common. Prophylactic antibiotic therapy Graft-versus-host disease T lymphocytes from donor stem cells recognize recipient as foreign Attacks organs such as skin, liver, and GI tract Peripheral blood stem cells cause fewer and less severe complications More mature than stem cells from bone marrow

HSCT (hematopoietic stem cell transplant)

5 cardiovascular clinical manifestations of CKD

HTN HF CAD Pericarditis PAD

Manifestations of hyperparthyroidism: 2 Cardiovascular:

HTN, arrhythmias

Phases of Burn Management: Emergent/Resuscitative Phase Fluid and electrolyte shift -greatest initial threat to pt w/ major burn = _?_ D/T massive shift of fluids out of the blood vessels; result of increased capillary permeability - start early as _?_ post-burn. With capillary wall more permeable - water, sodium and _?_ move into interstitial spaces and surrounding tissue. All above is termed _?_

HYPOVOLEMIC SHOCK. 20 minutes plasma proteins (especially ALBUMIN) intravascular volume depletion

_?_ and _?_ cancers are usually squamous cell carcinomas Characterized by: Altered cells of the respiratory tract Different levels of invasion Metastasis (lungs are a common site)

Head and Neck

Etiology of _?_ may include: Prolonged smoking and alcohol use HPV infection Diets poor in fruits and vegetables Diets high in processed and red meats Association w/ Epstein-Barr virus May also be a result of polluted air, inhalation of chemicals, and voice abuse. Head and neck cancer increases with age Most occur in people > 60 yrs of age Ethnicity

Head and Neck Cancer

Which manifestations in a patient with a T4 spinal cord injury should alert the nurse to the possibility of autonomic dysreflexia? Headache and rising blood pressure Irregular respirations and shortness of breath Decreased level of consciousness or hallucinations Abdominal distention and absence of bowel sounds

Headache and rising blood pressure

What does chronic rejection cause in each of the following organs? 1. Heart 2. Lungs 3. Liver 4. Kidney

Heart: accelerated CAD Lungs: bronchiolitis obliterans Liver: loss of bile ducts Kidney: fibrosis and glomerulopathy

What are the clinical manifestations of AKI for each of these systems? Hematologic (1) Neurologic (2) Metabolic (5)

Hematologic - anemia within 48 hours Neurologic - confused, decreased LOC Metabolic - hypermetabolic, metabolic acidosis, Increased P, Decreased Ca, Increased Mg

In _?_ and _?_, the voice is preserved in these surgeries, but the quality is hoarse and breathy. The patient is at high risk for aspiration following these procedures.

Hemilaryngectomy Supraglottic laryngectomy

In _?_, an artificial membrane used as the semi-permeable membrane and is in contact with patient's blood

Hemodialysis (HD):

_?_ jaundice: Causes by increased breakdown of RBC's

Hemolytic

3 types of Jaundice:

Hemolytic Hepatocellular Obstructive

_?_ : accumulation of blood in pleural space causing compression and collapse of the lung Often associated with _?_

Hemothorax; open pneumothorax

Mode of transportation: Fecal-oral Incubation time: 15-50 days; average 28 days Sources of infection/spread of disease: Crowded areas, poor sanitation, poor hygiene; contaminated food, milk, water, shellfish; persons with subclinical infections; infected food handlers Most infectious 2 weeks prior to onset of symptoms; still infectious for 1-2 more weeks after onset of symptoms Onset is acute - mild flu like symptoms Almost all resolve

Hep A

Drug Therapy for Hepatitis _?_ treated only if severe hepatitis and liver failure Hep C: treat w/ pegylated interferon within 1st 12-24 weeks of infection...decreases development of chronic hep C. Antiemetics & Analgesics - caution if liver secreted Prochlorperazine (Compazine), promethazine (Phenergan), ondansetron (Zofran)

Hep B

Mode of transportation: percutaneous (parenteral)/permucosal exposure to blood or blood products (before 1971); sexual contact; perinatal transmission, tatoos and piercings Incubation time: 45-180 days; average 56-96 days Infectivity: before and after symptoms appear; infectious for 4-6months; carriers continue for person's life 30% asymptomatic More infectious than HIV Gradual onset - symptoms more severe but fewer GI symptoms. Approximately 15-25% of chronically infected persons die from chronic liver disease

Hep B

Mode of transportation: percutaneous (parenteral)/permucosal exposure to blood or blood products; high risk sexual contact; perinatal contact Incubation period: 14-180 days; average 56 days Sources of infection/spread of disease: Blood & blood products; needles and syringes; sexual activity with infected partners Infectivity: 1-2 weeks before symptoms appear; continues during clinical course; 75-85% go on to develop chronic hepatitis 10% due to occupational exposure, hemodialysis, and perinatal transmission. 80% are asymptomatic - Scary!!

Hep C

Mode of transportation: can cause infection only when Hep B is present Incubation time: 2-26 weeks; Sources of infection/spread of disease: Same as HBV Infectivity: blood infectious at all stages Vaccination against HBV reduces risk

Hep D

Mode of transportation: fecal oral route; outbreaks associated with contaminated water supplies in developing countries Incubation period: 15-64 days; average 26-42 days Sources of infection/spread of disease: infected food-borne, contaminated water; poor sanitation; found in Asia, Africa, Mexico; not common in US Infectivity: not known; may be similar to HAV

Hep E

In HIT, platelet aggregation causes neutralization of _?_ requiring higher doses for target therapeutic aPTT levels

Heparin

Treatment for _?_ includes: sustain life and decrease ammonia levels Assess mental and motor status Assess fluid and electrolyte imbalances

Hepatic Encephalopathy

Treatment of _?_ includes: sustain life and decrease ammonia levels Assess mental and motor status Assess fluid and electrolyte imbalances

Hepatic Encephalopathy

The condition of a patient who has cirrhosis of the liver has deteriorated. Which diagnostic study would help determine if the patient has developed liver cancer? Serum α-fetoprotein level Ventilation/perfusion scan Hepatic structure ultrasound Abdominal girth measurement

Hepatic structure ultrasound

Nursing implications for _?_ Comfort measures for pruritus, headache, muscle aches, and pain - caution with liver toxic meds! Nutrition - no special diet needed; might see small frequent meals with high calorie, low fat, vitamin supplements Rest - reduces demands of liver and promotes cell regeneration Avoid alcohol intake and meds cleared by liver Teaching - prevention, bleeding tendencies, risk for injury, meds, and monitoring for relapse

Hepatitis

In _?_ jaundice, hepatocytes are damaged, leak bilirubin. Causes: hepatitis, cirrhosis, hepatocellular carcinoma.

Hepatocellular

_?_ jaundice: caused by the liver's altered ability to take up bilirubin from the blood or to conjugate or excrete it

Hepatocellular

Phosphorous levels in CKD are _?_ (high or low?) 3 S/S

High Tingling around mouth Tingling of fingers parathesia

_?_ regulates the maximal pressure the ventilator can generate to deliver the Vt. The usual setting is _?_ cm H2O above peak inspiratory pressure

High Pressure Limit 10-20

The nurse is caring for a 68-year-old patient admitted with abdominal pain, nausea, and vomiting. The patient has an abdominal mass and a bowel obstruction is suspected. The nurse auscultating the abdomen listens for which of the following types of bowel sounds that is consistent with the patient's clinical picture? Low pitched and rumbling above the area of obstruction High pitched and hyperactive above the area of obstruction Low pitched and hyperactive below the area of obstruction High pitched and hypoactive below the area of obstruction

High pitched and hyperactive above the area of obstruction

_?_- People with this disease have abnormal lymphocytes that are called Reed-Sternberg cells. These cells usually form from B cells;

Hodgkin lymphoma

_?_ is characterized by Reed-Sternberg cells (abnormal giant, multinucleated cells, located in lymph nodes) High cure rate (unless relapse within 12 mo) Occurs most freq 15-35 yrs of age & above 50 yrs of age 2 times as prevalent in men

Hodgkin's

Causes of _?_ are unknown but there seems to be a relationship to an infection with Epstein-Barr virus, genetic components, a relationship to HIV and environmental toxins

Hodgkins

If _?_ is in an advanced stage, treatment is BEACOPP. bleomycin etoposide, doxorubicin (Adriamycin) cyclophosphamide, vincristine,(Oncovin), procarbazine, prednisone

Hodgkins

S/S of _?_ are enlarged lymph nodes, weight loss, fatigue, fever, chills, tachycardia, weakness, night sweats

Hodgkins

Treatment for _?_ is the ABVD regimen: doxorubicin (Adriamycin), bleomycin, vinblastine, dacarbazine

Hodgkins

_?_ are chemo drugs which selectively attach to estrogen receptors, inhibits tumor growth. Two examples are: tamoxifen(Nolvadex), fulvestrant (Faslodex) Estrogens: interfere with hormone receptors and proteins estradiol (Estrace), estrogen (Menest) Aromatase Inhibitors: inhibit aromatase (enzyme that converts adrenal androgen to estrogen) letrozole (Femara)

Hormone Therapy Drugs

Practice: Pt weight 50 kg Has burns to all of both lower extremities (use rule of nines) How much IV fluid does pt need in first 24 hrs? How much will be given in first 8 hrs? How much will be given in next 8 hrs? How many ml/hr will be given each hour for the 1st 8 hrs?

How much IV fluid does pt need in first 24 hrs? 7200 ml How much will be given in first 8 hrs? 3600 ml How much will be given in next 8 hrs? 1800 ml How many ml/hr will be given each hour for the 1st 8 hrs? 450 ml/hr

5 Contraindications of peritoneal dialysis:

Hx of multiple abdominal surgeries or pathologies Abdominal wall or inguinal hernia Excessive obesity Back problems Severe COPD

Treatment of Addison's Disease Replacement therapy:

Hydrocortisone (Solu-cortef) has both glucocorticoids & mineralocorticoids;

S/S of _?_ are decreased LOC, EKG changes, muscle weakness, n/v

Hypercalcemia

_?_ is a metabolic oncologic emergency. It is d/t bone metastasis, multiple myeloma or a parathyroid tumor.

Hypercalcemia

The following CNS problems can lead to _?_. Brain stem injury/infarction Sedative/opioid overdose Spinal cord injury Severe head injury

Hypercapnic respiratory failure

The following are manifestation of _?_. Dysrhythmias Hypertension Tachycardia Bounding pulse

Hypercapnic respiratory failure

The following are manifestations of _?_ Muscle weakness decreased DTR Tremors/Seizures

Hypercapnic respiratory failure

The following are manifestations of _?_. Morning HA Disorientation progressive somnolence Elevated ICP Coma

Hypercapnic respiratory failure

The following chest wall problems can lead to _?_. Thoracic trauma Kyphoscoliosis Pain Severe obesity

Hypercapnic respiratory failure

The following neuromuscular system problems can lead to _?_. Myasthenia gravis Critical illness polyneuropathy Acute myopathy Toxin exposure Amyotrophic Lateral Sclerosis Phrenic nerve injury Guillain-Barre syndrome Poliomyelitis Muscular dystrophy Multiple sclerosis

Hypercapnic respiratory failure

The following respiratory system problems can lead to _?_. Asthma, COPD, cystic fibrosis

Hypercapnic respiratory failure

_?_ occurs when PaCO2 > 45 mm Hg with pH < 7.35

Hypercapnic respiratory failure (aka ventilation failure)

_?_ Head suddenly accelerated and then decelerated.

Hyperextension:

The nurse would recognize which clinical manifestation as suggestive of sepsis? Sudden diuresis unrelated to drug therapy Hyperglycemia in the absence of diabetes Respiratory rate of seven breaths per minute Bradycardia with sudden increase in blood pressure

Hyperglycemia in the absence of diabetes

Which assessment finding is a consequence of the oliguric phase of AKI? Hypovolemia Hyperkalemia Hypernatremia Thrombocytopenia

Hyperkalemia

__?__ is a rare condition that increases the secretion of parathyroid hormone

Hyperparathyroidism

2 types of disorders of Parathyroid Gland:

Hyperparathyroidism Hypoparathyroidism

Manifestations of Autonomic Hyperreflexia: _?_ Blurred vision _?_ Marked diaphoresis above lesion level _?_ Piloerection (erection of body hair) resulting from pilomotor spasm Flushing of skin above lesion Spots in visual field Nasal congestion Anxiety Nausea

Hypertension (up to 300 mm Hg systolic) Throbbing headache—take BP! Bradycardia (30-40 beats/min)

_?_Moves water out of edematous brain cells & into blood vessels Reduces swelling & improves cerebral blood flow Requires frequent monitoring of BP, serum Na levels Just as effective as Mannitol & often used concurrently

Hypertonic Saline (3 or 5%)

_?_ - is an asymmetric left ventricular hypertrophy without dilation; Impaired ventricular filling and diastolic dysfunction from left ventricular stiffness; Decreased ventricular filling and obstruction to outflow leads to decreased C.O.

Hypertrophic Cardiomyopathy

_?_ was the mainstay treatment; Found that it increases risk of focal cerebral ischemia and adversely affects outcome May see very brief periods of _?_ used for refractory intracranial hypertension

Hyperventilation therapy; hyperventilation

Acromegaly Treatment: Surgery:

Hypophysectomy usually using transphenoidal approach.

Complications of Hemodialysis: _?_- occurs from rapid fluid removal, ↓ cardiac output (CO), ↓system intravascular resistance S/S: nausea, light-headedness, sz, vomiting, visual ∆s, chest pain Treatment: ↓volume of fluid being removed and give N.S. iv

Hypotension

Adverse effects of propofol

Hypotension Bradycardia Elevated triglyceride levels

7 classifications of shock:

Hypovolemic shock Cardiogenic shock Distributive Shocks (Maldistribution of Blood Flow) Septic Neurogenic Anaphylactic Obstructive shock

The following are manifestations of _?_ Agitation combative restless disorientation delirium confusion decreased LOC Coma

Hypoxemia respiratory failure

The following are manifestations of _?_ Fatigue inability to speak in complete sentences w/o pausing to take a breath

Hypoxemia respiratory failure

The following are manifestations of _?_. Tachycardia hypertension cool, clammy, diaphoretic skin Dysrhythmias Hypotension

Hypoxemia respiratory failure

The following respiratory system problems can lead to _?_. Acute respiratory distress syndrome, PNA, Toxic inhalation, Hepatopulmonary syndrome, PE, pulmonary artery laceration/hemorrhage, inflammatory state

Hypoxemic respiratory failure

_?_ occurs when PaO2 < 60 mm Hg when patient is receiving inspired O2 of 60% or more

Hypoxemic respiratory failure (aka oxygenation failure)

Post-aneurysm surgery_?_ q shift with urine output hourly

I/O

_?_ is duration of inspiration to duration of expiration. Usual setting is _?_ to _?_ unless IRV is desired

I:E ratio; 1:2 to 1:1.5

For patients who are admitted with a GCS score or 8 or less AND an abnormal CT/MRI, _?_ is used

ICP monitoring

Where will patients be housed who are being treated for aortic dissection

ICU

If a patient has _?_, we do not want them to cough and deep breathe, shiver or be in pain. All of these _?_

IICP; increase ICP

Vasodilators (inhaled) are indicated for patients with Class _?_ or _?_ right sided heart failure. They are administered _?_ times per day using a disk inserted into a nebulizer. They may cause orthostatic hypotension, so hold on patients with Systolic BP of _?_

III or IV; 6-9 < 85

When changing a pleurovac, the chest tube (is/is not) clamped while switching the new in for the old. Is an MD order need for this procedure?

IS; NO

What is the process for administering stem cells to the patient? Administered via _?_ Harvested, frozen, then thawed and administered after chemo eliminated from body (about 24-48 hrs); don't want to harm newly infused cells After given, takes about _?_ wks for cells to produce hematopoietic blood cells In the meantime, the patient has _?_ so protect patient from infection Electrolyte supplements Nutrition Possible blood transfusions

IV (infusion or bolus) 2-4 pancytopenia

Post-hyperparathyroidism surgery: _?_ must be readily available (i.e. in the Pixus)

IV Calcium glucanate

How is Calcium given to treat tetany from hypocalcemia/hypoparathyroidism?

IV Calcium: (give slowly may cause bradycardia esp in patients on digitalis). Extravasation may cause necrosis and sloughing;

Acute IE (infective endocarditis) can also be classified d/t cause. Name 3

IV drug abuse IE = IVDA IE fungal endocarditis site of involvement prosthetic valve endocarditis (PVE)

Lower GI Bleed Treatment: Oxygen (ABCs); monitor V/S 2 large bore IVs infusing _?_ or _?_ Monitor Fluid & Electrolytes Foley catheter Keep HOB elevated

IV fluid LR or .9NS & blood products

What are the initial 4 drug treatments of choice for ACS?

IV nitroglycerin dual antiplatelet (aspirin and clopidogrel) anticoagulation (LMWH)

_?_ is used to reduce anginal pain; it decreases preload and afterload while increasing myocardial oxygen supply. Onset is immediate

IV nitroglycerin (IV NTG)

5 types of medications used in the conservative management of aortic dissection:

IV ß-adrenergic blockers (esmolol, propranolol), IV calcium channel blockers [diltiazem (cardizem), verapamil (Calan)], Angiotensin-converting enzyme inhibitors [enalapril (vasotec)], Sodium nitroprusside, Pain management (opioids)

Besides ruptured blebs, what are 5 other causes of closed pneumothorax?

Iatrogenic (we caused it) - ex: hit lung while inserting central line Blunt trauma Injury from mechanical ventilation (too much PEEP) Perforation of esophagus Injury from broken ribs

If CVP is too low, give _?_ If CVP is too high, give _?_ and _?_ If BP is OK, but CO is low, give _?_

If CVP is too low, give fluids If CVP is too high, give nitro and diuretics If BP is OK, but CO is low, give digoxin

After receiving 2L of normal saline, the CVP for a patient in hypovolemic shock is 14 mm Hg. The nurse will anticipate an order for A. nitroglycerine B. norepinephrine C. sodium nitroprusside D. hydralazine

If in shock, saline leak out - temporary measure B. Normal CVP 15 If map is normal..prob wont give vasopressor

S/S of large pneumothorax (8)

If large: dyspnea, tachypnea, respiratory distress, may have chest pain with or without hemoptysis Absent breath sounds over affected area Hyperresonance with percussion Decreased SaO2

If you can't get blood from a central line, what should you do first?

If you can't get blood from a central line, the tip may be up against the wall of the superior vena cava. Having the patient sit at different angles, turn on different sides, cough, or take a slow deep breath, in each case with his arms held over his head, may move the catheter away from the vessel wall. If these measures fail in making blood return possible, the physician should be notified. The line may be occluded with a clot. Positive pressure should not be applied to the line. Flushing could propel a clot into the heart.

Escape Mechanisms from Immunologic Surveillance includes _?_ - process by which cancer cells evade the immune system

Immunologic escape

A patient has a systemic blood pressure of 120/60 and an ICP of 24 mm Hg. After calculating the patient's cerebral perfusion pressure (CPP), how does the nurse interpret the results? High blood flow to the brain Normal intracranial pressure Impaired blood flow to the brain Adequate autoregulation of blood flow

Impaired blood flow to the brain Normal CPP is 60 to 100 mm Hg. The CPP is calculated with mean arterial pressure (MAP) minus ICP. MAP = SBP + 2 (DBP)/ 3: 120 mm Hg + 2 (60 mm Hg)/3 = 80 mm Hg. MAP - ICP: 80mm Hg - 24 mm Hg = 56 mm Hg CPP. The decreased CPP indicates that there is impaired cerebral blood flow and that autoregulation is impaired. Because the ICP is 24, it is elevated and requires treatment.

What is the prime nursing diagnosis for ARDS?

Impaired gas exchange

What nursing diagnosis will give you the "biggest bang for your buck" with ARDS or acute respiratory failure>

Impaired gas exchange

Manifestation of Acromegaly: endocrine

Impaired glucose tolerance Diabetes

Two days following a colectomy for an abdominal mass, a patient reports gas pains and abdominal distention. The nurse plans care for the patient based on the knowledge that the symptoms occur as a result of which of the following? Irritation of the bowel Nasogastric suctioning Impaired peristalsis Anastomosis site inflammation

Impaired peristalsis

A patient experienced sudden cardiac death (SCD) and survived. What should the nurse expect to be used as preventive treatment for the patient? External pacemaker An electrophysiologic study (EPS) Medications to prevent dysrhythmias Implantable cardioverter-defibrillator (ICD)

Implantable cardioverter-defibrillator (ICD)

A 70-year-old patient in the ICU has become agitated and inattentive since his heart surgery. The nurse knows that this ICU psychosis frequently occurs in individuals with pre-existing dementia, history of alcohol abuse, and severe disease. What interventions should the nurse provide this patient to improve the patient's cognition (select all that apply)? Improve oxygenation. Provide a small amount of beer. Have the family stay with the patient. Enable the patient to sleep on a schedule with dim lights. Decrease sensory overload by conversing away from patient's room.

Improve oxygenation. Enable the patient to sleep on a schedule with dim lights Decrease sensory overload by conversing away from patient's room.

Side effects of _? _include bone marrow suppression, neutropenia, anemia, thrombocytopenia

Imuran

The degree of SCI may be _?_ or _?_.

Incomplete Complete

What laboratory finding fits with a medical diagnosis of cardiogenic shock? Decreased liver enzymes Increased white blood cells Decreased red blood cells, hemoglobin, and hematocrit Increased blood urea nitrogen (BUN) and serum creatinine levels

Increased blood urea nitrogen (BUN) and serum creatinine levels The renal hypoperfusion that accompanies cardiogenic shock results in increased BUN and creatinine levels. Impaired perfusion of the liver results in increased liver enzymes, while white blood cell levels do not typically increase in cardiogenic shock. Red blood cell indices are typically normal because of relative hypovolemia.

Complications with positive pressure ventilation

Increased intra-thoracic pressure can cause compression of thoracic vessels. Decreased CVP Decreased CO Hypotension (We need to compensate to avoid shock)

What are 4 physiological changes in ARDS:

Increased pulmonary capillary permeability Destruction of elastin and collagen Formation of pulmonary microemboli Pulmonary artery vasoconstriction

A patient has renal failure. The nurse, reviewing the lab results, recognizes which finding as indicative of the diminished renal function associated with the diagnosis? Hypokalemia Increased serum urea and serum creatinine Anemia and decreased blood urea nitrogen Increased serum albumin and hyperkalemia

Increased serum urea and serum creatinine

Effects of Corticosteroid Therapy: _?_ effect of norepinephrine acts on renal tubules to reabsorb Na+ (so,.. potassium and hydrogen are excreted) Mood & behavior changes Hypo_?_ Peptic ulcer Skeletal muscle atrophy _?_ intolerance Delayed healing _?_B/P Edema Susceptibility to _?_ Hypo_?_- related to anti-vit D protein depletion, leading to _?_

Increased vasoconstriction effect of norepinephrine acts on renal tubules to reabsorb Na+ (so,.. potassium and hydrogen are excreted) Mood & behavior changes Hypokalemia Peptic ulcer Skeletal muscle atrophy Glucose intolerance Delayed healing ↑B/P Edema Susceptibility to infection Hypocalcemia- related to anti-vit D protein depletion, leading to pathologic fractures

The nurse is caring for a patient who has a duodenal ulcer. The nurse identifies what assessment data as indicative of a gastric perforation? Increasing abdominal distention and tight abdomen. Decreasing hemoglobin and hematocrit with bloody stools. Diarrhea with increased bowel sounds and hypovolemia Decreasing blood pressure with tachycardia and disorientation.

Increasing abdominal distention and tight abdomen

Diffusion, osmosis, and ultrafiltration occur in both hemodialysis and peritoneal dialysis. The nurse should know that ultrafiltration in peritoneal dialysis is achieved by which method? Increasing the pressure gradient Increasing osmolality of the dialysate Decreasing the glucose in the dialysate Decreasing the concentration of the dialysate

Increasing osmolality of the dialysate

When planning care for a patient with cirrhosis, the nurse will give highest priority to which nursing diagnosis? Impaired skin integrity related to edema, ascites, and pruritus Imbalanced nutrition: less than body requirements related to anorexia Excess fluid volume related to portal hypertension and hyperaldosteronism Ineffective breathing pattern related to pressure on diaphragm and reduced lung volume

Ineffective breathing pattern related to pressure on diaphragm and reduced lung volume

_?_ happens when blood turbulence within heart allows causative organism to infect previously damaged valves or other endothelial surfaces.

Infective Endocarditis

_?_ - Infection of the inner layer of the heart that usually affects the cardiac valves 10,000 - 15,000 cases diagnosed in the United States each year; Was almost always fatal until development of penicillin Treatment with antibiotics improved prognosis

Infective Endocarditis (IE)

3 causes of acute aortic valve regurg:

Infective endocarditis Trauma Aortic dissection

PD solutions high in glucose Three phases of PD cycle (or "the exchange") are: _?_- fill, usually 2 liters over 10 minutes _?_- equilibration, diffusion/osmosis, 20-30 minutes up to 8 hours _?_- 15-30 minutes, gentle massage of abdomen glucose is most commonly used agent in PD solutions, but problem → ↑ triglycerides, ↑glucose, membrane dysfunction Amino acid PD solution is available

Inflow Dwell Drain

PD solutions high in glucose Three phases of PD cycle (or "the exchange") are:

Inflow, Dwell, Drain

A patient presents to the ED with severe abdominal pain with rebound tenderness, anorexia, and chills. The vital signs include temperature of 38.7 degrees C, HR 130, RR 34, and BP 82/50. Of the following collaborative interventions, which one should the nurse implement first? Obtain a CT scan of the abdomen with and without contrast. Infuse 1000ml of lactated ringer's solution over 30min. Administer IV ketorolac (toradol) 15mg Give IV ceftriaxone (rocephen) 1gm

Infuse 1000ml of lactated ringer's solution over 30min.

When caring for a patient in acute septic shock, what should the nurse anticipate? Infusing large amounts of IV fluids Administering osmotic and/or loop diuretics Administering IV diphenhydramine (Benadryl) Assisting with insertion of a ventricular assist device (VAD)

Infusing large amounts of IV fluids

You note that Mrs. Thorne (who has a SAH) arrived from the Emergency Department (ED) with a peripheral IV line through which dextrose 5% in water is slowly infusing at 75 mL per hour. What should you do?

Infusion of dextrose 5% in water should be questioned. Infusion of dextrose 5% in water could increase intracranial pressure (ICP).

The nurse assesses the right femoral artery puncture site as soon as the patient arrives after having a stent inserted into a coronary artery. The insertion site is not bleeding or discolored. What should the nurse do next to ensure the femoral artery is intact? Palpate the insertion site for induration. Assess peripheral pulses in the right leg. Inspect the patient's right side and back. Compare the color of the left and right legs.

Inspect the patient's right side and back.

Nursing Assessment #3 -- (flail chest) **Inspect: _______________________ **Gentle Palpation: crepitus from bony fragments **No percussion due to rib pain: **Auscultate:_____________________________________________________

Inspect: Paradoxical breathing Auscultate: Absent/diminished breath sounds; heart tones,

Nursing Assessment #5 -- (cardiac tamponade) **Inspect:____________________________ __________________________ **Palpate: _________________________ **Auscultate: _________________________ ______________________

Inspect: appearance (color, neck veins) Palpate: pulses Ausculate: Heart sounds (distant?)

Nursing Assessment: #2 chest injury: (Open Pneumo) **Inspect:___________________________________ **Palpate:___________________________________ **Percuss: hyperressonance **Auscultate: decreased or absent breath sounds on affected side

Inspect: chest rise and fall; sucking wound? Palpate: subq air

Nursing assessment for injury #1: (Tension Pneumo) Inspect:__________________________________ Palpate: chest pain, poss subcutaneous emphysema Percuss: hyperressonance Auscultate:_______________________________

Inspect: chest rise and fall; symmetry, trachea (is it midline) Auscultate: diminished/absent breath sound on affected side

What does I:E stand for? What is a normal value?

Inspiration:Expiration; 1:2

_?_ is the speed with which the Vt is delivered. Usually setting is _?_ L/min and _?_ seconds

Inspiratory flow rate and flow; 40-80 L/min; 0.8 - 1.2 seconds

Treatment for pleural effusion:

Instill doxycyline or talc during pleurodesis; causes layers to stick together; chest tube clamped for 8 hrs while patient is turned in all positions to coat pleural space

In Mitral valve regurgitation, valve function depends on 4 things:

Intact mitral leaflets Integrity of mitral annulus Chordae tendineae Papillary muscles

2 types of cancer biologic therapy are _?_ and _?_

Interferons, Interleukins

_?_ and _?_ both stimulate T cells, NK cells, B cells and macrophages

Interleukin-2 (IL-2) ϒ interferon

_?_ requires that rate, Vt, inspiratory time, sensitivity and PEEP are set for the patient. In between "Mandatory breaths", patients spontaneously breath at their own rate and Vt. The ventilator synchronizes the mandatory breaths with the spontaneous breaths

Intermittent Mandatory Ventilation (IMV) and Synchronized Intermittent Mandatory Ventilation (SIMV)

The _?_ is the most common vessel used for bypass graft. Its origin is the subclavian artery. It is dissected from the chest wall and anastomosed to the coronary artery distal to the blockage

Internal mammary artery (IMA)

What vessel is the graft of choice for CABG?

Internal mammary artery (IMA)

_?_ occurs when reduced blood flow occurs- causes pain; can affect small or large intestine, or both; if blood flow not restored, intestines can be come necrotic Need surgery to remove necrotic section of intestines, possible colostomy depending of size/area

Intestinal ischemia

_?_ bleeding may occur Difficult to diagnose...no subjective signs (pain, tenderness, guarding) Indications of bleeding: Continued hypotension despite treatment Decreased hemoglobin and hematocrit Expanding girth may also be noted Neurogenic bowel Less voluntary neurogenic control when spinal shock present and for patients with injury level of T12 or below Bowel is areflexic, has decreased sphincter tone

Intraabdominal

2 Complications of Repair of aneurysm:

Intraabdominal hypertension and compartment syndrome

The patient has developed cardiogenic shock after a left anterior descending myocardial infection. Which circulatory-assist device should the nurse expect to use for this patient? Cardiopulmonary bypass Impedance cardiography (ICG) Intraaortic balloon pump (IABP) Central venous pressure (CVP) measurement

Intraaortic balloon pump (IABP) The most commonly used mechanical circulatory-assist device is the intraaortic balloon pump (IABP), and it is used to decrease ventricular workload, increase myocardial perfusion, and augment circulation. Cardiopulmonary bypass provides circulation during open heart surgery. It is not used as an assist device after surgery. ICG is a noninvasive method to obtain cardiac output and assess thoracic fluid status. CVP measurement is an invasive measurement of right ventricular preload and reflects fluid volume problems.

_?_ - Occurs from bleeding within the parenchyma Usually occurs within the frontal and temporal lobes Size and location of hematoma determine patient outcomes

Intracerebral Hematoma

3 Major Complications of Skull Fractures

Intracranial infections Hematomas Meningeal and Brain tissue damage

Tension pneumothorax is a medical emergency! _?_ pressure rises, lung collapses, and _?_ is shifted toward the unaffected side Treat with chest tube or _?_

Intrathoracic ; mediastinum; large bore needle

What should be on hand during an extubation procedure?

Intubation Kit. In-case the patient doesn't tolerate the procedure well, we need to be ready to put another tube back down.

3 causes of mechanical intestinal obstruction

Intussusceptions Volvulus Hernia

Demerol is no longer given for patients with CKD because _?_

It is not excreted appropriately with patients with renal issues

_?_ is the yellowish discoloration of body tissues d/t alteration in normal bilirubin metabolism or flow of bile into hepatic or biliary duct system. Jaundice is _?_, not a _?_ Results if bilirubin concentration in blood to high, has to be _?_ times normal for jaundice to occur.

Jaundice symptom, not a disease. 3

_?_ - best treatment for ESRD

Kidney transplant

You care for many patients with traumatic brain injury. Assessment for neurologic deterioration is critical. The best indicator of deterioration in a patient with traumatic brain injury is change in:

LOC

3 Autonomic manifestations of GBS

Labile blood pressures Cardiac dysrhythmias Tachycardia/bradycardia

_?_ involves actual tearing of the brain tissue Often occurs with depressed and open fractures and penetrating injuries _?_ can occur. Surgical repair is impossible due to nature of the tissue Poor prognosis with large injury

Laceration Intracerebral hemorrhage

Secondary Adrenocortical insufficiency is caused by a lack of _?_ but not necessarily a lack of __?_

Lack of pituitary ACTH secretion- may not lack mineralcorticosteroids

Surviving Sepsis Campaign: _?_ can help guide resuscitation because it is a marker for tissue hypoperfusion

Lactate

What is the fluid given during fluid resuscitation post-burn?

Lactated ringers

_?_ is titrated on the # of stools (usually desire 4 per day). It is used to eliminate _?_

Lactulose Ammonia

3 drug therapies for Hepatic Encephalopathy

Lactulose Neomycin; rifaximin

The following are signs of _?_ gradual onset Rare episodes of vomiting Low grade, crampy pain Absolute constipation or diarrhea around impaction Greatly increased abd size

Large Bowel Obstruction

The nurse is caring for a 29-year-old man who was admitted a week ago with multiple rib fractures, a pulmonary contusion, and a left femur fracture from a motor vehicle crash. After the attending physician tells the family that the patient has developed sepsis, the family members have many questions. Which information should the nurse include in explaining the early stage of sepsis? Antibiotics are not useful once an infection has progressed to sepsis. Weaning the patient away from the ventilator is the top priority in sepsis. Large amounts of IV fluid are required in sepsis to fill dilated blood vessels. The patient has recovered from sepsis if he has warm skin and ruddy cheeks.

Large amounts of IV fluid are required in sepsis to fill dilated blood vessels.

Rhabdomyolysis: Collaborative Care _?_helps maintain urine production prevents kidney failure; Monitor urine output/intake; _?_ - not always needed; _?_to relieve compartment syndrome if present; Usually IMC or ICU for close monitoring; If rhabdo d/t med, stop the med; If d/t medical condition, treat medical condition

Large amounts of IV fluids Dialysis fasciotomy

A patient is recovering in the intensive care unit (ICU) after receiving a kidney transplant approximately 24 hours ago. What is an expected assessment finding for this patient during this early stage of recovery? Hypokalemia Hyponatremia Large urine output Leukocytosis with cloudy urine output

Large urine output

What are 4 endotracheal tube complications associated with ARDS?

Laryngeal ulceration tracheomalacia tracheal stenosis tracheal ulceration

_?_of cancer - from 1 to 40 years; time elapsed between genetic alteration and evidence of cancer

Latent period

Jim has a basilar skull fracture. List 3 observations that are relevant to this diagnosis

Leakage of cerebrospinal fluid (CSF) from the nose or ear Raccoon sign Battle's sign

In Mitral valve regurgitation, valve patency depends on 2 things:

Left atrium Left ventricle Abnormality of any of these structures can result in regurgitation.

The nurse is caring for a 55-year-old man patient with acute pancreatitis resulting from gallstones. Which clinical manifestation would the nurse expect the patient to exhibit? Hematochezia Left upper abdominal pain Ascites and peripheral edema Temperature over 102o F (38.9o C)

Left upper abdominal pain

The nurse is caring for a patient admitted with suspected hyperparathyroidism. Because of the potential effects of this disease on electrolyte balance, the nurse should assess this patient for what manifestation? Neurologic irritability Declining urine output Lethargy and weakness Hyperactive bowel sounds

Lethargy and weakness

_?_ are cancers that begin in the blood-forming tissue of the bone marrow. Don't form solid tumors. Instead, large numbers of abnormal white blood cells build up in the blood and bone marrow, crowding out normal blood cells. The low level of normal blood cells can make it harder for the body to get oxygen to its tissues, control bleeding, or fight infections. There are four common types, which are grouped based on how quickly the disease gets worse (acute or chronic) and on the type of blood cell the cancer starts in (lymphoblastic or myeloid).

Leukemia

Magic Mouthwash contains _?_ and _?_. It is used to numb the throat.

Lidocaine and mylanta

How is SPAH treated if irreversible damage has already occurred?

Like IPAH (drugs, surgery, transplant)

The patient with systemic lupus erythematosus had been diagnosed with syndrome of inappropriate antidiuretic hormone (SIADH). What should the nurse expect to include in this patient's plan of care (select all that apply)? Obtain weekly weights. Limit fluids to 1000 mL per day. Monitor for signs of hypernatremia. Minimize turning and range of motion. Keep the head of the bed at 10 degrees or less elevation.

Limit fluids to 1000 mL per day. Keep the head of the bed at 10 degrees or less elevation.

_?_ cancer: indurated, painless ulcer _?_ cancer: ulcer or area of thickening; soreness/pain; increased saliva, slurred speech, dysphagia, toothache, earache (late signs) _?_ cancer: leukoplakia; erythroplakia; ulceration; sore spot; rough area; pain, dysphagia, difficulty chewing/speaking (late signs)

Lip cancer; Tongue oral cavity

A nurse is collecting data from a patient admitted with hepatitis A. Which information given by the patient may indicate the patient's susceptibility to contract hepatitis A? Select all that apply. Living in slums Working as local plumber Working in a chemical factory Working as a sewage cleaner Working as a waiter and dishwasher

Living in slums Working as local plumber Working as a sewage cleaner

_?_ - Inhalation injury to the trachea, bronchioles, and alveoli usually caused by inhalation of toxic chemicals or smoke.

Lower airway injury

Name 4 infections that can cause SIADH:

Lung, meningitis, encephalitis, HIV;

_?_: cancer that begins in lymphocytes (T cells or B cells). These are disease-fighting white blood cells that are part of the immune system;, abnormal lymphocytes build up in lymph nodes and lymph vessels, as well as in other organs of the body

Lymphoma

In _?_, lymphocytes undergo malignant changes and produce tumors in lymphoid tissue. Two types are _?_ and _?_

Lymphomas Hodgkins Non-Hodgkins

Normal autoregulation maintains CBF and CPP by adjusting cerebral blood vessel diameters and metabolic factors that impact ICP. It is critical that _?_ be maintained when ICP is elevated

MAP

How do you calculate Cerebral perfusion pressure?

MAP - ICP

Majority of cases of mitral valve regurg have been attributed to: _?_ _?_ _?_ Ischemic papillary muscle dysfunction Infective _?_

MI Chronic rheumatic heart disease Mitral valve prolapse endocarditis

Clinical Manifestations of _?_ may include: CV dysfunction (e.g. myocardial depression & vasodilation) Lung dysfunction (e.g. ARDS) GI dysfunction (e.g. decreased GI motility, abdominal distention, paralytic ileus, ischemia) Liver dysfunction (e.g. decreased protein synthesis)

MODS

Clinical Manifestations of _?_ may include: Liver dysfunction (e.g. decreased protein synthesis) CNS dysfunction (e.g. mental status changes) Renal dysfunction (e.g. acute renal failure) Hypermetabolism Failure of the coagulation system

MODS

_?_: the failure of two or more organ systems in an acutely ill patient such that homeostasis cannot be maintained without intervention Can result from uncontrolled septic shock so important to recognize SIRS/Sepsis early.

MODS

_?_ is used to assess for soft tissue and neurologic changes

MRI

SCI (spinal cord injury) are caused by _?_, _?_,_?_ and sports injuries

MVA, Violence Falls

_?_- secrete tumor necrosis factor (TNF) which causes necrosis of tumor cells, interleukin-1 (IL-1) which stimulates T lymphocyte activation and production, and colony-stimulating factors (CSF) which regulate production of blood cells in bone marrow

Macrophages

The nurse is conducting discharge teaching for a patient with metastatic lung cancer who was admitted with a bowel impaction. Which of the following instructions would be most helpful to prevent further episodes of constipation? Eat several small meals per day to maintain bowel motility. Reduce intake of medications causing constipation. Sit upright during meals to increase bowel motility by gravity. Maintain a high intake of fluid and fiber in the diet.

Maintain a high intake of fluid and fiber in the diet.

A patient has undergone cholecystectomy. What postoperative care should the nurse perform for this patient? Select all that apply. Maintain a low-fat diet. Monitor for any bleeding. Instruct not to do deep breathing. Place patient in shock position. Place the patient in Sims' position.

Maintain a low-fat diet. Monitor for any bleeding. Place the patient in Sims' position.

A patient arrives at the emergency department following a head injury. The nurse's first priority for this patient is: Monitoring for changes in neurologic status Maintaining cervical spine precautions Establishing IV access with a large bore catheter Determining the presence of increased ICP Maintaining a patent airway

Maintaining a patent airway

_?_ is a rare disorder Functional defect is hypermetabolism of skeletal muscle resulting from altered control of intracellular calcium. This leads to muscle contracture, hyperthermia, hypoxemia, lactic acidosis, and hemodynamic and cardiac alterations.

Malignant hyperthermia

A male patient suffered a diffuse axonal injury from a traumatic brain injury (TBI). He has been maintained on IV fluids for 2 days. The nurse seeks enteral feeding for this patient based on what rationale? Free water should be avoided. Sodium restrictions can be managed. Dehydration can be better avoided with feedings. Malnutrition promotes continued cerebral edema.

Malnutrition promotes continued cerebral edema.

Nursing implications for _?_ give through filter watch temp -- too cold = crystallization measure serum osmolaltiy watch f/e balance

Mannitol

A nurse is attending to a patient with jaundice. The health care provider instructs the nurse to prepare the patient for percutaneous liver biopsy. The nurse understands that presence of certain conditions may need the procedure to be rescheduled. Which conditions may require percutaneous liver biopsy to be rescheduled? Select all that apply. Chronic hepatitis Marked ascites Hepatic cirrhosis Low hemoglobin levels Bleeding disorder

Marked ascites Low hemoglobin levels Bleeding disorder

Once inserted, what is important for the initial nurse to establish concerning the ET tube?

Measurement of the tube (on the tube itself) at a specific point on the patient. Ex: 22 cm at the patient's lip. If this changes, we know the tube has moved.

_?_intestinal obstruction: May result from a detectable occlusion of the lumen of the intestinal tract

Mechanical

Treatment for obstructive shock includes: _?_: pericardial tap/drain for cardiac tamponade; needle decompression or chest tube for tension pneumothorax or hemopneumothorax Radiation or removal of mass Decompressive laparotomy

Mechanical decompression

2 types of intestinal obstructions

Mechanical: Non-Mechanical:

Spinal cord injury is Classified by: (3)

Mechanism of injury Skeletal and neurologic level of injury Completeness or degree of injury

Patho of SIRS: _?_ excess: cytokines (e.g. tumor necrosis factor, interleukins), oxygen free radicals, etc. Widespread _?_injury and dysfunction along with hypermetabolism _?_ (excessive) and increased capillary permeability Tissue edema _?_ entrapment in microcirculation

Mediator endothelial Vasodilation Neutrophil

Collaborative Intervention for #1 injury: (Tension Pneumo) ** ______________ Emergency

Medical emergency! Need needle decompression and/or chest tube

_?_ cancer that begins in cells that become melanocytes, which are specialized cells that make melanin (the pigment that gives skin its color). Most melanomas form on the skin, but melanomas can also form in other pigmented tissues, such as the eye.

Melanoma:

S/S of GI bleeds: _?_ indicates a slow bleed from upper GI source _?_is bloody vomitus appearing as fresh, bright red blood or coffee ground material in NG aspirate _?_is small amounts of blood in gastric secretions/vomitus/stools not apparent by appearance; detected by hemocult/guaiac

Melena (black, tarry stools) Hematemesis: Occult bleeding:

What population has the greatest risk of spinal cord injury (SCI)?

Men between 16-30

_?_ - Oxygen delivery or consumption is impaired, with the resulting hypoxia, and death; These may result in death even with the absence of any burn to the skin Note the cherry red coloring in photo, indicative of CO poisoning

Metabolic asphyxiation

After an abdominal hysterectomy, a 45-year-old woman complains of severe gas pains. Her abdomen is distended. It is most appropriate for the nurse to administer which prescribed medication? Morphine sulfate Incorrect Ondansetron (Zofran) Acetaminophen (Tylenol) Metoclopramide (Reglan)

Metoclopramide (Reglan)

Mayo Clinic: Investigators want to determine if early administration of oral _?_in participants diagnosed with sepsis will impact blood pressures and decrease the need for and/or doses of intravenous pressor agents.

Midodrine (pro-amatine)

10 clinical manifestations of ARDS

Mild at first (ABG indicates mild hypoxemia & respiratory alkalosis) Dyspnea &Tachypnea Intercostal and suprasternal retractions Tachycardia Changes in mentation Increase in crackles Pulmonary capillary wedge pressure < 18mmHg X-Ray shows whiteout As hypoventilation develops respiratory acidosis occurs Refractory hypoxemia, despite increased FIO2

A patient with type 2 diabetes and cirrhosis asks the nurse if it would be okay to take silymarin (milk thistle) to help minimize liver damage. The nurse responds based on what knowledge? Milk thistle may affect liver enzymes and thus alter drug metabolism. Milk thistle is generally safe in recommended doses for up to 10 years. There is unclear scientific evidence for the use of milk thistle in treating cirrhosis. Milk thistle may elevate the serum glucose levels and is thus contraindicated in diabetes.

Milk thistle may affect liver enzymes and thus alter drug metabolism.

During _?_, the heart is slowed with BB or stopped temporarily with adenosine; IMA is sutured in.

Minimally invasive direct coronary artery bypass (MIDCAB)

_?_ offers patients with limited disease an approach to surgical treatment that does not involve a sternotomy and CPB

Minimally invasive direct coronary artery bypass (MIDCAB)

_?_ is a 4 lumen tube used for the tamponade of gastro-oesophageal bleeding that is unresponsive to medical and endoscopic treatment

Minnesota Tube

5 Factors determining response to chemo:

Mitotic rate of tissue where tumor arises Size of tumor Age of tumor Location of tumor Presence of resistant tumor cells

Clinical Manifestations of Mitral Valve Prolapse: Usually benign, but serious complications can occur. Name 5

Mitral valve regurgitation Infective endocarditis Sudden death Heart failure Cerebral ischemia

_?_ Neck Dissection - An alternative to radical neck dissection. This procedure limits disfigurement and functional loss (this procedure is modified to spare as many structures as possible). _?_Neck Dissection - Only removes lymph nodes within the area of involvement.

Modified Simple/Selective

Post-aneurysm surgery Interventions -

Monitor BUN/Cr daily; Monitor resp status and listen breath sounds q 4 hrs and prn; TCDB q2hrs and prn; encourage splinting of incision; IS q1hr while awake; Ambulate; NG tube to low intermittent suction till return of bowel sounds; notify MD when bowels sounds return; Assess pain, admin pain meds as ordered; Assess incision site for s/s of bleeding/infection; d/c teaching including pain management, wound care, activity restrictions; No driving care till follow up visit with MD

What nursing intervention should be implemented in the care of a patient who is experiencing increased ICP? Monitor fluid and electrolyte status carefully. Position the patient in a high Fowler's position. Administer vasoconstrictors to maintain cerebral perfusion. Maintain physical restraints to prevent episodes of agitation.

Monitor fluid and electrolyte status carefully.

A patient with ascites is admitted to the hospital. What should be the primary nursing actions in this case? Select all that apply. Monitor fluid and electrolytes. Provide a high-sodium diet. Encourage high-fluid intake. Administer an albumin infusion. Anticipate paracentesis

Monitor fluid and electrolytes. Administer an albumin infusion. Anticipate paracentesis

The patient has a form of glomerular inflammation that is progressing rapidly. She is gaining weight, and the urine output is steadily declining. What is the priority nursing intervention? Monitor the patient's cardiac status. Teach the patient about hand washing. Obtain a serum specimen for electrolytes. Increase direct observation of the patient.

Monitor the patient's cardiac status.

A patient with peptic ulcer disease (PUD) complains he has been nauseated for most of the day and is now feeling lightheaded and dizzy. Based on these complaints, which nursing actions would be most appropriate for the nurse to take? Select all that apply. Administer an antacid hourly until the nausea subsides. Monitor the patients vital signs. Notify the MD of the patient's symptoms. Initiate oxygen therapy. Reassess the patient in 1 hour.

Monitor the patients vital signs. Notify the MD of the patient's symptoms.

Nursing Assessment for AA -

Monitor vital signs; Assess risk factors for arterial disease; Question pt about: chest/back/abdominal pain, pulsations in abdomen; Inspect skin for presence of vascular disease or breakdown; Check peripheral circulation - pulses, temp, color; Observe for signs/symptoms of rupture; Note tenderness in abdomen; Monitor for abdominal distention

Side effects of _?_ include: fever, chills, n/v/d, anaphylaxis, capillary leak syndrome, hepatotoxicity, bone marrow depression, CNS effect and cardiotoxic.

Monoclonal antibodies

5 Drug Therapies for Acute Pancreatitis

Morphine Antispasmodics - [dicyclomine (bentyl)] Carbonic anhydrase inhibitors [acetazolamide (Diamox)] Antacids PPI

Common means of sedation for patient receiving mechanical ventilation

Morphine Ativan propofol ("milk of amnesia")

_?_ is the drug of choice for chest pain that is unrelieved by NTG. it decreases cardiac workload by lowering oxygen consumptoion, reducing contractility and decreasing BP and HR.

Morphine sulfate

Assessment Findings for Ascending aorta and aortic arch aneurysms:

Most do not have any symptoms, Angina d/t decreased blood flow to the coronary arteries , Hoarseness d/t pressure on the laryngeal nerve, Dysphagia d/t pressure on esophagus

4 types of manifestations with Guillain Barre Syndrome:

Motor manifestations Sensory manifestations Cranial nerve manifestations Autonomic manifestations

6 s/s of cardiac tamponade

Muffled, distant heart sounds Hypotension Neck vein distention Increased CVP pulsus paradoxus decreased urine output

Causes of _?_ may be d/t exposure to radiation, organic chemicals (benzene); metals; herbicides; insecticides; genetic factor; viral infection

Multiple Myeloma

_?_ cancer that begins in plasma cells (type of immune cell). The abnormal plasma cells, called myeloma cells, build up in the bone marrow and form tumors in bones all through the body. It is also called plasma cell myeloma and Kahler disease.

Multiple Myeloma:

In _?_, myeloma cells collect in bone marrow - crowd out normal blood cells

Multiple myeloma

Clinical manifestations of Mitral Valve Prolapse: Most patients asymptomatic for life _?_ from insufficiency that gets more intense through systole Late or _?_murmur _?_ mid to late systole that may be constant or may vary from beat to beat

Murmur; holosystolic Clicks

Complications of Hemodialysis: _?_ - due to hypotension, hypovolemia, high ultrafiltration rate, and low Na dialysate solution

Muscle cramps

The nurse is caring for a 36-year-old woman with possible hypoparathyroidism after a thyroidectomy. It is most appropriate for the nurse to assess for which clinical manifestations? Polyuria, polydipsia, and weight loss Cardiac dysrhythmias and hypertension Muscle spasms and hyperactive deep tendon reflexes Hyperpigmentation, skin ulcers, and peripheral edema

Muscle spasms and hyperactive deep tendon reflexes

What are 4 neuromuscular abnormalities that may lead to a decrease in ventilatory supply?

Muscular dystrophy Guillain-Barré syndrome Multiple sclerosis Phrenic nerve injury

Small Bowel Obstruction Clinical signs and symptoms vary (depends on location of obstruction): May include (4):

N/V, abdominal pain, distention, inability to pass flatus and/or stool

Is a lumbar puncture part of the treatment for IICP?

NO; it could allow the brain to herniate into the brainstem

Collaborative Care for Gallbladder disease: IV fluids _?_, later progress to low-fat diet Antiemetics Analgesics (sphincter of Oddi spasms) _?_ vitamins Anticholinergics and antispasmodics Antibiotics (for secondary infections)

NPO with NG tube Fat-soluable

Treatment for HIT: _?_ flush for IVs If clotting severe, _?_ or _?_ to counteract circulating Heparin, thrombolytic agents/surgery for clots No _?_transfusion: could make it worse NO _?_ or _?_....never, never, never, never again

NS plasmapheresis, protamine platelet heparin or LMWH

Severe hypoxia causes a shift from aerobic to anaerobic respiration. We may need to give _?_ to increase pH and avoid metabolic acidosis which would cause a drop in _?_, _?_ and cell death

NaHCO3 Decreased CO Decreased BP

When discussing chemotherapy side effects often you will hear the word _?_ mainly in reference to the blood counts, particularly white blood cell count and platelet count. This term means low point. When chemotherapy is given it not only affects the rapidly dividing cancer cells but it also affects some of the normal cells of the body. These effects particularly occur on normal cells that divide rapidly such as, the hair, the lining of the mouth, the cells lining the intestinal tract and the blood cells (white and red blood cells as well as platelets).

Nadir

_?_ Affects Blood Cell and Platelet Counts; it is the "low point" of chemotherapy

Nadir

_?_ - directly lyse tumor cells without prior sensitization; stimulated by IL-2, ϒ interferon

Natural Killer (NK) cells

7 GI s/e of chemo

Nausea and Vomiting Diarrhea Anorexia Stomatitis, Pharyngitis/Esophagitis

Care after _?_ includes: Parenteral fluids for the first 24-48 hours Tube feedings started next Calorie counts Nutrition is vital because calories and protein are needed for tissue repair. Daily weights Maintain intake and output Monitor labs as ordered Speech therapy to evaluate swallowing

Neck disection

_?_ involves the use of chambers that encase the chest or body and surround it with intermittent subatmospheric pressure

Negative pressure ventilation

What type of diabetes insipidus can be caused by medications? Which medications? Explain how this affects ADH

Nephrogenic; Lithium and dilantin (can cause hypokalemia and hypercalcemia leading to nephrogenic DI), renal tubules no long respond to ADH

_?_: form from cells that release hormones into the blood in response to a signal from the nervous system. These tumors, which may make higher-than-normal amounts of hormones, can cause many different symptoms; may be benign or malignant.

Neuroendocrine Tumors

Treatment for _?_ shock: Patent airway and oxygen Prevent spinal cord trauma with stabilization Cautious administration of fluid (crystalloids) Treat hypotension w/ Vasopressors (e.g. phenylephrine (neosynephrine) Atropine for bradycardia Prevent hypothermia: monitor temperature

Neurogenic

_?_ Shock causes include: SCI at or above T5 Spinal anesthesia Vasomotor center depression

Neurogenic

_?_ is due to the loss of vasomotor tone caused by injury. It is characterized by hyptotension and bradycardia.

Neurogenic

What type of DI is the most common? Explain how ADH insufficiency occurs:

Neurogenic DI; disruption of the hypothalamus or pituitary by surgery or IICP

Aortic stenosis has a poor prognosis when experiencing symptoms and valve obstruction is not relieved _?_is contraindicated because it reduces preload.

Nitroglycerin

_?_ are like alkylating agents; they break DNA helix, interfere with DNA replication; cross blood-brain barrier. Example: carmustine (Gliadel)

Nitroureas

A histamine (H2) receptor antagonist will be prescribed for a patient. The nurse identifies which of the following meds as being H2 receptor antagonists? Select all that apply. Nizatidine (Axid) Ranitidine (Zantac) Famatidine (Pepcid) Cimetidine (Tagamet) Esomeprazole (Nexium) Lansoprazole (Prevacid)

Nizatidine (Axid) Ranitidine (Zantac) Famatidine (Pepcid) Cimetidine (Tagamet)

Are these patients equal? Patient A: 14% Hgb, SaO2 100% Patient B: 8% Hbg, SaO2 100%

No

Most common cause of closed pneumothorax

No associated external wound Most common form is spontaneous caused by rupture of blebs on visceral pleura (underweight males who smoke) Ruptured blebs or bullae in patients with COPD

If PaO2/FiO2 is < 200, should you automatically assume ARDS? Why or why not?

No, must also look at the amount of FiO2 being given. If it is greater than 60% AND ratio is less than 200, then assume ARDS

_?_ -large group of cancers that start in lymphocytes. The cancers can grow quickly or slowly and can form from B cells or T cells.

Non-Hodgkin lymphoma

_?_ is a lymphoma which affects all ages Treatment: chemo & sometimes radiation More aggressive lymphomas: better response to treatment HSCT - some benefit in certain subtypes w/ aggressive or refractory lymphomas Monoclonal Antibodies will cause lysis/cell death

Non-Hodgkin's

_?_intestinal obstruction: May result from a neuromuscular or vascular disorder

Non-Mechanical

In the non-ventilated patient, water level in the water seal chamber rises with _?_ and drops with _?_. In the ventilated patient, water level drops with __?_ and rises with _?_.

Non-vented: rises with inspiration and drops with expiration Ventilated: rises with expiration, drops with inspiration

What is normal ICP? When should we begin to worry?

Normal 5-15 mm Hg worry if > 20

Which nursing intervention best protects Patients with neutropenia? Allowing fresh plants and flowers in the patient's room Not allowing raw fruit to be served to patient Encouraging a minimum of 150ml of water intake daily Dusting patient's room daily

Not allowing raw fruit to be served to patient

Jim's intracranial pressure (ICP) has dramatically increased. In addition, he has systolic hypertension, bradycardia, a deteriorating level of consciousness, and poor pupil responses. These signs and symptoms reflect a severely elevated ICP with loss of compensation and autoregulation. _?_ is your first action.

Notify MD

Treatment for flail chest. Give _?_, stabilize hand with chest (temporarily), give _?_ meds, and position patient to allow best oxygenation. Intubation, sx for internal stabilization, _?_ .

O2 pain chest tube

What other intervention is almost always administered with mechanical ventilation?

OG or NG tube to intermittent suction

_?_ jaundice: caused by damage from liver tumors, hepatitis, or cirrhosis, common bile duct obstructions from stones, strictures, sclerosing cholangitis, pancreatic cancer.

Obstructive

Chest tube care: _?_ dressing at insertion site _?_ to check CT placement and re-expansion of lung Assess resp status and lung sounds q4hrs and prn Monitor for s/s of pneumo or hemothorax expansion Drainage system below level of chest, no kinks in tube, no dependent tube loops, no obstructions

Occlusive sterile; Chest x-ray

_?_ procedure uses full or partial sternotomy to access all coronary vessels. It is performed on a beating heart using mechanical stabilizers

Off-Pump Coronary Artery Bypass (OPCAB)

_?_ - found on inside and outside of cancer and fetal cells; cancer cells revert and regain embryonic capability to differentiate

Oncofetal antigens

_?_- mutated protooncogenes; cause cells to change in appearance and function

Oncogenes

The patient receiving chemotherapy rings the call bell and reports the onset of nausea. The nurse should prepare an as-needed dose of which medication? Morphine sulfate Zolpidem (Ambien) Ondansetron (Zofran) Dexamethasone (Decadron)

Ondansetron (Zofran)

The patient with a gastric ulcer has an order for sucralfate (carafate) 1 gm po QID. The nurse would schedule the medication for which of the following times? With meals and at bedtime One hour before meals and at bedtime Every 6 hours around the clock One hour after meals and at bedtime

One hour before meals and at bedtime

CKD characteristics: onset _?_ (gradual or sudden?) #1 cause _?_ diagnosis GFR < _?_ for _?_ reversible _?_ (yes or no?) mortality _?_ primary cause of death_?_

Onset: gradual (years) Cause: diabetic nephropathy Dx: GFR < 60 x 3 months &/or kidney damage > 3 months Reversible: NO Mortality: 19-24% Primary cause of death: Cardiovascular disease

AKI characteristics: onset _?_ cause _?_ diagnosis _?_ reversible? _?_ mortality _?_ primary cause of death_?_

Onset: sudden Cause: acute tubular necrosis (ATN) Dx: UOP ↓ and Cr ↑ Reversible: maybe Mortality: up to 60% Primary cause of death: infection

_?_- For AAA repair; Large abdominal incision; Surgical resection or excision of aneurysm, area replaced with graft sewn end to end

Open Aneurysm Repair (OAR)

4 causes of tension pneumothorax

Open or closed pneumothorax Mechanical ventilation Resuscitation efforts Blocked, clotted or clamped chest tubes

Types of injuries from penetrating trauma (5)

Open pneumothorax Hemopneumothorax Cardiac tamponade Esophageal, tracheal, great vessel tear Tension pneumothorax

_?_ - sounds bad, but opening provides vent to decrease trapped air _?_ (can kill, - most dangerous when receiving positive pressure ventilation)

Open pneumothorax (sucking chest wound); Tension pneumothorax

Collaborative care of pneumo & hemothorax (7)

Oxygen Ensure patent airway Continuous SaO2 ABG's Monitor V/S Maintain adequate H&H Maintain chest tube system

Treatment for ARDS: _?_ administration Mechanical ventilation with _?_ _?_ Positioning/Lateral Rotation Maintenance of _?_and tissue perfusion (inotropic drugs such as dopamine or dobutamine) Maintenance of _?_& fluid balance Oxygen carrying capacity (PRBC administered to increase O2 carrying capacity) sedation Increase _?_ time I:E diuretics pain management neuromuscular blockade

Oxygen administration Mechanical ventilation with PEEP Prone Positioning/Lateral Rotation Maintenance of cardiac output and tissue perfusion (inotropic drugs such as dopamine or dobutamine) Maintenance of nutrition & fluid balance Oxygen carrying capacity (PRBC administered to increase O2 carrying capacity) sedation increased expiratory time (increase from 1:2 to 1:3 or 1:4) diuretics pain managment neuromuscular blockade

What defines acute lung injury?

P/F (PaO2 to FiO2) ratio of 200-300

What are 4 s/s of ARDS?

P/F < 200, severe dyspnea, refractory hypoxemia, infiltrates

_?_ creates positive pressure at end exhalation and restores functional residual capacity (FRC).

PEEP

_?_ is positive pressure applied at the end of a ventilator breath. The usual setting is _?_ cm H20

PEEP - positive end-expiratory pressure; 5 cm H2O

A normal physical assessment of the cardiovascular system includes: Palpation: _?_ palpable in 4th ICS at left MCL; no _?_ or _?_; Slight palpable pulsations of the _?_ in the epigastric area. Carotid and extremity pulses _?_ and equally bilaterally. No _?_

PMI thrills or heaves abdominal aorta 2+ pedal edema

Diet with CKD: PO4 (high or low?) K+ (high or low?) Na (high or low?) Protein (high or low?) Maybe _?_ restriction

PO4 - low K+ - low Na - low Protein - low Maybe fluid restriction

_?_ is a balloon tipped cath that can be inflated in a stenosed area to increase blood flow.

PTCA Percutaneous Transluminal Coronary Angioplasty

Hypoparathyroidism is an uncommon condition associated with inadequate circulating _?_

PTH;

Name 3 hemodynamic measurements that relate to afterload

PVR SVR MAP

FiO2 is usually adjusted to maintain PaO2 level > _?_ or SpO2 level > _?_

PaO2 > 60 SpO2 > 90

The nurse is caring for a 72-year-old man in cardiogenic shock after an acute myocardial infarction. Which clinical manifestations would be of most concern to the nurse? Restlessness, heart rate of 124 beats/minute, and hypoactive bowel sounds Mean arterial pressure of 54 mm Hg, increased jaundice, and cold, clammy skin PaO2 of 38 mm Hg, serum lactate level of 46.5 mcg/dL, and bleeding from puncture sites Agitation, respiratory rate of 32 breaths/minute, and serum creatinine level of 2.6 mg/dL

PaO2 of 38 mm Hg, serum lactate level of 46.5 mcg/dL, and bleeding from puncture sites Severe hypoxemia, lactic acidosis, and bleeding are clinical manifestations of the irreversible state of shock. Recovery from this stage is not likely because of multiple organ system failure. Restlessness, tachycardia, and hypoactive bowel sounds are clinical manifestations that occur during the compensatory stage of shock. Decreased mean arterial pressure, jaundice, cold/ clammy skin, agitation, tachypnea, and increased serum creatinine are clinical manifestations of the progressive stage of shock.

_?_ is used to assess the degree of ARDS impairment

PaO2 to FiO2 ratio

Besides the "CAUTION" warning signs of cancer, what are the 4 other signs?

Pain Fatigue Weight loss Risk factors

Name 8 clinical manifestations of pancreatitis:

Pain Nausea/Vomiting Fever Jaundice Bowel sounds decreased or absent Abdomen tenderness & distention Skin discoloration to umbilical area (Cullen's Sign) & flank area (Grey Turner's Spots) Shock

Assessment Findings for Thoracic aneurysm:

Pain extending to neck, shoulders, lower back, or abdomen, Syncope, Dyspnea, Increased pulse, Cyanosis. Weakness, Hoarseness, difficulty swallowing d/t pressure from the aneurysm

Integumentary manifestations of DIC include 4 things:

Pallor Petechiae Purpura Oozing (needle stick sites)

Nursing Assessment #4-- (Hemothorax) **Inspect: resp. distress, dyspnea, collapsed neck veins due to volume, possible mediastinal shift **Palpate:________________________ **Percuss:_______________________ **Auscultate:________________________

Palpate: swelling and rib movement Percuss: dull Auscultate: decreased heart and lung sounds

Initial symptoms of mitral valve regurg include Weakness Fatigue _?_ _?_ that gradually progresses to _?_, paroxysmal nocturnal dyspnea, and peripheral edema Auscultatory findings of accentuated left ventricular filling leading to audible _?_ Murmur is loud _?_ or _?_murmur located at apex radiating to left axilla.

Palpitations Dyspnea; orthopnea S3 pansystolic or holosystolic

2 drug therapies for Chronic Pancreatitis

Pancreatic enzyme products [pancrelipase (Pancrease, Zenpep, Creaon, Viokase)] Insulin

_?_ indicates that recipient's sensitivity to various HLAs before receiving a transplant. The potential recipient may have been exposed to HLA antigens by means of previous blood transfusions, pregnancy, or a previous organ transplant.

Panel of Reactive Antibodies - (PRA)

Treatment of hyperparathyroidism: Surgical therapy:

Parathyroidectomy

What is the most effective treatment of primary and secondary hyperparathyroidism?

Parathyroidectomy

2 Sensory manifestations of GBS

Paresthesias unpleasant sensations of burning, stinging, and prickley feeling Pain (cramping)

The location of the skull fracture determines the clinical manifestations the patient presents with. Postauricular ecchymosis: _?_, _?_ and _?_ skull fractures Otorrhea: _?_, _?_ and _?_ skull fractures _?_ skull fractures are linear fractures that occur at the skull base Other signs and symptoms of skull fractures include: Blindness, vision field cuts, deafness, ataxia, & loss of taste depending on cranial nerve involvement

Parietal, basilar, & temporal; Temporal, parietal, & basilar; Basilar

_?_ remission - cancer shrinks but not disappeared _?_ remission- all sx of disease have disappeared _?_ - complete remission for several years (different cancers have different time lengths)

Partial Complete Cure

Which assessment parameter is of highest priority when caring for a patient undergoing a water deprivation test? Serum glucose Patient weight Arterial blood gases Patient temperature

Patient weight

_?_ - classified by speed of object causing injury.

Penetrating injury

_?_ is when a foreign body impales or passes through the body tissues

Penetrating trauma:

What are 7 s/e of steroids?

Peptic ulcers HTN Osteoporosis Na and H2O retention Hyperglycemia Infection Delayed Healing

2 Complications of acute Pericarditis:

Pericardial Effusion Cardiac Tamponade

Treatment for cardiac tamponade

Pericardiocentesis with surgical repair (if sac is intact)

The following is the process for _?_ get peripheral blood as outpatient use cell separator equipment; stem cells separated from blood; donor gets remaining blood components back takes 2-4hrs; often more than 1 time to get enough stem cells might give hematopoietic growth factors; bone marrow release more stem cells; harvest 4-5 days after injection

Peripheral Stem Cell Transplant (a type of HSCT (hematopoietic stem cell transplant))

In _?_, the peritoneal membrane used and acts as semi-permiable membrane

Peritoneal dialysis (PD):

_?_ aims to limit pressure in the pulmonary system, and avoid volutrauma (alveolar overdistention and damage from high alveolar pressures). This is accomplished with a low tidal volume setting (6 mL/kg of predicted body weight) and a normal breathing rate.

Permissive hypercapnia

Manifestations of hyperparthyroidism: 4 Neuro:

Personality (emotional) disturbances, memory impairment, hyperactive deep-tendon reflexes, fatigue

Objective data in nursing assessment for patient with IE: Osler's nodes Splinter hemorrhage Janeway's lesions _?_ _?_ _?_ _?_ Dysrhythmias Tachycardia Leukocytosis Anemia ↑ ESR and cardiac enzymes Positive blood cultures ECG showing chamber enlargement

Petechiae Clubbing Tachypnea Crackles

In secondary injury, _?_ noted in central gray matter of cord and later in white matter of cord may result. _?_ happens when cord compressed by hemorrhage or bony fragments. Hemorrhage and dilation of blood vessels can result in _?_ and lead to shock.

Petechial hemorrhages; Edema hypovolemia

_?_- Used for sedation, seizure control, and reduction of metabolic rate Reduces cerebral edema, CBF, & ICP levels Dosing based on EEG tracing & ICP Effective when total burst suppression displayed on EEG tracing

Phenobarbital Sodium (Nembutal) - a barbiturate

_?_ is an endocrine disorder which may be related to medications (anti-hypertensives, opiods, radiologic contrast media, and tricyclic antidepressants (TCAs)

Pheochromocytoma

A patient with esophageal cancer may undergo _?_ therapy which involves injecting photphrin into the tumor. The patient may not _?_ for 4 weeks post-treatment

Photodynamic; be in direct sunlight

three things that help diagnose flail chest

Physical exam, chest X-Ray, ABG assist with diagnosis

Drug therapy for Esophageal Varices (Arterial bleeding): _?_which can be given systemically or intraarterially

Pitressin

The physician orders fluid replacement (hourly matching of the previous hours' urine total) for Jim, who is severely dehydrated. Besides fluid replacement, what medication is the physician likely to order to treat the diabetes insipidus?

Pitressin (vasopressin)

_?_ great toe flexes toward the top of the foot and the other toes fan out after the sole of the foot has been firmly stroked. This is normal in younger children, but abnormal after the age of 2. The presence of this reflex indicates damage to the nerve paths connecting the spinal cord and the brain (the corticospinal tract). Because this tract is right-sided and left-sided, this reflex can occur on one side or on both sides.

Plantar/Babinski:

_?_: - Removes circulating antibodies Plasma selectively separated from whole blood. Blood cells returned to pt. Plasma can replace itself, may be transfused w/ colloidal substitute (albumin) Fresh frozen plasma generally not used d/t associated risk of infection/allergic pulmonary edema. Usually 3 or 4 treatments, 1 to 2 days apart.

Plasmapheresis

_?_ and _?_ have been given/used to lower the number of preformed HLA antibodies in highly sensitized patients in need of organ transplant

Plasmaphoresis and IV immune globulin

3 stages make up acute GBS _?_ (several days to 2 weeks)

Plateau period

_?_ is a type of surgery in which part of the pleura is removed. This procedure helps to prevent fluid from collecting in the affected area. Resulting adhesions causes pleural layers to fuse

Pleurectomy

Blunt trauma may result in 8 conditions. Name them:

Pneumothorax Rib fractures Flail chest Cardiac or pulmonary contusion Hemothorax Cardiac tamponade Great vessel tear ARDS

7 Clinical Manifestations HHS

Polyuria Polydipsia Blurred vision/H/A Orthostatic hypotension Change in LOC Seizures Reversible paralysis

PEEP

Positive End-Expiratory Pressure. The amount of residual pressure in the lungs at the end of expiration that keeps alveoli partially inflated so they are more easily inflated and perfused with the next breath. ~ 5 cm H2O

The patient's cardiac index is declining significantly. The RN notifies the MD. The RN would expect the MD to consider ordering which of the following to directly and quickly improve the cardiac index? Calcium channel blockers (e.g. diltiazem) Betablockers (e.g. metaprolol) Vasodilator (e.g. Nitroglycerin) Positive inotropic agents (e.g. dopamine)

Positive inotropic agents (e.g. dopamine)

The following are s/s of _?_ which may be diagnosed _?_ to _?_ post injury: Persistent headache Lethargy Personality and behavior changes Shortened attention span Decreased short term memory Changes in intellectual ability

Post Concussion Syndrome Diagnosed if the patient presents with the following symptoms 2 weeks to 2 months after concussion:

_?_- Very rare condition Usually dorsal columns are damaged Results in loss of proprioception Pain, temperature sensation, and motor function below level of lesion remain intact

Posterior cord syndrome

The following are _?_ etiologies which may lead to AKI: benign prostatic hyperplasia (BPH) and prostate cancer, renal calculi, bladder cancer, trauma, and extrarenal tumors

Postrenal AKI

The breakdown of RBC's after a severe burn will result in false highs of serum _?_ levels

Potassium

Keep __?__ within normal parameters to decrease the risk of ventricular dysrhythmias.

Potassium (around 2.0)

Reduced _?_ can cause low post-op CO. If CVP or PAWP indicated that it needs to be increased, it may be with cystalloid, colloid or PRBC. Want PWP or PAD to be 18-20 for greatest hemodynamic stability.

Preload

The following are characteristics of _?_ AKI: hypovolemia, decreased CO, decreased peripheral vascular resistance; decreased renovascular blood flow.

Prerenal

What are the 3 etiologies which may lead to AKI?

Prerenal, Intrarenal, Postrenal

_?_ combines pressure-limited ventilation with an inverse ratio of inspiration to expiration. The clinician selects the pressure level, rate, and inspiratory time. With the prolonged inspiratory time, autoPEEP may result

Pressure Control Inverse Ratio Ventilation (PC-IRV)

PSV

Pressure Support Ventilation Positive pressure that is applied to the airway ONLY DURING INSPIRATION. The patient initiates and performs that actual breaths, but the machine makes them easier with positive pressure. Think of really smart CPAP. Decreases the work of breathing. Patient has to have their own stable respiratory drive.

_?_ provides an augmented inspiration to a spontaneous breathing patient. The clinician selects inspiratory pressure level, PEEP and sensitivity. When the patient initiates a breath, a high flow of gas is delivered and pressure is maintained throughout inspiration. The patient determines the parameters of Vt, rate and inspiratory time.

Pressure Support Ventilation (PSV)

_?_ is positive pressure used to augment patient's inspiratory pressure. Usual setting is _?_ to _?_ cm H2O

Pressure support; 6-18 cm H2O

_?_ hyperparathyroidism is caused by benign neoplasms or adenoma in the parathyroid gland

Primary

3 Hyperparathyroid Classifications:

Primary Secondary Tertiary

TBI Categories _?_ occur at the time of injury due to physical stress or forced caused by a trauma _?_occur after the time of injury that worsen or negatively effect the patient outcome.

Primary TBIs Secondary TBIs

What are the principle risk factors for Infective Endocarditis (IE)? Name several:

Prior endocarditis Prosthetic valves Acquired valvular disease (Mitral valve prolapse with regurg, calcified aortic stenosis) UTI Cardiac lesions (VSD, asymmetric septal hypertrophy) Rheumatic heart disease (mitral valve regurg) Congenital heart disease Pacemakers Cardiomyopathy IV drug abuse Nosocomial bacteremia Intravascular devices (pulm artery cath)

Surviving Sepsis Campaign: _?_ used to support discontinuation of empiric antibiotics in pts who appeared to have sepsis, but have limited clinical evidence to support infection

Procalcitonin

Name 3 Antiemetics & Analgesics that may be used for hepatitis - caution if liver secreted

Prochlorperazine (Compazine), promethazine (Phenergan), ondansetron (Zofran)

_?_ is the third stage of cancer. It is characterized by an increase in growth rate of tumor, invasiveness and metastasis

Progression

S/S of the _?_ stage of shock: Listless or agitated, decrease responsiveness Increased HR Decreased B/P (mean <60 mmHg) Dysrhythmias, MI, decreased cap refill and peripheral pulses Anasarca Increased RR, crackles, decreased SaO2

Progressive stage

_?_ stage of shock: begins as compensatory mechanisms fail In this stage, aggressive interventions are necessary to prevent the development of multiple organ dysfunction syndrome (MODS)

Progressive stage

A patient reports having a dry mouth and asks for something to drink. The nurse recognizes that this symptom can most likely be attributed to a common adverse effect of which medication that the patient is taking? Digoxin (Lanoxin) Cefotetan (Cefotan) Famotidine (Pepcid) Promethazine (Phenergan)

Promethazine (Phenergan)

6 Assessment Findings for Abdominal aneurysms:

Prominent, pulsating mass in abdomen, at or above umbilicus, Systolic bruit over aorta, Tenderness on deep palpation, Abdominal or lower back pain, Epigastric discomfort, Altered bowel elimination

Stage 2 of cancer is _?_ it is characterized by reversible proliferation of the altered cells

Promotion

Treatment of Addison's Disease besides replacement therapy: Protect against _?_ Protect from noise, light, and environmental temperature extremes; Wear _?_ Carry emergency kit with _?_ Diligent administration of _?_ Other meds that require glucocorticoid dose to be increased (_?_, _?_, _?_, _?_) _?_ inhibits steroid metabolism

Protect against infection Protect from noise, light, and environmental temperature extremes; Wear medical alert bracelet Carry emergency kit with IM inection of cortisone Diligent administration of corticosteroids Other meds that require glucocorticoid dose to be increased (Dilantin, barbs, rifampin, antacids) Estrogen inhibits steroid metabolism

_?_- normal genes that promote cell growth; act as a "lock" so that cells stay mature and functioning

Protooncogenes

6 Complications of chronic pancreatitis include:

Pseudocyst Bile duct or duodenal obstruction Pancreatic ascites or pleural effusion Splenic vein thrombosis Pseudoaneurysms Pancreatic cancer

Which type of Diabetes Insipidus is also called "primary". Explain what occurs:

Psychogenic extreme water intake caused by a structural lesion in thirst center or psychiatric disorder

The patho of pulmonary HTN and cor pulmonale: Insult Occurs (hormonal, mechanical) then _?_. This is followed by _?_, _?_, _?_ and _?_

Pulmonary Endothelial Injury Sustained Pulm HTN RV Hypertrophy Cor Pulmonale R HF

_?_ = Elevated pulmonary artery pressure, resulting in an increase in resistance to blood flow through the pulmonary circulation.

Pulmonary HTN

Following coronary artery bypass graft surgery a patient has postoperative bleeding that requires returning to surgery to repair the leak. During surgery, the patient has a myocardial infarction (MI). After restoring the patient's body temperature to normal, which patient assessment is the most important for planning nursing care? Cardiac index (CI) 5 L/min/m2 Central venous pressure 8 mm Hg Mean arterial pressure (MAP) 86 mm Hg Pulmonary artery pressure (PAP) 28/14 mm Hg

Pulmonary artery pressure (PAP) 28/14 mm Hg Pulmonary hypertension as indicated by an elevated PAP indicates impaired forward flow of blood because of left ventricular dysfunction or hypoxemia. Both can be due to the MI. The CI, CVP, and MAP readings are normal.

_?_ causes right ventricle hypertension and hypertrophy

Pulmonic Valve Stenosis

__?_ is almost always congenital; results in backward flow of blood into right ventricle

Pulmonic Valve Stenosis

_?_ is defined as a fall of systolic blood pressure of >10 mmHg during the inspiratory phase; can be observed in cardiac tamponade and in conditions where intrathoracic pressure swings are exaggerated or the right ventricle is distended, such as severe acute asthma or exacerbations of chronic obstructive pulmonary disease. Both the inspiratory decrease in left ventricular stroke volume and the passive transmission to the arterial tree of the inspiratory decrease in intrathoracic pressure contribute to the occurrence

Pulsus paradoxus

Circulatory system is also affected by hemolysis of ________ from circulating factors (oxygen free radicals) released at the time of the burn.

RBC

_?_classification is used to describe the stages of AKI.

RIFLE Classification

Complications and Management after Thoracic Surgery: Acute Respiratory Failure: assess _?_ and lung sounds q4hrs and prn (more freq in immediate post-op phase); _?_ Monitor ABGs, pulse ox O2 to keep sat >92% usually Monitor for _?_ Monitor depth of breathing, might be shallow due to pain, anesthesia. Atelectasis and pneumonia might develop. Monitor CT for patency, drainage amount, air leak

RR & work of breathing; turn, cough and deep breathe; restlessness, dyspnea, cyanosis

Cor Pulmonale Physical signs _?_ is seen on echo increase in intensity of the 2nd heart sound Chronic hypoxemia: leads to _?_, increased total blood volume and viscosity of the blood

RV hypertrophy polycythemia

_?_causes acute progressive viral encephalitis Spreads from contact site through CNS via peripheral nerve & muscle fibers Progression 2-14 days after exposure: flu like symptoms - pain, numbness 2-7 days later: agitation, hypersalivation, hydrophobia, dysarthria, vertigo, diplopia, hallucinations, hyperactive reflexes, nuchal rigidity, + Babinski reflex 7-10 days later: Coma

Rabies virus

The _?_ is a vessel that may be used in CABG but it is prone to spasm. _?_ and _?_ may be used to control these.

Radial artery; CCB Long-acting nitrates

_?_Dissection-removal of lymph nodes and their lymphatic channels; including, muscles, nerves, veins, and possibly bone. ***Generally will be done with total laryngectomy to decrease the risk of lymphatic spread. ***Involves extensive dissection and reconstruction.

Radical Neck

_?_is a medication used to prevent kidney-transplant rejection (it is not for liver or lung transplant patients) S/E: leukopenia, thrombocytopenia, hyperlipidemia, increased risk of lymphomas and skin cancers.

Rapamune

The patient with an adrenal hyperplasia is returning from surgery for an adrenalectomy. For what immediate postoperative risk should the nurse plan to monitor the patient? Vomiting Infection Thomboembolism Rapid BP changes

Rapid BP changes

Electrical Burns - Assessment _?_ is needed, and transfer to a Burn Center is indicated. Most damage is not readily apparent (iceberg effect as below the surface). Strong currents can cause muscle spasms strong enough to _?_ Risk for dysrhythmias (caution first 24 hours--cardiac arrest possible) Risk for _?_(renal damage and kidney failure). Identify entrance wound and exit wound

Rapid assessment fracture bones. myoglobinemia

5 s/s of flail chest

Rapid, shallow respirations Tachycardia Paradoxical chest movement Crepitus over fractures d/t subq air Splinting

A 64-year-old woman is admitted to the emergency department vomiting bright red blood. The patient's vital signs are blood pressure 78/58 mm Hg, pulse 124 beats/minute, respirations 28 breaths/minute, and temperature 97.2° F (36.2° C). Which physician order should the nurse complete first? Obtain a 12-lead ECG and arterial blood gases. Rapidly administer 1000 mL normal saline solution IV. Administer norepinephrine (Levophed) by continuous IV infusion. Carefully insert a nasogastric tube and an indwelling bladder catheter.

Rapidly administer 1000 mL normal saline solution IV.

f

Rate # of breaths administered / minute 8 - 12 bpm

3 stages make up acute GBS _?_ (usually 4-6 months, can be up to 2 years) Coincides w/ remyelination and axonal regeneration) Not all recovery completely - have permanent neuro deficits, refer to as chronic GBS

Recovery phase

Gerontologic considerations: Reduced _?_ causes increased risk for resp failure Physiologic aging of lungs Alveolar dilation Larger air spaces Loss of _?_ for gas exchange Diminished _?_ in airways Decreased _?_ Decreased resp muscle strength Takes longer for resp system to be stimulated to alter resp rate/ depth of breathing Smoking hx Poor nutritional status Less available physiologic reserve in heart/lung

Reduced ventilatory capacity causes increased risk for resp failure Physiologic aging of lungs Alveolar dilation Larger air spaces Loss of surface area for gas exchange Diminished elastic recoil in airways Decreased chest wall compliance Decreased resp muscle strength Takes longer for resp system to be stimulated to alter resp rate/ depth of breathing Smoking hx Poor nutritional status Less available physiologic reserve in heart/lung

Long-term considerations post-esophageal cancer surgery: Reoccurence Chronic _?_ Strictures Care with future admissions/future medical care Careful intubation: use _?_ Careful NG tube placement Aspiration precautions No _?_ tube placement because Chest xrays Location of stomach NG tube placement

Reflux scope Gastric tube/PEG - if esophagus tied to stomach or post-stomach

_?_ stage of sepsis: The patient in this stage of shock will demonstrate profound hypotension and hypoxemia. Failure of the liver, lungs, and kidneys will cause an increase in waste products Recovery is unlikely

Refractory stage

6 components of hyperkalemia treatment

Regular insulin IV NaHCO3 (IV) sodium bicarbonate Calcium gluconate (IV) Dialysis Kayexalate Dietary restriction

Primary nursing concerns with Vents

Regularly auscultate breath sounds and monitor RR for irregularities. Maintain appropriate tube placement and cuff inflation. Prevent skin breakdown. Develop plan for communication (before insertion if possible). Administer sedatives, analgesics, and/or paralytics prn to maintain comfort and promote cooperation with vent. ET tube suctioning. Monitor ABGs.

2 major types of valvular heart disease:

Regurgitation (prolapse/insufficiency) Stenosis (narrowing)

Following administration of a dose of metoclopramide (Reglan) to the patient, the nurse determines that the medication has been effective when what is noted? Decreased blood pressure Absence of muscle tremors Relief of nausea and vomiting No further episodes of diarrhea

Relief of nausea and vomiting

The patient with a large tumor pressing on the esophagus is scheduled to have a debulking procedure. The nurse explains that the expected outcome of this surgery is to: Relieve pain/symptoms Cure the esophageal tumor Prevent the metastasis of the esophageal cancer Classify the cancer

Relieve pain/symptoms

_?_- period of time when the cancer is responding to treatment or is under control. It can be partial or complete.

Remission

_?_of MV is recommended in most patients with Moderate-Severe or Severe chronic MR.....but new studies show replacement may be favorable long term. Surgery is indicated for _?_ patients with acute, severe MR and patients with severe, primary MR who are asymptomatic with LVEF _?_ and end-systolic dimension of_?_

Repair; symptomatic > 30% < 55 mm.

The nurse assesses a patient for signs of meningeal irritation and observes for nuchal rigidity. What indicates the presence of this sign of meningeal irritation? Tonic spasms of the legs Curling in a fetal position Arching of the neck and back Resistance to flexion of the neck

Resistance to flexion of the neck

What are the clinical manifestations of AKI for each of these systems? Respiratory (2) GI (5)

Respiratory - edema, crackles GI - hypoactive bowel sounds; ileus, anorexia, n/v

When caring for a patient during the oliguric phase of acute kidney injury (AKI), what is an appropriate nursing intervention? Weigh patient three times weekly. Increase dietary sodium and potassium. Provide a low-protein, high-carbohydrate diet. Restrict fluids according to previous daily loss.

Restrict fluids according to previous daily loss.

The nurse is caring for a 68-year-old man who had coronary artery bypass surgery 3 weeks ago. If the patient is now is in the oliguric phase of acute kidney disease, which action would be appropriate to include in the plan of care? Provide foods high in potassium. Restrict fluids based on urine output. Monitor output from peritoneal dialysis. Offer high protein snacks between meals.

Restrict fluids based on urine output.

_?_ cardiomyopathy: Rarest of the 3 Results in restriction of filling of the ventricles Impairs diastolic filling and stretch

Restrictive

Physiologic consequences of aortic regurg: _?_ blood flow from ascending aorta to left ventricle Results in _?_ Initially, left ventricle compensates by dilation and hypertrophy. Myocardial contractility eventually declines. _?_ and _?_ eventually develop

Retrograde volume overload Pulm HTN and R Ventricular Failure

4 types of Etiology for valvular heart disease:

Rheumatic heart disease Congenital heart problems Acquired (e.g. infective endocarditis) Autoimmune disease (e.g. SLE)

Fractured ribs are the most common chest injury resulting from trauma. Ribs _?_ to _?_ are most commonly affected. Two clinical manifestations are: _?_ and _?_

Ribs 5-10 are most commonly affected Pain Shallow breathing

The nurse is teaching the patient undergoing chemotherapy about how to prevent chemo-induced nausea (CIN) and vomiting. When does the nurse direct the patient and family to administer antiemetic therapy? At the first sign of nausea A few hours before a chemotherapy session Right before a chemotherapy session and on a scheduled basis As needed to control nausea, vomiting and pain

Right before a chemotherapy session and on a scheduled basis

An EVD is placed in the _?_ of the brain. This allows measure of ICP and drainage of CSF. The transducer must be leveled. If it is too low, it will __?__. If too high, it will _?_. After leveling the transducer, make sure to _?_

Right lateral ventricle; too low - drain off too much CSF and maybe even brain tissue; too high - drain off too little; LOCK THE BED

Diagnostic studies for IPAH: Definitive test = _?_ Other diagnostics ECG, CXR, PFTs, echocardiogram, and CT scans Diagnosis is made on average 2 years from onset of symptoms Diagnosis - in advanced states by then, and the pulmonary artery pressure is 2-3 times the normal. Must do through workup to exclude conditions that cause secondary pulmonary HTN

Right sided heart cath

CCB can not be used in patients who have _?_.

Right sided heart failure

What does "RIFLE" stand for?

Risk, injury, failure, loss, end stage

Sepsis clinical criteria infection + change in "SOFA" greater than or equal to 2. SOFA stands for

S - sepsis-related O - organ F - failure A - assessment

6 S/S and 2 Treatments of hypotension due to hemodialysis:

S/S: nausea, light-headedness, sz, vomiting, visual ∆s, chest pain Treatment: ↓volume of fluid being removed and give N.S. or albumin given by iv (may also increase squeeze with dopamine or levafed)

Auscultatory findings of aortic valve regurg: Soft or absent _?_ Presence of _?_ and _?_ Soft, high-pitched _?_murmur _?_ murmur

S1 S3 and S4 diastolic Austin-Flint

A normal physical assessment of the cardiovascular system includes: Auscultation: _?_ and _?_ heard; _?_ and _?_ pulses equal; No _?_ or _?_

S1 and S2 heard radial and apical pulse rate equal murmurs or extra heart sounds

GOH would like the following vital signs for donors: SBP HR Urine Output ABG pO2 Temp

SBP > 100 HR <100 Urine Output > 100 ml/hr ABG pO2 > 100 Temp between 36 and 38.5

_?_ IS PRIMARY CAUSE OF DEATH IN THOSE WITH MAJOR BURNS AND MAY LEAD TO MODS. AT THIS TIME, SYSTEMIC ANTIBIOTICS MAY BE USED (PNEUMONIA IS COMMON) FUNGAL INFECTIONS MAY OCCUR.

SEPSIS

Name 4 Metabolic Oncologic Emergencies

SIADH Hypercalcemia Tumor lysis syndrome Septic shock & DIC

Overproduction/oversecretion of ADH results in _?_

SIADH (syndrome of inappropriate antidiuretic hormone)

_?_ is when the vent delivers preset Vt at preset frequency in synchrony with patient's spontaneous breathing

SIMV - Synchronized Intermittent Mandatory Ventilation

-?_ is a systemic inflammatory response to a nonspecific insult either infectious (referred to as sepsis), or noninfectious (i.e. ischemia, infarct, trauma)

SIRS (Systemic inflammatory response syndrome)

How is CO calculated?

SV x HR

_?_aneurysm is pouchlike with a narrow neck connecting the bulge to one side of the arterial wall.

Saccular

The _?_ is a vessel that is also used in CABG. It is removed from one or both legs. _?_ and _?_ are needed post-surgery to improve patency.

Saphenous veins; antiplatelets statins

_?_ cancers that form in bone and soft tissues, including muscle, fat, blood vessels, lymph vessels, and fibrous tissue (such as tendons and ligaments). _?_ is the most common cancer of bone. The most common types of soft tissue cancer are leiomyosarcoma, Kaposi sarcoma, malignant fibrous histiocytoma, liposarcoma, and dermatofibrosarcoma protuberans.

Sarcoma: Osteosarcoma

Treatment for Cirrhosis Complication - Esophageal Varices: _?_ or ligation procedures Balloon tamponade ?_ Shunt (TIPS)

Sclerotherapy Transjugular Intrahepatic Portosystemic

3 treatments for esophageal varices bleeding

Sclerotherapy or ligation procedures Balloon tamponade Transjugular Intrahepatic Portosystemic Shunt (TIPS)

__?__ hyperparathyroidism is caused by a compensatory response to chronic hypocalcemia

Secondary

_?_ is Pulmonary Hypertension (PH) caused d/t primary disease pathology resulting in anatomic or vascular changes

Secondary Pulmonary Arterial Hypertension (SPAH)

_?_ is the amount of effort the patient must generate to initiate a ventilator breath. Usually setting is _?_ cm H20 below baseline pressure and flow trigger is set _?_ L/min below baseline flow.

Sensitivity; 0.5 - 1.5 1-3

_?_ is a life-threatening organ dysfunction due to a dysregulated host response to infection

Sepsis

What are 6 possible causes of ARDS

Sepsis Aspiration PNA Transfusion-related acute lung injury Embolism Trauma Hypovolemic shock

S/S of Severe Sepsis include: Sepsis induced _?_ Elevated _?_ levels Urine output <_?_ mL/kg/hr for more than 2 hrs (even though adequate fluid resuscitation) Acute lung injury with PaO2/FIO2 _?_ (in the absence of pneumonia as infection source)

Sepsis induced hypotension Elevated lactate levels Urine output < 0.5 mL/kg/hr for more than 2 hrs Acute lung injury with PaO2/FIO2 < 250

_?_Shock causes include: Infection - gram-negative bacteria most common, gram positive bacteria, viruses, fungi and parasites At risk patients

Septic

Clinical manifestations of _?_: ↑ coagulation and inflammation ↓ fibrinolysis Formation of microthrombi Obstruction of microvasculature Hyperdynamic state: increased CO and decreased SVR Tachypnea/hyperventilation Temperature dysregulation ↓ urine output Altered neurologic status GI dysfunction Respiratory failure is common.

Septic Shock

_?_ : Presence of sepsis with hypotension despite adequate fluid resuscitation, that is not explained by other causes; presence of tissue perfusion abnormalities

Septic Shock

_?_: Presence of sepsis with hypotension despite adequate fluid resuscitation, that is not explained by other causes; presence of tissue perfusion abnormalities

Septic shock

Liver Diagnostic Tests: What are the normal lab values for the following? Serum albumin: Prothrombin Time: Alkaline phosphatase: PT/INR

Serum albumin: 3.5 -5 g/dl Prothrombin Time: 12-16 seconds Alkaline phosphatase: 17-142 U/L (biliary tract obstruction) PT/INR: Prolonged (normal PT is 11-16 seconds)

The nurse is attending to a patient with tentative diagnosis of acute pancreatitis. Which diagnostic tests should the nurse anticipate for the health care provider to prescribe to confirm the diagnosis? Select all that apply. Serum amylase Serum lipase Computed tomography (CT) scan Liver biopsy Serum alpha-fetoproteins

Serum amylase Serum lipase Computed tomography (CT) scan

Assessment Findings for Rupturing abdominal aneurysm:

Severe abdominal or lower back pain, Lumbar pain radiating to flank/groin, Hypotension, Increased pulse, Signs of shock, Hematoma at flank

8 clinical manifestations of tension pneumothorax

Severe respiratory distress Absent breath sounds on the affected side Tachycardia Hypotension Neck vein distention Anxiety - impending doom! Decreased SaO2 Mediastinal and tracheal shift to unaffected side (which impedes on CO and good lung's fx)

_?_: sepsis plus organ dysfunction with hypoperfusion/hypotension

Severe sepsis

Assessment Findings for Rupturing ascending or aortic arch aneurysm:

Severe, sudden, intense and persistent chest or back pain, Pain that radiates to your back, Trouble breathing, Low blood pressure, Loss of consciousness, Shortness of breath, Trouble swallowing, Weakness or paralysis of one side of the body, difficulty speaking, or other signs of stroke

What are 3 non-modifiable risk factors for aortic aneurysm?

Sex (males are more at risk), family history and age

First 2 stages of grief and mourning post-injury 1. _?_ struggles for survival, has complete dependence, may withdraw, sleep, have fantasies that they will recover, unrealistic expectations Nursing Interventions: meticulous nursing care, provide honest information, use simple explanations/diagrams to describe injury, encourage patient 2. _?_ refuses to discuss paralysis, has decreased self esteem, manipulates, hostile, abusive language Nursing Interventions: coordinate care with pt and encourage self care, support family members, use humor as appropriate, allow patient outbursts with some limits, do not allow fixation on injury

Shock and Denial: Anger:

The nurse is caring for a 56-year-old man receiving high-dose oral corticosteroid therapy to prevent organ rejection after a kidney transplant. What is most important for the nurse to observe related to this medication? Signs of infection Low blood pressure Increased urine output Decreased blood glucose

Signs of infection

GI care post esophageal cancer surgery: PO intake Eating position = _?_ for at least _?_ hours Eat at least _?_ before bedtime (reduces reflux) Drink fluids_?_ meals Prevent _?_syndrome

Sit up-right to eat and for 2 hours after 3 hours between dumping

Classification of Spinal Injury: _?_Level - Vertebral level where most damage to vertebral bones and ligaments _?_ Level - Lowest segment of spinal cord with normal sensory and motor function on both sides of body

Skeletal Neurologic

A normal physical assessment of the cardiovascular system includes: Inspection: Skin is _?_, Cap refill is _?_, no _?_ visible, No_?_

Skin is warm, normal color cap refill < 2 seconds PMI not visible No JVD with patient at 45 degrees

Where are re-epithelializing cells located? What does this mean for patients with severe burns? (i.e. deep partial thickness or full-thickness)

Skin-reproducing (re-epithelializing) cells are located along shafts of hair follicles, sweat glands, and oil glands; severe burn patients may not be able to produce new skin

The following are signs of _?_ Rapid onset Frequent/Copious Vomiting Colicky, Intermittent Pain Feces for a short time Minimally increased abd size

Small Bowel Obstruction

Why are small syringes not normally used with central lines and PICCs?

Small syringes exert high pressure. IV catheters can break or rupture from high pressure. Catheter breakage or rupture can result in catheter embolus, and require catheter replacement. Agency protocol should be followed, and may require that a 5 mL or larger syringe be used with central lines, peripherally inserted central catheters (PICCs), and midline catheters

During the Fluid and Electrolyte shift post-burn: _?_ rapidly moves to the interstitial spaces and remains there until edema formation ceases. _?_ shift develops because injured cells and hemolyzed RBC's release potassium into the circulation.

Sodium Potassium

Normal lab values for the following are: Sodium: Potassium: Chloride:

Sodium: (normal 135-145 mEq/L) Potassium: (normal 3.5-5.0 mEq/L) Chloride: (normal 98-110 mEq/L)

Drug therapy for upper GI bleed: _?_ or _?_: reduce splanchnic blood flow and acid secretion _?_ and _?_ if H-Pylori infection

Somatostatin or octreotide (Sandostatin) Antibiotics and PPI

What is the mnemonic for remembering if cranial nerves are sensory, motor or both?

Some Say Marry Money But My Brother Says, "Be Bold, Make Money!"

_?_ occurs when a tumor is occupying the epidural space. Results in weakness, bowel/bladder dysfunction. Treatment may include radiation or steroids to decrease inflammation

Spinal cord compression

_?_ shock is characterized by decreased reflexes, loss of sensation and flaccid paralysis below the level of injury. This syndrome lasts _?_ to _?_ and may mask postinjury neurologic function.

Spinal shock days to months

The nurse is caring for a patient admitted with a spinal cord injury following a motor vehicle accident. The patient exhibits a complete loss of motor, sensory, and reflex activity below the injury level. The nurse recognizes this condition as which of the following? Central cord syndrome Spinal shock syndrome Anterior cord syndrome Brown-Séquard syndrome

Spinal shock syndrome

Collaborative care for acute liver failure: Rest Admin of B-complex vitamins Avoid alcohol; minimize/avoid: aspirin/tylenol/ NSAIDS Sodium restriction May give albumin Diuretic therapy (loop & potassium sparing) _?_ : antagonist of aldosterone IV antibiotics Paracentesis Peritoneovenous Shunt If ammonia elevated: _?_ and neomycin or rifaximin (Rifampin)

Spironolactone Lactulose (Cephulac)

5 Vascular manifestations of IE

Splinter hemorrhages in nail beds Petechiae Osler's nodes on fingers or toes Janeway's lesions on palms or soles Roth's spots

Nursing Assessment #7.....penetrating trauma **_?_ Patient will need immediate cardiothoracic surgery

Stabilize the penetrating object....do not attempt to remove

Clinical staging of cancer Stage 0 = Stage 1 = Stage 2 = Stage 3 = Stage 4 = Stage 5 =

Stage 0 = in situ Stage 1 = localized Stage 2 = limited local spread Stage 3 = extensive local/regional spread Stage 4 = metastasis Stage 5 = distant metastasis

_?_ - occur from bleeding between the dura mater and arachnoid layer of the meningeal covering of the brain; usually caused by _?_ Can be acute _?_ hrs, subacute _?_ hr to _?_ wk, or chronic _?_da

Subdural Hematomas; veins acute = 24-48 hr subacute = 48 hr - 2 weeks chronic > 20 days

Clinical manifestations of aortic valve regurgitation are those of sudden cardiovascular collapse because Left ventricle exposed to aortic pressure during diastole. Name 4 s/s: Constitutes a medical emergency

Sudden weakness Severe dyspnea Chest pain Hypotension

Assessment Findings for Rupturing thoracic aneurysm:

Sudden, intense and persistent chest or back pain, Pain that radiates to your back, Trouble breathing, Low blood pressure, Loss of consciousness, Shortness of breath, Trouble swallowing, Weakness or paralysis of one side of the body, difficulty speaking, or other signs of stroke

_?_ occurs when the large vein entering the heart it blocked. This causes fluid build up in the upper torso and distention of the neck veins. May result in seizures. Possible treatment: debulking surgery

Superior vena cava syndrome

Unless medically contraindicated, how should a mechanically ventilated patient be positioned?

Supine, HOB at 30 - 45 degrees (to prevent aspiration)

A patient who is not intubated may need _?_ to reach his caloric intake goal.

Supplemental, high calorie foods

Treatment for head/neck cancer may include: _?_-involves removing structures above true cords (the false cords and epiglottis). High risk for aspiration, requires temp tracheostomy. Pt can speak as true vocal cords are intact.

Supraglottic Laryngectomy

When a patient is in septic shock, there is an increase in _?_ because _?_

SvO2; patient cannot use the O2 in the blood

SIMV

Synchronized Intermittent Mandatory Ventilation A preset rate and tidal volume are set, but the machine allows the patient to initiate AND perform their own breaths as well. Positive pressure is applied to ASSIST with the breath, but it is the patient doing the WOB. The ventilator synchronizes the automated breaths with the patient's own breaths.

_?_ is a type of allogeneic stem cell transplant in which the patient gets stem cells from identical twin

Syngeneic

_?_ antibiotics are not routinely used to control burn wound flora, as burn _?_ does not have much blood supply/ but topical may penetrate the eschar (_?_ and _?_ especially used).

Systemic; eschar Silvadene and Sulfamylon

A 50-year-old female patient smokes, is getting a divorce, and is reporting eye problems. On assessment of this patient, the nurse notes exophthalmos. What other abnormal assessments should the nurse expect to find in this patient? Puffy face, decreased sweating, and dry hair Muscle aches and pains and slow movements Decreased appetite, increased thirst, and pallor Systolic hypertension and increased heart rate

Systolic hypertension and increased heart rate

If cord injury above _?_ primary GI problems r/t hypomotility Decreased GI motor activity contributes to development of Paralytic ileus _?_ Nasogastric tube may help; Reglan may be used to treat delayed gastric emptying. _?_ common d/t excessive hydrochloric acid in stomach. Histamine H2 blockers (Zantac, Pepcid) or PPIs (Protonix, Prilosec, Prevacid) used to prevent ulcer.

T5, Gastric distention: Stress ulcers

Cord injury above level _?_greatly ↓ influence of sympathetic nervous system : Bradycardia occurs Hypotension d/t peripheral vasodilation Hypovolemia d/t ↑ in venous capacitance Cardiac monitoring is necessary Peripheral vasodilation ↓ Venous return of blood to heart ↓ Cardiac output IV fluids or vasopressor drugs may be required to support BP

T6

Neurogenic shock is generally associated with injuries of _?_ or higher

T6

Any cord injury above the level of _?_ greatly decreases the affect of the sympathetic nervous system. _?_ occurs and peripheral vasodilation results in _?_

T6 bradycardia hypotension

5 Other causes of Addison's Disease (besides autoimmune dysfunction):

TB, AIDS, carcinoma, histoplasmosis, medications (NIZORAL)

Respiratory manifestations of DIC include 3 things:

Tachypnea, Hemoptysis, Othopnea

Respiratory manifestations of DIC include 4 things

Tachypnea, dyspnea, PE, ARDS

Drug alert for _?_ and _?_: A substance in grapefruit and grapefruit juice prevents metabolism of these drugs. Consuming grapefruit or grapefruit juice while using these drugs could increase their toxicity.

Tacrolimus Cyclosporine

The following are exclusion criteria for _?_ Absolute Exclusions History of terminal illness or pre-existing comatose state Improving neurological status Documented primary intracranial event (CVA, trauma, status epilecticus) Relative Exclusions (physician has determined potential benefits outweigh risks). Coagulopathies or active bleeding Surgery within 14 days Documented sepsis Other causes of coma (drug overdose, encephalic causes, CNS infections.) Persistent hypoxia Continued hemodynamic instability despite vasopressors, fluid resuscitation Pregnancy

Targeting temp management

The following are inclusion criteria for _?_ Presenting rhythm Ventricular fibrillation or tachycardia (other presenting rhythms at physicians discretion) Return of spontaneous circulation within 60 minutes from time of collapse Persistent comatose state secondary to post arrest (GCS <10) Mechanically ventilated SBP>90 with or without vasopressors No uncontrolled dysrhythmias Initial temperature >30C Adult victim over 18 Women with negative UCG

Targeting temp management

What are 8 types medications that can cause SIADH?

Tegretol (carbamazepine) thiazide diuretics SSRI & tricyclic antidepressants opioids oxytocin anesthesia agents neoplastic agents

SIRS diagnosis requires two or more of the following signs or symptoms: Temperature less than _?_ or greater than _?_ HR _?_ Respiratory rate_?_ or paCO2 _?_ WBC greater _?_ or less than _?_ or _?_ bands (immature neutrophils)

Temperature > 100.4F (38 C) or < 93.8 F (36C) HR > 90 Respiratory rate > 20/min or PaCo2 <32 WBC > 12,000 or < 4000 or >10% bands (immature neutrophils)

Dialysis Vascular Access Sites: _?_ and _?_ are sites for immediate use, but use for only 1-3 weeks, no longer; placed in I.J., subclavian or femoral veins. They may NOT be used for _?_ unless directly ordered by nephrology; you must also know whether they contain _?_

Temporary (Quintins) and semipermanent venous catheters (Ash cath - tunneled cath); IV fluids Must ALSO know if that line has HEPARIN in it!!

#1 identify the pathology: *one-way valve exists, air leaks from the lung into pleural space-- unable to escape *increased intrapleural pressure; lung on involved side collapses, causes mediastinal shift (opposite side) * intrapleural pressures eventually cause shock

Tension pneumothorax

_?_: accumulation of air in pleural space without means of escape, causing complete collapse of lung and potential mediastinal shift

Tension pneumothorax

_?_ hyperparathyroidism is caused by a loss of response to serum Ca levels (Clinical manifestations r/t hypercalcemia and hypophosphatemia)

Tertiary:

Adrenal Cortex: Synthesizes 3 classes of steroid hormones. 1 class is androgens _?_ and _?_

Testosterone, Estrogen

_?_ is tingling of lips, fingertips and occasionally feet with increased muscle tension. It is caused by hypocalcemia/ hypoparathyroidism

Tetany

3 Manifestations of Hypocalcemia--hypoparathyroidism:

Tetany Positive Chvostek's Positive Trousseau's signs

_?_ is commonly used for classifying the severity of impairment resulting from SCI

The American Spinal Injury Association (ASIA) Impairment Scale

Barotrauma

The PRESSURE of the air can cause damage to lungs / alveoli. Patients with decreased pulmonary accommodation (emphysema) are at increased risk. Can lead to pneumothorax.

Volutrauma

The VOLUME of the air is more than the lungs can handle and physical damage to the lungs occurs.

The patient with a history of lung cancer and hepatitis C has developed liver failure and is considering liver transplantation. After the comprehensive evaluation, the nurse knows that which factor discovered may be a contraindication for liver transplantation? Has completed a college education Has been able to stop smoking cigarettes Has well-controlled type 1 diabetes mellitus The chest x-ray showed another lung cancer lesion.

The chest x-ray showed another lung cancer lesion

The patient with cirrhosis has an increased abdominal girth from ascites. The nurse should know that this fluid gathers in the abdomen for which reasons (select all that apply)? There is decreased colloid oncotic pressure from the liver's inability to synthesize albumin. Hyperaldosteronism related to damaged hepatocytes increases sodium and fluid retention. Portal hypertension pushes proteins from the blood vessels, causing leaking into the peritoneal cavity. Osmoreceptors in the hypothalamus stimulate thirst, which causes the stimulation to take in fluids orally. I Overactivity of the enlarged spleen results in increased removal of blood cells from the circulation, which decreases the vascular pressure.

There is decreased colloid oncotic pressure from the liver's inability to synthesize albumin. Hyperaldosteronism related to damaged hepatocytes increases sodium and fluid retention. Portal hypertension pushes proteins from the blood vessels, causing leaking into the peritoneal cavity.

Treatment for esophageal and gastric varices: To avoid bleeding /decrease risk, avoid _?_, _?_ and irritating foods. Treat respiratory infections promptly and control coughing _?_ are meds that may reduce BP

To avoid bleeding/decrease risk, avoid alcohol, aspirin, and irritating foods Treat respiratory infections promptly and control coughing Beta blockers

_?_- removal of the entire larynx and pre-epiglottic region. A permanent tracheostomy is performed. The trachea and the larynx are no longer connected. Need to carry a Medic Alert Tag for total neck breather. This procedure is for advanced lesions

Total Laryngectomy

_?_ care includes: care of the stoma Wash area around the stoma daily. Laryngectomy tube/tracheostomy tube should be removed and cleaned daily. Shield the stoma using a scarf or loose shirt. Cover stoma when coughing and during inhalation (foreign materials). Wear a plastic collar when bathing or showering. Maintain an adequate fluid intake. Wear a Medic Alert bracelet (neck breathing).

Tracheostomy

_?_ - Initially offered to only prohibitive risk patients but trials have shown safety in lower risk patients. Procedure is performed in the _?_ with both surgeon and cardiologist participating. Performed with conscious sedation or general anesthesia LOS is _?_ in healthier patients Treatment with _?_ postprocedure for 6 months

Transcatheter Aortic Valve Replacement. cath lab 2-3 days dual antiplatelet (Plavix and aspirin)

_?_ - is the standard for diagnosis of acute AD.

Transesophageal echocardiography (TEE)

_?_is a cancer that forms in a type of epithelial tissue called transitional epithelium, or urothelium. This tissue, which is made up of many layers of epithelial cells that can get bigger and smaller, is found in the linings of the bladder, ureters, and part of the kidneys (renal pelvis), and a few other organs. Some cancers of the bladder, ureters, and kidneys are transitional cell carcinomas.

Transitional cell carcinoma

_?_ is an indirect revascularization used for patients with advanced CAD who are not candidates for traditional CABG. Uses a high-energy laser to create channels in the heart to allow blood flow to ischeic areas

Transmyocardial laser revascularization (TMR)

3 causes of chylothorax

Trauma Surgical procedures Malignancy

Treat chylothorax with _?_ or chest tubes May require _?_

Treat chylothorax with thoracentesis or chest tubes May require pleurodesis

_?_ Occurs almost exclusively in patients with rheumatic mitral stenosis; those who are IV drug abusers or those who are treated with a dopamine agonist (ex: Parkinson's)

Tricuspid Valve Stenosis

_?_ results in right atrial enlargement and ↑ systemic venous pressures

Tricuspid Valve Stenosis

_?_: - Wall of the artery forms the aneurysm; At least one vessel layer still intact

True aneurysm

When instructing a patient regarding a urine study for free cortisol, what is most important for the nurse to tell the patient? Save the first voided urine in the morning. Maintain a high-sodium diet 3 days before collection. Try to avoid stressful situations during the collection period. Complete at least 30 minutes of exercise before collecting the urine sample.

Try to avoid stressful situations during the collection period.

_?_ is a type of metabolic oncologic emergency. It occurs when there is rapid death of tumor cells. There will be increases in P, K, and uric acid and a decrease in Ca. Watch for renal failure and heart dysfunction.

Tumor lysis syndrome

_?_ may occur 24-48 post-chemo and persist for 5-7 days. S/S are weakness, muscle cramps, diarrhea. Treatment is to decrease K, give fluids and allopurinol

Tumor lysis syndrome

_?_ - normal genes that suppress growth; prevent cells from going through cell cycle;

Tumor suppressor genes

Role of immune system a. _?_- antigens displayed on surface of cancer cells b. _?_ - response of immune system to TAAs

Tumor-associated antigens (TAAs) Immunologic surveillance

DKA most likely occurs in _?_ DM

Type 1

GOH's Organ Recovery Coordinator receives a list of possible "matches" from _?_ ; coordinator calls the transplant center for the potential recipient who tops the list for each organ; the recipient's transplant surgeon accepts or declines the organ.

UNOS

Collaborative Care for Gallbladder disease: List 3 diagnostic tests:

Ultrasound ERCP --(Endoscopic retrograde cholangiopancreatography) Percutaneous transhepatic cholangiography

The _?_ Regulates organ and tissue donation

Uniform Anatomical Gift Act (UAGA)

_?_ allows for fair and consistent transplantation laws among all states. Patients are matched to available donors based upon: ABO blood HLA typing, medical urgency, time on the waiting list, geographic location, and other factors specific to organs: age, weight, etc.

Uniform Anatomical Gift Act (UAGA)

Immediate assessment of patient with suspected _?_ Oxygen (ABCs); monitor V/S 2 large bore IVs & IV fluid LR or .9NS & blood products Monitor Fluid & Electrolytes PPI therapy Foley catheter NG or OG tube (might see lavage) Keep HOB elevated

Upper GI Bleed

Health promotion for _?_: Identify high risk patients Avoid irritants Take only prescribed medications Reduce stress If patient has _?_ avoid irritants and treat any _?_

Upper GI Bleed esophageal varices upper respiratory infection.

Interventions for _?_ Thorough Assessment and History for dehydration & F/E imbalances NPO Strict I&O (include vomitus and tube drainage) Comfort Measures Care of NG/ Intestinal Tubes

Upper GI Perforation

Emergency care for _?_ : ABCs; Respiratory status: airway, breathing V/S, peripheral perfusion Abdominal exam History BMP (monitor BUN), CBC, PT/INR, PTT, liver enzymes, ABG, Type and cross match, UA including specific gravity Test stools/emesis for occult blood

Upper GI bleed

Associated manifestations of _?_ include: blisters & edema hoarseness difficulty swallowing copious secretions, stridor substernal and intercostal retractions total airway obstruction.

Upper airway injury

_?_ - Results from inhalation injury to the mouth, oropharynx, and/or larynx.; Caused by thermal or inhalation injury due to steam, hot air, or smoke. _?_ may occur rapidly and progress to obstruction of airway, and may be lethal.

Upper airway injury Swelling

GI manifestations of DIC include 5 things

Upper/lower GI bleeds, Abdominal distention Abdominal pain, paralytic ileus

What are the clinical manifestations of AKI for each of these systems? Urinary (7) Cardiovascular (4)

Urinary - oliguria, cells in urine, increased BUN, Cr; changes in K, Mg, Na Cardiovascular - edema; increased BP initially; possible dysrhythmias (d/t increase in K); HF (d/t increase in fluids)

Signs & Symptoms of Oliguric Phase of AKI _?_ changes, Metabolic _?_, _?_ disorders, _?_accumulation _?_ disorders

Urinary changes, Metabolic acidosis, Hematologic disorders (impaired WBC and RBC production) Waste product accumulation Neurologic disorders (seizures, coma, confusion)

What points should a nurse emphasize while teaching a patient ways to protect oneself from exposure to hepatitis B infection? Select all that apply. Use disposable needles and syringes. Avoid eating food prepared in unhygienic ways. Avoid sharing toothbrushes and razors. Avoid touching or coming in contact with people with hepatitis B. Avoid unsafe sex with multiple partners, and always use condoms.

Use disposable needles and syringes. Avoid sharing toothbrushes and razors. Avoid unsafe sex with multiple partners, and always use condoms.

When caring for a patient with liver disease, the nurse recognizes the need to prevent bleeding resulting from altered clotting factors and rupture of varices. Which nursing interventions would be appropriate to achieve this outcome (select all that apply)? Use smallest gauge needle possible when giving injections or drawing blood. Teach patient to avoid straining at stool, vigorous blowing of nose, and coughing. Advise patient to use soft-bristle toothbrush and avoid ingestion of irritating food. Apply gentle pressure for the shortest possible time period after performing venipuncture. Instruct patient to avoid aspirin and NSAIDs to prevent hemorrhage when varices are present.

Use smallest gauge needle possible when giving injections or drawing blood. Teach patient to avoid straining at stool, vigorous blowing of nose, and coughing. Advise patient to use soft-bristle toothbrush and avoid ingestion of irritating food. Instruct patient to avoid aspirin and NSAIDs to prevent hemorrhage when varices are present.

Large bowel obstruction- vomiting may be absent if the ileocecal valve is competent Vomiting and dehydration - late or early signs? Abdominal pain and constipation Abdominal distention - late or early sign? Abdominal tenderness/rigidity

Vomiting and dehydration - late Abdominal distention - early

12 Manifestations of Addison's Disease:

Weakness, fatigue, anorexia, Skin hyperpigmentation, Hypotension, hypovolemia, Hyponatremia, hyperkalemia, hypoglycemia, Nausea, vomiting, diarrhea, ↓muscle size & ↓tone

Diuretic phase of AKI: Begins with _?_ and may last _?_

a gradual ↑ in daily urine output to 1-3L per day (may be 3-5L/day); May last 1-3 weeks

_?_ is normally the cause of anatomical shunting

a heart defect

What is a Roth's spot?

a hemorrhage in the retina of patients with IE

Severity of AKI dysfunction can range from a small increase in _?_ to a decrease in _?_ to the development of _?_

a small ↑ in serum creatinine or ↓ in urine output to the development of azotemia (BUN, creatinine in the blood)

If the patient's Hgb is below 10, what should they received prior to radiation therapy?

a transfusion

Examples of tumor suppressor genes and the cancer they cause when mutated a. BRCA1 - _?_ b. BRCA2 - _?_ c. APC - _?_ d. P53 - _?_

a. BRCA1 - breast/ovarian b. BRCA2 - breast/ovarian c. APC - precursor to colorectal cancer d. P53 - bladder, breast, colorectal, esophageal, liver, lung, ovarian

The nurse is caring for a patient with ARDS. Which intervention should the nurse initiate to prevent stress ulcers? a. Begin enteral feedings as soon as bowel sounds are present b. Administer prescribe lorazepam (Ativan) to reduce anxiety c. Observe stool for frank bleeding and occult blood d. Maintain HOB at 30-45 degrees

a. Begin enteral feedings as soon as bowel sounds are present

What is normal for each of the following? a. CI b. PVR c. SVR d. SvO2

a. CI = 2.2 - 4 L/min b. PVR = 160-380 dynes/sec/cm-5 c. SVR = 800-1200 dynes/sec/cm-5/m2 d. SvO2 = 65-75

What are the following characteristics of a malignant tumor? a. Encapsulated b. Differentiated c. Metastasis d. Recurrence e. vascularity f. Mode of growth g. Cell characteristics

a. Encapsulated - rarely b. Differentiated - poorly c. Metastasis - capable d. Recurrence - possible e. vascularity - moderate to marked f. Mode of growth - infiltrative and expansive g. Cell characteristics - abnormal; not like parent cells

What are the following characteristics of a benign tumor? a. Encapsulated b. Differentiated c. Metastasis d. Recurrence e. vascularity f. Mode of growth g. Cell characteristics

a. Encapsulated - usually b. Differentiated - normal differentiation c. Metastasis - absent d. Recurrence - rarely e. vascularity - slight f. Mode of growth - expansive g. Cell characteristics - fairly normal; similar to parent cells

Hematogenous metastasis includes: a. Primary tumor cells penetrate _?_ and _?_ b. Pass through _?_ ; some are trapped, some are not c. Create environment conducive to _?_

a. Primary tumor cells penetrate blood vessels and enter circulation b. Pass through lymph nodes; some are trapped, some are not c. Create environment conducive to growth/development

As the Aortic Dissection progresses down the aorta, the _?_ and _?_ demonstrate evidence of decrease tissue perfusion

abdominal organs (ischemic bowel) and lower extremities

List 4 s/s of ileus/paralytic ileus:

abdominal pain distention vomiting constipation

12% of patients on dialysis receive PD Access: obtained by inserting catheter through _?_ into _?_ Wait _?_ to use catheter - pending healing and sealing of exit site

abdominal wall into peritoneal cavity. 7-14 days

Acute kidney injury has a(n) _?_ onset. With _?_ it is often reversible.

abrupt onset (hours or days) and with prompt intervention is often reversible (has a high mortality rate).

Nursing interventions for _?_ may include: Monitor for hemodynamic stability Monitor for respiratory failure Monitor for hypocalcemia Monitor for infection Monitor for hyperglycemia and steatorrhea Maintain IV fluids as ordered Relieve pain NPO - NG tube

acute pancreatitis

Classification of aortic dissection based on duration of onset - _?_versus _?_

acute, chronic

How is V/Q mismatch treated in something like asthma?

add O2 to hopefully increase PaO2 by using medications such as nebs

Alkali burns may be more difficult to manage than acids, as alkalis _?_ causing protein hydrolysis and liquefaction. Organic compounds (phenols and petroleum products) produce contact burns and _?_

adhere to tissues; systemic toxicity

ACTH deficiency can lead to _?_ and _?_ due to sodium and water depletion.

adrenal insufficiency and hypovolemic shock

Treatment for Cushing syndrome: If adrenal tumor, treat with _?_

adrenalectomy

Etiology and Patho of Aortic Dissection - 11 Predisposing factors

age; aortitis (syphilis); blunt or iatrogenic trauma; congenital heart disease (bicuspid aortic valve); connective tissue disorders (Marfan's syndrome); cocaine or meth use; history of heart surgery; atherosclerosis; male gender; pregnancy; hypertension

Respiratory complications post burn: Vulnerable to upper airway burns AND lower airway injury. 4 things to watch for:. Admission x-ray and exam may not show the potential injury.

agitation, restlessness, tachypnea, or anxiety

If _?_ or _?_ enters the pleural space between the parietal and visceral pleura, the -4cmH20 pressure gradient that normally keeps the lung against the chest wall disappears and the lung _?_

air or fluid; collapses

_?_ is a post-operative complication of head/neck surgery caused by: Edema Manipulation/trauma to tissue Increased mucous production Mucous plug Monitor: Respiratory pattern, rate and effort Oxygen saturation Trach site

airway obstruction

_?_ is a "complete carcinogen" both initiate and promote the development of cancer

alcohol

_?_ regulates Na and K balance

aldosterone (a mineralocorticoid)

_?_ directly damage DNA to prevent the cancer cell from reproducing. Examples are: ifosfamide (Ifex) and cyclophosphamide (Cytoxan)

alkylating agents

3 types of hematopoietic stem cell transplant (HSCT) are:

allogeneic, syngeneic, autologous

Damage to the capillary membrane leads to interstitial edema. In ARDS, this will lead to _?_ which results in poor oxygenation.

alveolar edema

_?_ occurs when gas exchange across alveolar capillary membrane is compromised by a process that thickens or destroys the membrane or affects blood flow through pulmonary capillaries

alveolar hypoventilation

Positive end-expiratory pressure (PEEP) is used to hold open _?_ that are at risk for collapse. It is also used to increase alveolar volume and subsequently decrease _?_ . Increasing the area available for gas exchange should improve arterial oxygen levels, and allow FiO2 (oxygen concentration) to be decreased. PEEP can also enhance pulmonary compliance and decrease the work of breathing. PEEP is added cautiously.

alveoli ; intrapulmonary shunting

What type of feeding/decompression tube will a patient with a basilar skull fracture receive?

an OG tube (never an NG...it might end up in their brain)

Approximately 85% of the cases of endogenous Cushing syndrome are due to _?_

an adrenocorticotropic hormone (ACTH) secreting pituitary tumor

_?_ is a spinal cord syndrome results in motor paralysis and loss of pain/temperature sensation below the level of injury; sensation of touch, position, vibration and motion remain intact

anterior cord syndrome

Nursing Implementation for Valve Problems: Teach importance of completing _?_. Teach drug side effects. International normalized ratio for _?_therapy Follow-up care

antibiotic regimen; anticoagulation

Fungal and prosthetic valve endocarditis: Responds poorly to _?_ _?_ is adjunct procedure

antibiotics; Valve replacement

Neurogenic Bladder Drug therapy Suppress bladder contraction - _?_ Relaxation of urethral sphincter - _?_ Suppress pelvic floor spasticity -_?_ Fluid intake 3-4 L/day if indwelling cath Urine drainage Voluntary or reflex voiding Intermittent cath Indwelling cath Condom cath Surgery Sphincterotomy Electrical stimulation Urinary diversion

anticholenergics [oxybutynin ditropan)] anticholenergics [oxybutynin ditropan)] antispasmodics (baclofen)

ADH is _?_ (or _?_); it regulates water balance and osmolarity

antidiuretic hormone (or vasopressin)

The following are _?_ medications Reglan - metoclopramide Zofran - ondanestron Kytril - granisetron Decadron - dexamethasone Emend -aprepitant Compazine - prochlorperazine Phenergan - promethazine Tigan - trimethobenzamide hydrochloride

antiemetic

Collaborative Care for aneurysm: Pharmacological interventions: Administer _?_

antihypertensives to maintain BP within normal limits and to prevent strain on aneurysm

What are 2 psychological clinical manifestations of CKD?

anxiety depression

Classification of aortic dissection based on anatomic location - _?_versus_?_

ascending, descending aorta

Liver Transplant if possible, but preferable before disease worsens - before _?_ and _?_develop 2 options for obtaining a liver:

ascites and coagulopathies Cadaver or live (partial)

Prevention of _?_ may include: cease alcohol consumption early identification of viral hepatitis or other causes Nutrition - regenerate liver tissue small frequent meals, high calorie caution - protein & sodium use Rest - restore liver tissue: keep HOB elevated caution - pneumonia, pressure ulcers, clots Teaching - signs of complications, skin therapy, counseling for sexual problems, avoidance of hepatoxic meds, abstinence from alcohol (AA), nutrition, rest

ascites/liver failure

What does the waveform look like when assisted by a balloon device?

assisted end-diastolic has trough that is lower than normal while assisted systole has a peak that is lower than normal

Alveolar hypoventilation is usually d/t underlying causes such as CNS problems, _?_, chest wall trauma. It is considered both _?_ and _?_

asthma attack; hypercapnic hypoxemic

To be eligible for _?_ , the patient must have a non-survivable illness or injury that results in ventilator dependency, with a Glasgow Coma scale score of 5 or < A physician must determine that the pt. most likely will _?_ and _?_ within _?_ after the withdrawal of life support

asystolic donation arrest and expire 60 min.

Alcoholics and elderly are more at risk of head injuries because of _?_

atrophy of the brain that allows brain to "bounce around" in the skull during a fall

In marked bradycardia, (less than 40 beats/minute) appropriate drugs such as _?_ to increase heart rate and prevent hypoxemia are necessary.

atropine

A great deal of damage in SCI is related _?_ of the cord. _?_ areas in the center of the spinal cord appear within one hour and then by 4 hours there may be infarction in the grey matter

autodestruction; hemorrhage

_?_ and _?_ are medications which suppress T Cell activation and proliferation

azathioprine (Imuran) Sirolimus (Rapamune)

The nurse is caring for a patient with multiple fractured ribs. Which clinical manifestation, if experienced by the patient, is an early indication that the patient is developing respiratory failure? a. kussmaul respirations and hypotension b. frequent position changes and agitation c. cyanosis and increased cap refill time d. tachycardia and pursed lip breathing

b. frequent position changes and agitation

Degree of spasms- mild twitches to convulsive movements below the level of injury may occur Relief of spasms- controlled with use of antispasmodic and muscle relaxant drugs (_?_, dantolene, tizanidine) Relief of spasms- _?_ injections may also be given to treat severe spasticity

baclofen; botulism toxin

Nursing Assessment for AA - Key is to have a _?_for comparison with post-op assessments.

baseline

Keep drain _?_ for gravity drainage Gravity drainage without suction: the level of water in the water seal chamber + intrathoracic pressure (chamber is calibrated manometer) - slow , gradual rise in water level over time means more negative pressure in pleural space and signals healing; GOAL IS TO _?_

below the chest; RETURN TO -8CM H2O

Tumors can be _?_ or _?_. The first are well-differentiated. The second range from well-differentiated to undifferentiated

benign malignant

Complications of Hemodialysis: Loss of _?_ Hepatitis - hep B (now low incidence due to vaccine; now, hep C is more common --- with 10% of HD patients + for anti HCV (meaning previous infection) _?_ _?_ Syndrome

blood sepsis Disequilibrium

S/S of GI Perforation: pain, _?_ tenderness, _?_ abdomen distention, fever, chills

board-like rigidity of abdomen, rebound tenderness,

Hematopoietic growth factors stimulate _?_

bone marrow

3 sources of hematopoietic stem cell transplant (HSCT) are: Many risks and complications

bone marrow, peripheral circulating blood, umbilical cord blood

A patient with ARDS is on positive pressure ventilation (PPV). The patient's cardiac index is 1.4 L/min and pulmonary wedge pressure is 8 mm Hg. What order by the MD should be questioned by the nurse. a. Change the intravenous (IV) rate from 75 to 125 mL/hour b. Initiate a dobutamine infusion at 3 mcg/kg/min c. Increase PEEP from 10 to 15 cm H2O d. Administer one unit packed RBC over the next 2 hours

c. Increase PEEP from 10 to 15 cm H2O

Mechanisms of Action for _?_ Prevents cell-mediated attack against transplanted organ - no bone marrow suppression or alteration of normal inflammatory response. Used in combo w/ corticosteroids, sirolimus, & mycophenolate mofetil. Acts on T helper cells to prevent production and release of IL-2 and y- interferon Inhibits production of T cytotoxic lymphocytes and B cells

calcineurin inhibitors (cyclosporine/tacrolimus)

Side Effects of _?_ used in transplant patients: Nephrotoxicity Neurotoxicity (tremors, seizures) Hepatotoxicity Hirsutism Gingival hyperplasia Increased risk of infection Lymphoma Leukopenia

calcineurin inhibitors (cyclosporine/tacrolimus)

During _?_, blood is diverted from the patient's heart to a machine where it is oxygenated and then returned to the patient.

cardiopulmonary bypass (CPB)

Mediatinal shift: quickly lead to _?_ The vena cava and the right side of the heart cannot accept _?_= no cardiac output = unable to sustain life When the pressure is external,_?_ will not help - the heart will still not accept venous return Needle to relieve pressure followed by a _?_

cardiovascular collapse; venous return CPR chest tube

Assessment for co-morbidities before aneurysm surgery - Correction of the existing _?_ or _?_artery blockages may be needed before aneurysm repair

carotid or coronary

With burns, _?_ are released, proteins are broken down and there is an increased demand for _?_. In addition, there is likely an increase in body temperature. All of these factors require additional calories to be completed.

catecholamines glucose

After aneurysms repair, must monitor intraabdominal pressure indirectly via _?_ and _?_

cath and transducer system (usually via foley cath)

What is diabetes insipidus?

caused by decrease in anti-diuretic hormone; treat by replacing fluids and giving vasopressor

What is "steal syndrome"? Three methods to assess:

caused by ischemia (not enough blood flow) resulting from a vascular access device pallor, peripheral pulses, Allen's test

What is blue toe syndrome?

caused by plaque that forms within an aneurysm

Stages of Shock: Initial stage: Occurs at a _?_ level and is usually not clinically apparent Metabolism changes from_?_ to _?_. _?_accumulates and must be removed by blood and broken down by liver. Process requires unavailable O2.

cellular aerobic to anaerobic Lactic acid

_?_ is a spinal cord syndrome that manifests as motor weakness and sensory loss in both upper and lower extremities. (upper affected more than lower)

central cord syndrome

_?_ is bruising of brain tissue within a focused area that maintains the integrity of the pia mater and arachnoid layers Usually associated with closed head injury Coup-Contrecoup injuries Prognosis is dependent on the amount of bleeding around the contusion site

cerebral contusion

4 Focal Injuries

cerebral contusions Lacerations Hematomas cranial nerve injuries

Nurses must be __?_ to hang chemo drugs. Chemo drugs are _?_ that require proper handling

certified; vesicants

When a CT scan is not readily available, __?_ are obtained but it is often difficult to visualize C7 and T1.

cervical x-ray

What are 5 EARLY s/s of IICP

change in LOC Ocular signs (but NOT pupils) Motor/Sensory changes HA Possible seizures

Awareness of hypoxia or hypercapnia involves watching pulse ox, ABG trends, _?_ and _?_ as these may be the first signs

changes in mental status; changes in level of consciousness

The group of people who experienced SCD with an acute MI generally have prodromal symptoms such as _?_, _?_ and _?_. Death usually occurs within 1 hour of the onset of the acute symptoms

chest pain palpitations dyspnea

What are the 3 classic symptoms of aortic stenosis:

chest pain, heart failure and syncope

Evaluation of Plan for patient with IE: Vital signs WNL Absence of _?_, _?_ or _?_ Adequate _?_ Completion of ADLs with no fatigue or physiologic distress Increased understanding of disease process and self-care management

chills, diaphoresis, headache cardiac output

Gall Bladder disease is a common health problem for adults with _?_ ranking as one of the most common surgical procedures in the US. _?_ is a term meaning "stones in gallbladder" _?_ is a term meaning "inflammation of gallbladder"

cholecystectomy Cholelithiasis Cholecystitis

Complications of _?_ Gangrenous _?_ _?_— disease characterized by accumulation of infected fluid between diaphragm, liver, & spleen Pancreatitis _?_ (inflammation of biliary ducts) Biliary cirrhosis Fistulas Rupture of gallbladder Cholelithiasis (stones) Carcinoma Peritonitis

cholecystitis cholecystitis Subphrenic abscess Cholangitis

The following are clinical manifestations of _?_ Severe symptoms or no symptoms - depends if _?_stationary or mobile and if obstruction present Lodged _?_ in ducts or moving _?_ in ducts can cause spasms May produce severe steady pain Tachycardia, diaphoresis Pain usually 3-6 hours after high-fat meal or when person lies down

cholelithiasis stones stones stones

Risk factors for _?_ include: Liver disease Systemic Lupus Erythematosus Malignancy

chronic DIC

3 other causes of hypocalcemia besides hypoparathyroidism:

chronic kidney disease vitamin D deficiency hypomagnesium

Nursing Interventions for _?_ Focus on prevention of further attacks, relief of pain, control of pancreatic exocrine and endocrine insufficiency Pain control: may take large, frequent doses of analgesics for pain control diet, pancreatic enzyme replacement (Creon, Zenpep, Pancrease), control of DM

chronic pancreatitis

The following may cause _?_: EtOH abuse CF, Lupus Gallstones Tumor Trauma

chronic pancreatitis

What is Cushing's triad

classic vital sign changes with IICP; 1. increased systolic BP with widening pulse pressure 2. Bradycardia 3. Irregular respirations

Jim has a basilar skull fracture with a small subdural hematoma on the left side. As you assess Jim, you recall that the pathophysiology of subdural hematoma involves:

collection of blood between the dura mater and arachnoid mater

Lower GI Bleed Treatment: Usually no need for emergent _?_ If stable but continuing to bleed: Rapid ingestion of golytely and colonoscopy done in 6-12hrs

colonoscopy

Chronic hypercapnia/hypoxemia is generally better tolerated than acute. Why?

compensation is better for chronic because their body has had longer to adjust HOWEVER -- if they are no longer compensating, they are in RESPIRATORY FAILURE

_?_ results in total loss of sensory and motor function below the level of injury. _?_ results in a mixed loss of voluntary motor activity and sensation, leaving some tracts intact

complete cord involvement incomplete cord involvement

_?_ is considered a minor diffuse head injury A sudden transient mechanical head injury with disruption of neural activity and a change in level of consciousness

concussion

Liver Diagnostic Tests: Serum bilirubin, direct Serum bilirubin, total Urine bilirubin Urine urobilirubin Fecal urobilirubin All of these measure the liver's ability to _?_ and _?_ bilirubin

conjugate and excrete

What is the pain like with a paralytic ileus?

constant

_?_ are chemo drugs which disrupt cell membrane & inhibit synthesis of protein; decrease circulating lymphocytes; inhibit mitosis; depress immune system; increase sense of well being; commonly used as a pre-med to assist with decreasing side effect of chemotherapy. Examples are: dexamethasone (Decadron); hydrocortisone (Cortef); prednisone

corticosteroids

3 classes of medication which may cause GI bleeding:

corticosteroids, nonsteroidal anti-inflammatory drugs, salicylates

The adrenal cortex synthesizes 3 classes of steroid hormones. 1 class is glucocorticoids, primary _?_

cortisol

Diagnostic Studies for Addison's Disease: cortisol levels are _?_ potassium levels are _?_ sodium levels are _?_ chloride levels are _?_ glucose levels are _?_ BUN levels are _?_ RBC -levels are _?_ due to pernicious anemia MRI and CT to check for _?_ Also check for infections such as _?_

cortisol levels (decreased); potassium (increased) sodium (decreased) chloride (decreased) glucose (decreased); BUN (increased) RBC - decreased due to pernicious anemia MRI and CT to check for tumors/adrenal calcifiction Also check for infections such as TB

Hyperacute rejection is a rare event as the final _?_ usually determines whether the recipient is sensitized to any of the donor's _?_ (Most common in _?_ transplant)

crossmatch HLAs. kidney

A patient's arterial blood gas result is pH 7.31, PaCO2 49 mm, HCO3, 26 mEq/L and PaO2 52 mm Hg. What action is most important for the nurse to take? a. Instruct patient to breathe into a paper bag. b. Increase fluid intake to 2500 mL per 24 hours c. Perform chest physical therapy four times per day. d. Initiate oxygen at 2 L/min by nasal cannula

d. Initiate oxygen at 2 L/min by nasal cannula

In the Emergency Department (ED), Mr. Bennett's blood pressure dropped from 94/60 to 80/58. This was: a. not significant, because the drop in pressure was minimal b. not significant, since both readings were dangerously low c. significant, because a diastolic pressure below 60 cannot maintain renal perfusion d. significant, because it indicated that shock had progressed

d. significant, because it indicated that shock had progressed Pulse pressure (difference between systolic and diastolic pressures) narrows as hypovolemic shock progresses. Diastolic pressure remains adequate as systolic pressure decreases. On admission to the Emergency Department (ED), Mr. Bennett's blood pressure was 94/60. It dropped to 80/58. An admitting pulse pressure of 34 mm Hg narrowed to 22 mm Hg, which was significant.

Definitive treatment of MH is prompt administration of _?_ which slows metabolism, reduces muscle contraction, and mediates the catabolic processes associated with MH.

dantrolene (Dantrium).

Name some foods that would be high in calcium and therefore included in a hypoparathyroid patient's diet:

dark green veggies (broccoli), soy tofu Raw Milk Kale (cooked) Sardines (with bones) Yogurt or Kefir Watercress Cheese Bok Choy Okra Almonds

_?_ is a term meaning to remove as much of a tumor as possible by surgery or shrinking it with radiation or chemo

debulking

Hypoxemia results in a(n) __?__ in PaO2 and a(n) _?_ in SaO2

decrease in PaO2 and a(n) decrease in SaO2

Fluid loss and edema formation end, and interstitial fluid returns to the vascular space; diuresis occurs, and the urine has a ______ specific gravity.

decreased

Clinical manifestations of septic shock may include: Tachypnea/hyperventilation Temperature dysregulation _?_urine output Altered_?_ status GI dysfunction Respiratory failure is common.

decreased neurologic

H and H in CKD is _?_ because of _?_. Treat with _?_

decreased ; decreased erthyropoetin epogen

Clinical manifestations of septic shock may include: _?_ coagulation and inflammation _?_ fibrinolysis Formation of _?_ Obstruction of microvasculature Hyperdynamic state: increased _?_ and decreased SVR

decreased coag increased fibrinolysis microthrombi CO

If intra-aortic balloon pump (IABP) migrates proximally, what clinical sign would you see?

decreased left radial pulse

If a intra-aortic balloon pump (IABP) migrates distally, what clinical sign would you see?

decreased urine output

With any head injury, check the _?_ using impulses from a reflex hammer to stretch the muscle & tendon Assess: Biceps, Brachioradialis, Triceps, Patellar, Achilles Tendon

deep tendon reflexes

If you are secreting too little ADH, _?_ will occur. If you are secreting too much ADH, _?_ will occur

dehydration; edema

Older adults are more at risk of _?_ which increases mortality and morbidity. Get them up and get them moving! Don't use _?_ for anxiety!

delirium benzos

Important History and Physical Information with suspected Infective Endocarditis: Recent _?_, _?_, _?_, or _?_ procedures Heart disease Recent cardiac _?_ Skin, respiratory, or _?_ infection

dental, urologic, surgical, or gynecological catheterization urinary tract

Classification of Burn Injury: Severity of injury related to: _?_ of burn (destruction of integumentary system) (partial or full thickness) _?_ of burn calculated in percent of total body surface area (TBSA) _?_of burn Patient _?_ (age, past medical hx)

depth extent location risk factors

The _?_ in the suction bottle determines the amount of negative pressure that can be transmitted to the chest, NOT the reading on the suction regulator. There is no research to support the setting of _?_ just convention Suction can improve the speed at which air and fluid are pulled from the chest

depth of the water -20cmH2O,

a. Primary tumor cells penetrate blood vessels and enter circulation i. Most tumor cells are _?_ ii. Cells that survive continue on

destroyed by this process

Diagnostic Studies for Acute Pericarditis: EKG (_?_ ...all leads) ECHO (see _?_) MRI or CT to visualize pericardium/pericardial space Chest x-ray to see if _?_ d/t pericardial effusion

diffuse, wide spread ST segment elevation; pericardial effusion or cardiac tamponade cardiomegaly

The classic sign of _?_ is when hypoxemia is present during exercise but not at rest

diffusion limitation

Diagnostics for _?_cardiomyopathy Pt hx Exclusion of other causes of HF ECHO Chest xray EKG LAB Cardiac Cath

dilated

Syndrome of Inappropriate ADH (SIADH): Occurs when ADH is released despite normal or low plasma osmolarity with 7 results:

dilutional hyponatremia hypochloremia, fluid retention, weight gain without edema, hypo-osmolality, concentrated urine (high specific gravity) normal renal function

Spinal cord is wrapped in tough layers of _?_ Rarely torn or transected by direct trauma SCI due to cord _?_ by Bone displacement, Interruption of blood supply to cord or Traction resulting from pulling on cord Can be due to penetrating trauma (gunshot wound or stab wounds) can result in _?_ or _?_

dura ; compression; tearing or transection

Presence of Rhinorrhea or Otorrhea indicates that the fracture has traversed the _?_ Any leaking fluid should be tested to determine if the fluid is CSF

dura;

Tetany involves painful tonic spasms of smooth and skeletal muscles. It leads to _?_ and _?_; Positive _?_ and _?_ signs are seen

dysphasia and laryngealspasms; Positive Chvostek's & Trousseau's signs :

Secondary Pulmonary Arterial Hypertension (SPAH) Symptoms: 1. 2. 3. 4.

dyspnea fatigue lethargy chest pain

S/S of IPAH: d/t inability of cardiac output to increase in response to increased oxygen demand; with disease progresses, _?_.

dyspnea occurs at rest

5 Signs of high K

dysrhythmias peaked T waves bradycardia flaccid paralysis muscle weakness (put them on cardiac monitor)

Even if the injury is below C4, spinal cord _?_ and _?_ can affect the function of the phrenic nerve and cause respiratory insufficiency.

edema and hemorrhage

Etiology and Patho Non-traumatic aortic dissection: Degenerated _?_in the arterial wall; Hastened by_?_; Tear in the _?_of the aorta; Blood goes through tear into the middle layer, inner and middle layers separate (dissect); Aortic dissection often fatal

elastic fibers; hypertension; inner layer

The group of people who experience SCD without an acute MI are at risk of another event d/t _?_

electrical instability of the heart

_?_ is performed under fluoroscopy; electrodes are placed, stimuli are used to attempt to produce dysrhythmias. Then antidysrhythmic meds are used

electrophysiology study (EPS)

_?_ is caused by virus transmitted by ticks and mosquitoes complications of measles, chickenpox, mumps HSV or cytomegalovirus Signs & symptoms Headache, N/V, Fever, Mental alterations to coma, any CNS abnormality Diagnostic Tests CT, MRI, PET scan, EEG, test for antibodies in CSF to diagnose

encephalitis

The follow are _?_ clinical issues: Delirium Intractable or Refractory Symptoms Dyspnea Terminal Congestion Wound Care Ambivalence

end of life

Subjective data in nursing assessment for patient with IE: History of valvular, congenital, or syphilitic cardiac disease Previous _?_ _?_ infection _?_ therapy Recent surgeries and procedures Functional health patterns _?_ Alcohol abuse Weight changes Chills

endocarditis Staph or strep Immunosuppressive IVDA (IV drug abuse)

Airway Management Involves early_?_. If burns of face or neck, intubate in 1-2 hours after injury (before the edema sets in). Give _?_ High fowler's positioning unless ____??______ injury. Cough and deep breath every hour. PEEP and bronchodilators may be indicated. CO poisoning treated with 100% ___________ until Carboxyhemoglobin back to normal.

endotracheal intubation; humidified 100% oxygen. spinal cord humidified O2

GI care post_?_cancer surgery: Bowel function Assess for return of bowel sounds Monitor for bloating/diarrhea r/t tube feeds Monitor for constipation r/t narcotic use Monitor for diarrhea r/t antibiotic use

esophageal

Genitourinary Care post _?_ cancer surgery Foley immediately post op Monitor I/O q1hr: < 30ml/hr 2 consecutive hrs, call d/c foley as soon as possible (prevent UTI) Monitor chemistries/renal labs Neurological care Monitor for confusion Monitor for alcohol withdrawal

esophageal

Hematology care post _?_ surgery: Monitor incisions for bleeding DVT prophylaxis Monitor CBC Integumentary care Monitor incisions and invasive lines for s/s of infection Monitor skin for breakdown Daily braden scores Early and frequent mobilization PT/OT/ST

esophageal

Pulmonary Care after _?_ cancer surgery Acute respiratory distress syndrome - Can develop early in these patients (even by evening of OR) Difficult to predict All patients should be monitored for ARDS Careful monitoring of ABG's, oxygen saturations, chest xrays

esophageal

Pulmonary Care after _?_ cancer surgery Risk of pulmonary complications great Aggressive pulmonary toileting immediately postop Prevent atalectasis and pneumonia (major complications of surgery) Intubated after surgery, may be extubated by pm of surgery Monitor oxygenation closely

esophageal

Pulmonary Care after _?_ cancer surgery Check CT site for drainage, keep dressing clean, dry, intact. CT free of kinks Palpate surrounding tissue for subcutaneous air- may be due to_?_ from OR or may be due to leak at _?_.

esophageal pleural injury esophageal anastomosis

Pulmonary Care after _?_ cancer surgery May have chest tube Assess drainage initially after surgery q1hr and prn, then q4hrs-q8hrs and prn as pt condition improves. Drainage should decrease gradually Sudden change in color of CT drainage may indicate _?_, call surgeon immediately.

esophageal anastomotic leak

Pulmonary Care after _?_ cancer surgery Immediately post-op may need chest physiotherapy, nebulizers. While on vent may need suctioned. After extubation: initiate coughing, deep breathing, use of IS. Avoid _?_ suctioning due to risk of passing catheter through new anastomosis.

esophageal nasotracheal

Hemodynamics post _?_ surgery: IV fluids: isotonic NS or Lactated Ringers at rate of 100-200 ml/hr for 1st 12-16 hrs after surgery Fluid boluses: major fluid shifts occur 1st few days postop. Hypovolemia may be problem. Avoid fluid overload d/t lungs already compromised because of decreased lymph clearance r/t surgical removal of mediastinal lymphatics and nodes. (pt predisposed to interstitial pulmonary edema.) Need meticulous skin care due to 3rd spacing/ edema. Turn q2hrs & prn. Pressure relieving surface.

esophageal cancer

3 causes of GI bleeding in the Esophagus:

esophageal varices, esophagitis, Mallory-Weiss tear

General principles of _?_ surgery: Resection of area Use of Stomach as a Conduit Jejunostomy Tube Other Conduits: if entire esophagus and stomach removed, part of bowel used to maintain gastrointestinal continuity

esophagus

Hormone therapy drugs are chemo drugs which selectively attach to estrogen receptors, inhibits tumor growth. A specific type is _?_ which interfere with hormone receptors and proteins. Examples are: estradiol (Estrace), estrogen (Menest)

estrogens

Auscultatory findings in a patient with aortic stenosis: Normal to soft_?_ heart sound Diminished or absent _?_heart sound Systolic_?_ that ends before second heart sound Prominent _?_ sound

first; second; crescendo-decrescendo murmur fourth

_?_ is the most common post-operative complication of head/neck surgery related to: Infection Radiation Poor wound healing

fistulas

Motor manifestations of GBS - Ascending symmetric muscle weakness to _?_without muscle atrophy __?_ deep tendon reflexes (DTRs) _?_--dyspnea, diminished breath sounds, decreased tidal volume and vital capacity. This can all lead to respiratory failure and the need for mechanical ventilation. Most serious complication of GB Occurs d/t paralysis to nerves innervating thoracic area Loss of bowel and bladder control Pt may get ileus d/t decreased mobility _?_- inability to coordinate voluntary muscular movements that is symptomatic of some nervous disorders

flaccid paralysis ; Decreased or absent Respiratory compromise; Ataxia

_?_ is the most unstable type of SCI because all of the ligamentous structures that stabilize the spine are torn

flexion-rotation

With an increase in calcium, we want to increase _?_!

fluids!

GI care post esophageal cancer surgery: _?_ swallowing exam 5-7 days postop Check for _?_ before oral intake started If leak suspected, alt form nutrition continues No leak - started on clear liquid diet and advanced as tol PO intake 6 to 8 small frequent meals each day Avoid very hot/cold beverage & spicy foods Protein supplements, high energy foods, soft dysphagia diet may be needed Weekly lab panels to guide nutritional supplements/tube feeds Daily weights

fluorscopic anastomosis leak

You prepare to administer the Dilantin (phenytoin) piggy-backed through a Y-tube IV infusion set. The primary IV is a solution of Lactated Ringer's. Before infusing the Dilantin (phenytoin), you:

flush the IV line with normal saline

_?_ hormone is secreted by the anterior pituitary. It is involved in reproduction and puberty

follicle-stimulating hormone (FSH)

If an autotransplant of parathyroid tissue is performed, where might the tissue be implanted? What if it fails?

forearm or near sternocleidomastoid muscle may be done must take calcium for life

_?_ , uncontrolled _?_ influx, ischemia and _?_ peroxidation are classified as secondary injury

free radial formation uncontrolled calcium influx ischemia lipid peroxidation

True aneurysms are further classified into

fusiform and saccular types

6 causes of bleeding in the Stomach & duodenum:

gastric CA, hemorrhagic gastritis, peptic ulcer disease, polyps, stress-related mucosal disease, drug induced.

_?_ and _?_ vessels may be used in CABG, but rarely are d/t extensive time it takes to dissect them.

gastroepiploic inferior epigastric artery

Missing or altered genes may lead to cancer. _?_ is the transfer of exogenous genes into cells of patients in an effort to correct defective gene

gene therapy

Sargramostim (Leukine) is a hematopoietic growth factor which _?_. Its side effects are n/v, fever, chills, HA, fatigue

given after bone marrow transplant

Reduce risk for CVD: Optimal _?_ control Maintain _?_within normal range Prevent kidney insult (such as UTI's or kidney infections) Appropriate screening -[African(4x's that of whites) & Native Americans (2x's that of whites)] at increased risk

glycemic B/P

_?_ is a hormone released by the anterior pituitary. It is involved with carb/fat/protein metabolism, growth of long bones and skeletal muscle development

growth hormone (GH)

Some gene therapy targets _?_ while others target _?_

healthy cells cancer cells

Acute IE typically affects those with _?_ Shorter clinical course _?_onset

healthy valves/prosthetic valves; Rapid

Antibiotic administration in IE (based on organism): Monitor antibiotic serum levels Subsequent blood cultures Renal function monitored Treat symptoms Fluids, rest Monitor for S&S of _?_ _?_ may be necessary

heart failure Valve replacement

What is Grey-Turner's syndrome?

hematoma at flank; caused by ruptured abdominal aneurysm

The following medications are all types of _?_ filgrastim (Neupogen) erythropoietin (Epogen, Procrit) sargramostim (Leukine) Interleukin-11 (Neumega)

hematopoietic growth factors

Urinary manifestations of DIC include:

hematuria

2 methods of dialysis

hemodialysis Periotoneal dialysis

Tissue oxygen needs: Major threat of respiratory failure is the inability of the lungs to meet the oxygen demands of the tissues. Tissue O2 delivery is determined by the amount of O2 carried by the _?_ , as well as the _?_ _?_ and _?_ increase the risk of poor oxygenation.

hemoglobin; cardiac output. heart disease anemia

In _?_ jaundice, unconjugated bilirubin is high conjugated bilirubin is normal urine bilirubin is negative urine urobilinogen is high stool urobilnogen is high

hemolytic

3 Major complications for hyperparathyroidism surgery :

hemorrhage, fluid and electrolyte disturbances tetany

Secondary injury worsens the primary injury. Secondary injuries include: __?_, _?_ or _?_ Spinal cord: contused, lacerated or compressed by injury.

hemorrhage, ischemia, neurogenic shock.

__?_ is an inflammatory process throughout whole liver; Infected Hepatocytes destroyed; Cellular necrosis _?_ is most common cause;

hepatitis Viral hepatitis

In _?_ jaundice, unconjugated bilirubin is high conjugated bilirubin is high or low urine bilirubin is high urine urobilinogen is normal to high stool urobilnogen is normal to low

hepatocellular

In CKD potassium is usually _?_

high

Intracranial pressure (ICP) monitoring is initiated with Mrs. Thorne. She has an intraventricular catheter and ICP monitor in place. An initial ICP reading of 20 mm Hg is obtained. An ICP reading of 20 mm Hg is:

high

The PRA is calculated as a %, with a_?_ (high or low?) PRA indicating a _?_ (high or low?) number of cytotoxic antibodies, and is highly sensitized, so is a poor chance of finding a crossmatch-negative donor.

high high

Surviving Sepsis Campaign: If pt has sepsis induced ARDS moderate to severe ARDS: use _?_ over _?_ use prone over supine position in adult patients with a PaO2/FIO2 ratio <150.

higher PEEP over lower PEEP

Transplantation success has been due to advances in _?_testing, surgical techniques, and more effective immunosuppressants.

histocompatibility

How might post-menopausal women be treated for hyperparathyroidism if they have signs of osteoporosis? Does this treat the underlying parathyroidism? What 2 conditions/diseases does it put them at greater risk for?

hormone-replacement therapy; no; CA, DVT

A patient with acute pericarditis may appear to be SOB because breathing _?_. Therefore they breath _?_ and _?_.

hurts; fast shallow

Increased ICP causes _?_ and _?_ state; Increased need for glucose. Malnutrition causes _?_

hyerpmetabolic & hypercatabolic cerebral edema

_?_ occurs minutes to hours after transplant because the blood vessels are rapidly destroyed d/t person having preexisting antibodies No treatment except _?_

hyperacute rejection to remove the affected organ.

Treatment for _?_ includes fluids and diuretics

hypercalcemia

_?_ is a condition in which there is an increase of PaCO2

hypercapnia

_?_ occurs when ventilation demand is greater than ventilation supply

hypercapnia

_?_ results from an imbalance between ventilatory supply (the maximum gas flow in and out of the lungs) and ventilatory demand (amount of ventilation needed to keep the PaCO2 within normal limits) Acute or Chronic

hypercapnic respiratory failure

In the post acute phase of SCI, the atonic bladder becomes _?_; this results in reflex emptying

hyperirritable

Nutritional therapy for burns: Early and aggressive nutritional support -- within hours of the burn injury decreases mortality risks and complications, optimizes wound healing, and minimizes _?_ and _?_

hypermetabolism and catabolism.

Malignant hyperthermia is diagnosed after all other causes of _?_ are ruled out. 7 Early symptoms include:

hypermetabolism; increased end tidal CO2 Skeletal muscle rigidity Muscle spasms Tachycardia Metabolic and Resp. acidosis Tachypnea Sweating

4 endocrine/reproductive clinical manifestations of CKD

hyperparathyroidism Thyroid abnormalities amenorrhea erectile dysfunction

Name 4 skin reactions to chemo:

hyperpigmentation, photosensitivity, acne, extravasation

In _?_, there is an increase in the number of cells in an organ or tissue that appear normal under a microscope

hyperplasia

Cancers may follow a progression of abnormal changes from normal to _?_ to _?_ to cancerous.

hyperplasia dysplasia

"Triple H" therapy is commonly used to treat cerebral vasospasm, with the aim of preventing stroke. You know that "triple H" therapy involves use of:

hypertension hypervolemia hemodilution

1 ocular clinical manifestation of CKD

hypertensive retinopathy

A lack of PTH causes _?_ and _?_

hypocalcemia and ↑ phosphate

An 18-year-old male patient is undergoing a growth hormone stimulation test. The nurse should monitor the patient for hypothermia. hypertension. hyperreflexia. hypoglycemia

hypoglycemia

If the SCI has occurred above the level of T5, the GI problems are related to _?_

hypomotility

With aortic dissection, Elderly may not c/o pain so much as _?_ and _?_

hypotension and vague symptoms

Antidiuretic hormone (ADH) is made in the _?_, but stored in the __?_. It regulates _?_

hypothalamus; posterior pituitary fluid balance

_?_ almost always occurs with diaphragmatic respirations because of the decrease in vital capacity and tidal volume (a result of impairment of the intercostal muscles)

hypoventilation

A cord injury above T6 will result in relative _?_ because of the increase in the capacity of dilated vessels due to vasodilation

hypovolemia

_?_ is inadequate oxygen transfer to the blood

hypoxemia

The following cardiac system problems can lead to _?_. Anatomic shunt, cardiogenic pulmonary edema, shock, diffusion limitation

hypoxemic respiratory failure

Hypoxemia can lead to _?_

hypoxia

_?_ occurs when PaO2 decreases allowing s/s of inadequate oxygenation

hypoxia

_?_ is the most common cause of Cushing Syndrome.

iatrogenic administration of corticosteroids such as prednisone

Most common cause of hypoparathyroidism

iatrogenic damage to or removal of the parathyroid glands during thyroidectomy, or radical neck

Pulmonary HTN: Can be a primary disease (_?_ ) OR as a secondary complication of a respiratory, cardiac, autoimmune, hepatic, or connective tissue disorder (_?_)

idiopathic pulmonary arterial hypertension -- IPAH secondary pulmonary arterial hypertension

Describe the "treat hyponatremia" portion of SIADH treatment:

if less than 120 mEq/l, give hypertonic saline (3-5%) by very slow infusion; Don't raise the Na level too quickly or overshoot!

The most common approach to preventing a recurrence of SCD is _?_

implantable cardioverter-difibrillator (ICD)

What are the 2 categories of Subjective Data collected during a nursing assessment on a patient in acute respiratory failure?

important health info (past Health hx, Medications, surgeries) and Functional health patterns (health perception, nutrition, activity/exercise, sleep/rest, cognitive, coping)

If serum osmo is much lower than urine osmo, this indicates _?_

inappropriate excretion of concentrated urine and presence of dilute serum

6 syndromes are associated with _?_: central cord syndrome, anterior cord syndrome, Brown-Sequard syndrome, posterior cord syndrome, cauda equina syndrome and conus medullaris syndrome

incomplete cord involvement

AKI is characterized by: a rapid ↓ of kidney function demonstrated by a progressive increase in _?_ and/or a decrease in _?_

increase in serum creatinine and/or decrease in urine output

To prevent an Addisonian Crisis, Addison's patients should _?_ during times of stress

increase their doses of steroids

Magnesium in CDK is usually _?_ (increased or decreased?) 5 S/S

increased don't give Mylanta or MOM Dysrhythmias GI Issues Decrease in RR Hypotension Impaired CNS functioning

The primary goal after cranial surgery is to prevent _?_. Frequent assessment of neuro status, monitoring for infection, monitoring for _?_ and hypovolemia are key.

increased ICP; Diabetes Insipidus

Nursing Acute Interventions for patients with head injuries: Major focus of nursing care relates to _?_ Eye problems _?_ Raise head of patients leaking CSF

increased ICP; Hyperthermia

Name three conditions which would increase ICP

increased brain volume increased blood volume increased CSF

6 s/s of hypovolemic shock:

increased respirations increased pulse; decreased b/p; decreased urine output restlessness; cool, pale, clammy skin;

If a patient has a temperature, this _?_ metabolic rate which will _?_ ICP

increases increases

Emotional /psychologic needs of patients and caregivers Encourage discussion. Encourage appropriate _?_ and some return to the pre-burn activities. Address spiritual and cultural needs. Pastoral care as a resource. May need psychiatric counseling. Many support groups now available Burn nurses may need help with dealing with this challenging patient population.

independence;

In the Oliguric phase of AKI, there is impaired production of WBC and RBC. Due to this, there is an increased risk for _?_ -- one of the main causes of death in AKI

infection

Sepsis clinical criteria = _?_ + change in "SOFA" greater than or equal to 2

infection

_?_ is the primary cause of death in cancer patients

infection

Biggest problem with GVHD is _?_. _?_ and _?_ are types which predominate No treatment once it is established. _?_ may be used to alleviate some symptoms. _?_ and _?_ are used for prevention

infection - bacterial and fungal infection Steroids Methotrexate; cyclosporin

Complications of peritoneal dialysis: Exit site _?_ - staph aureus and s. epidermis _?_- improper technique; note if cloudy peritoneal effluent- signs: abdominal pain and GI symptoms Outflow problems -- adhesions; constipation Back problems & hernias Bleeding - (may be a normal finding for ovulating or menstruating females) Pulmonary problems - pneumonia Protein loss-- plasma proteins, amino acids, polypeptides can pass Carbohydrate & lipid abnormalities -- ↑ triglycerides and ↑ glucose

infection; Peritonitis

Identify those at risk. Assess history and understanding of disease process. Teach importance of adherence to treatment regimen. Stress need to avoid _?_. Avoidance of stress and fatigue Rest Hygiene Assess nonspecific manifestations. Monitor laboratory data. Monitor patency of IV.

infectious people;

An intra-aortic balloon pump (IABP) is timed to inflate with _?_ and deflate with _?_

inflate during diastole deflate just before systole (creates vacuum in the aorta, causing blood to flow unimpeded -- i.e. decreased afterload)

_?_ and _?_ are 2 classes of medication which do not work as well in acidic environments (such as in metabolic acidosis)

inotropes vasopressors

Clinical Manifestations of acute Pericarditis: Progressive, freq severe, sharp chest pain (worse with _?_ and _?_) Pain may radiate to neck, arms, left shoulder Hallmark finding is a_?_ May have fever

inspiration; lying supine pericardial rub (scratching, grating, high-pitched sound)

ARDS progresses from Normal Alveoli to _?_ edema to _?_ edema

interstitial; alveolar

_?_ shunting occurs when capillary problems within the lungs allows the blood to exit prior to being oxygenated. Blood is not being oxygenated due to conditions such as _?_ or _?_

intrapulmonary shunting PNA or ARDS

With lung collapse, _?_ develops on the affected side. Alveoli are _?_, although they may still be _?_ Overall gas exchange and oxygenation are impaired. Cyanosis may be present.

intrapulmonary shunting underventilated or not ventilated, perfused.

In _?_, etologies lead to actual damage to the nephrons

intrarenal AKI

The following are _?_ etiologies which may lead to AKI: prolonged prerenal ischemia, nephrotoxic injury, acute glomerulonephritis or acute pyelonephritis, myoglobinuria

intrarenal AKI

AKI that has not caused _?_usually resolves quickly with correction of the cause.

intrarenal damage

With suction: Level of water in suction control + level of water in water seal chamber = _?_

intrathoracic pressure

What are the 11 main effects of cardiopulmonary bypass (CPB)?

intravascular fluid deficit (Hypotension) 3rd spacing (weight gain, edema) Myocardial depressing (low CO) Coagulopathy Pulmonary dysfunction Hemolysis Hyperglycemia hypokalemia hypomagnesemia neurological dysfunction hypertension

Examples of secondary SCI include _?_, _?_. _?_, and _?_

ischemia hypoxia hemorrhage edema

Hemodynamics post-esophageal cancer surgery: IV fluids: _?_ or _?_ at rate of 100-200 ml/hr for 1st 12-16 hrs after surgery Fluid boluses: major _?_ occur 1st few days postop. Hypovolemia may be problem. Avoid _?_ d/t lungs already compromised because of decreased lymph clearance r/t surgical removal of mediastinal lymphatics and nodes. (pt predisposed to interstitial pulmonary edema.) Need meticulous skin care due to 3rd spacing/ edema. Turn q2hrs & prn. Pressure relieving surface.

isotonic NS or Lactated Ringers fluid shifts fluid overload

Why is the phrenic nerve important? How do you test whether or not it is damaged?

it innervates the diaphragm; if the sniff test results in parodoxial chest movement

Examples of oncofetal antigens: 1. _?_ - colon cancer 2. _?_- epidermal growth factor; lung cancer 3. _?_ - human epidermal growth factor receptor 2 - breast cancer

kRAS EGFR HER-2

Common surgical procedures Decompression _?_by anterior cervical and thoracic approaches with fusion Posterior _?_with use of acrylic wire mesh and fusion Insertion of stabilizing rods Criteria for early surgery Cord decompression may result in ↓ secondary injury Evidence of cord compression Progressive neurologic deficit Compound fracture Bony fragments Penetrating wounds

laminectomy laminectomy

4 Signs/symptoms of Severe cases of ascending aortic dissection:

left heart failure with dyspnea, orthopnea, and pulmonary edema and possibly cardiac tamponade

_?_ is the artery that supplies the apex of the heart as well as part of the lateral wall, anterior wall and 2/3 of the septum

left main (widow maker)

Angina Syncope Exertional dyspnea are a triad that reflect _?_ failure

left ventricular

Hypertrophic Cardiomyopathy is an asymmetric _?_ hypertrophy without dilation; Impaired ventricular filling and _?_ dysfunction from left ventricular stiffness; Decreased ventricular filling and obstruction to outflow leads to _?_

left ventricular diastolic decreased C.O.

Structural heart conditions such as _?_, myocarditis and _?_ also cause sudden cardiac death (SCD)

left ventricular hypertrophy hypertrophic cardiomyopathy

Describe the "fluid restriction" portion of SIADH treatment:

less than 800-1000 ml/day (but could be as low as 500 ml/day)

What is the PAWP in ALI and ARDS?

less than or equal to 18 mm Hg or no clinical evidence of heart failure

Sometimes subtypes of _?_can't be identified - usually don't respond to treatment, poor prognosis

leukemia

Treatment for_?_: therapeutic agents used varies. Combination therapy is mainstay. 3 purposes for multiple drug therapy: Decrease drug resistance, minimize drug toxicity, interrupt cell growth at multiple points in cell cycle

leukemias

Labs for acute pericarditis: _?_ , elevated CRP & ESR, may have elevated _?_ levels, _?_ w/fluid to lab

leukocytosis troponin pericardiocentesis

In SCI, respiratory complications closely correspond to the _?_

level of injury

Treatment goal of aortic aneurysm:

limit progression

The pancreas will release _?_ and _?_ into the blood when injured or inflamed. Both levels rise within _?_ to _?_ hours. In 3 days, _?_ levels return to normal, but _?_ stays elevated for up to 7 to 10 days, helping in the late diagnosis of pancreatitis

lipase and amylase 2 to 12 hours. 3 days, amylase levels return to normal, but lipase stays elevated

What 3 enzymes do the following meds contain? Creon, Zenpep, Pancrease

lipase, amylase, trypsin

List 3 target organs for GVHD phenomenon and the associated s/s

liver (elevated liver enzymes to hepatic coma) GI tract (diarrhea, abdominal pain, GI bleeding, malabsorption) skin (pruritic or painful maculopapular rash - involves palms and soles of feet).

Calcium in CKD is _?_ (high or low?) 4 S/S are:

low tingling around mouth TIngling of fingers tetany convulsions

After an aneurysm repair, if BP is too low, _?_ could occur. If BP is too high, _?_ could occur

low - thrombus formation high - anastomosis tear

In addition to trying to correct the underlying cause, fluids (if necessary), other medications and patient teaching, what are 2 other possible treatments used in nephrogenic DI?

low sodium diet thiazide diuretics (HydroDiuril)

IN CKD, Na can be _?_, _?_ or _?_

low, normal or high

Associated manifestations of _?_ include: presence of facial burns or singed nares hairs or facial hair, dypsnea carbonaceous sputum wheezing hoarseness altered mental status.

lower airway injury

There should be a high degree of suspicion of _?_ if patient was trapped in an enclosed space or clothing caught on fire,

lower airway injury

Nursing assessment for Respiratory failure and ARDS: History of _?_ Exposures to lung toxins _?_ Related hospitalizations _?_ trauma Extreme obesity Use of O2, inhalers, nebulizers, immunosuppressant therapy Previous intubation Thoracic or abdominal surgery

lung disease Tobacco use Spinal cord

If the vegetation from IE travels through the right side of the heart, where might it manifest?

lungs

Name some contraindications to renal transplant: Are HIV and Hep B/Hep C included in this list?

lupus, untreated CAD, chronic infections, malignancies, unresolved psychosocial issues; NO

7 Risks associated with development of atubular necrosis while in the hospital:

major surgery shock sepsis blood transfusion reaction muscle injury from trauma prolonged hypotension nephrotoxic agents

Complications of _?_ chest pain heart palpatations tachycardia f/e imbalances Hypo or hypernatremia Hyperkalemia acidosis seizures renal failure

mannitol

It is necessary to do _?_ when measuring a patient's abdominal girth

mark on ptnt abdomen at the top and bottom of the tape measure both anterior and posterior to make sure all nurses measure at the same level

The location of the obstruction determines the extent of fluid, electrolyte, and acid-base imbalances. High obstruction (in pylorus) _?_ secondary to loss of hydrochloric acid from the stomach through -?_ or _?_

metabolic alkalosis vomiting or NG tube suctioning.

After SCI, nasogastric suctioning may lead to _?_; while decreased tissue perfusion may lead to _?_

metabolic alkalosis; acidosis

Nasogastric suctioning may lead to _?_ ↓ Tissue perfusion may lead to _?_ Monitor electrolyte levels until suctioning is discontinued and normal diet is resumed Loss of body weight is common Nutritional needs much greater than expected for immobilized person Positive nitrogen and high-protein diet Prevents skin breakdown and infection Decreases rate of muscle atrophy

metabolic alkalosis; acidosis

Diagnostic studies for CKD H&P Urinalysis: for presence of 5 things: _____ Renal ultrasound, CT scan, renal biopsy CMP CBC Creatinine Clearance GFR KUB - x-ray that looks at _?_ IVP - intravenous pylogram renal angiogram

microalbumin, RBCs, WBCs, protein, casts & glucose KUB - kidney, ureter, bladder

_?_ are often called plant alkaloids; inhibits cell division Examples are: paclitaxel (Taxol), docetaxel (Taxotere), vinblastine (Velban), vincristine (Oncovin), eribulin (Halaven)

mitotic inhibitors

_?_ are a type of medication used short-term to reduce the risk of acute rejection

monoclonal antibodies

The American Spinal Injury Association (ASIA) Impairment Scale combines assessments of _?_ and _?_ function to determine neurological level and completeness of injury

motor sensory

In the spinal cord, the ventral root carries _?_ information to _?_ and _?_

motor; muscles and glands

Nasogastric Tube post esophageal cancer surgery: Do not _?_, _?_ or _?_ NG tube in these pts. If pulled out accidently/dislodged, do not try to reinsert. Call surgeon immediately. Notify surgeon immediately if tube not functioning/draining properly. Low intermittent suction

move, manipulate, or irrigate

Bence-Jones protein - up production & secretion of free light-chain proteins from myeloma cell seen, detected in urine is an indicator of _?_

multiple myeloma

Late S/S of _?_ are: bone pain, fractures, plasma cell tumors. This is not a bone cancer, but an abnormality in which parts of bone are destroyed.

multiple myeloma

Treatment of _?_ may include: chemo with corticosteroids 3 most common: MPT (melphalan, prednisone, thalidomide); MPV (melphalan, prednisone, bortezomib [velcade]; MPR (melphalan, prednisone, lenalidomide [revlimid];

multiple myeloma

clinical manifestations of IE Nonspecific Can involve _?_ _?_ occurs in 90% of patients Chills Weakness Malaise Fatigue Anorexia _?_ in most patients _?_ in up to 80% with aortic valve endocarditis If have embolism, see manifestations secondary to where emboli traveled to

multiple organ systems; Low grade fever Murmur Heart failure

Development of cancer - usually an orderly process with stages... 1. Initiation - first stage; requires _?_

mutation

In Guillain Barre Syndrome, immune system destroys _?_ Affects ability to feel textures, heat, & pain Brain gets altered signals Patients feel _?_

myelin sheath surrounding axons of peripheral nerves; tingling/crawling skin feeling

Rhabdomyolysis Leads to _?_ which puts the kidneys at risk for _?_

myoglobinuria (Kidneys @ risk for AKI)

List 3 s/e that are common to external radiation, brachytherapy and chemo

n/v anorexia fatigue

A _?_ crossmatch indicates that no preformed antibodies are present and it is safe to proceed with the transplantation.

negative

Patient is likely to be in shock from hypovolemia. Full thickness and deep partial-thickness burns are initially anesthetic because the _________ __________ have been destroyed. Partial thickness burns have blisters filled with fluid and protein. Superficial to moderate partial thickness burns are very ______________. Bowel sounds are _______ or ________. Shivering may occur due to heat loss, anxiety or pain. LOC

nerve endings; painful; decreased or absent

Diagnosis of DI: History - If dehydrated, then: Water deprivation (restriction) test confirms _?_ .

neurogenic (central) DI

Due to SCI, loss of voluntary neurologic control over the bowel results in _?_

neurogenic bowel

Organ Donor Criteria: Any pt. who suffers a devastating & irreversible _?_ insult or injury can become an organ donor The pt. must be maintained on a ventilator with intact blood pressure and HR It must be determined that there is no hope for recovery & that all medical & surgical intervention has been exhausted or is futile

neurologic

Brain Dead Donors: Patient must lack all _?_ function including _?_ reflexes

neurologic brain stem

What are the Chest X-ray findings in ALI and ARDS?

new bilateral interstitial and alveolar infiltrates

describe the chest x-ray of a patient with ARDS

new, bilateral, diffuse, homogeneous pulmonary infiltrates without cardiac failure, fluid overload, chest infection or chronic lung disease

Phosphodiesterase (Type 5) enzyme inhibitors are contraindicated in patients taking _?_ because it may cause _?_

nitro; refractory hypotension

What should ascites fluid look like if there is no infection? If there is infection?

no infection = light straw yellow, clear infection= cloudy

What type of isolation is necessary for a patient with neutropenia?

no reverse isolation is necessary unless visitor/staff is sick with respiratory infection

Does a small pneumothorax always require a chest tube? If not, what would they do for it?

no; serial x-rays and watch patient for tolerance

Ileus/paralytic ileus are _?_ causes of bowel obstruction

non-mechanical

You dim the lights around Mrs. Thorne and examine her eyes, comparing pupil size bilaterally. Pupil size is similar at 4 mm. This finding is:

normal

Syndrome of Inappropriate ADH (SIADH): Occurs when ADH is released despite _?_ or _?_

normal or low plasma osmolarity

Carcinoembryonic antigen (CEA) is useful as tumor markers to monitor effectiveness of therapy or indicate tumor recurrence 1. If elevated post-surgery, _?_ 2. If elevated post-chemo,_?_

not all tumor has been removed return/spread of cancer

What should you do if while monitoring your patient, their GCS score falls below 8?

notify the MD

GI Perforation Goals: Relief of _?_ Return of normal bowel function within _?_ days Minimal to no discomfort No _?_ Normal fluid and electrolyte balance Normal acid-base state

obstruction 3 days emesis

In _?_ jaundice, unconjugated bilirubin is somewhat high conjugated bilirubin is moderately high urine bilirubin is high urine urobilinogen is low stool urobilnogen is low

obstructive

In _?_ shock, the patient has decreased CO and decreased filling of right ventricle

obstructive

_?_ jaundice: Caused by decreased or obstructed flow of bile through the liver or biliary duct system

obstructive

In _?_ shock, the patient will experience: Decreased CO Increased afterload Variable left ventricular filling pressures JVD Pulsus paradoxus

obstructive shock

Most common location of aneurysm:

occur in the abdominal aorta

Treatment if esophageal or gastric varcies do begin to bleed: Manage airway and give blood products Drugs: _?_- constricts splanchnic arterial bed _?_ is man-made form of ADH - from posterior pituitary - vasoconstrictor _?_ (vaso-dilator - used in combination with vasopressin); _?_ (propranolol) reduces portal venous pressure

octreotide (Sandostatin) Pitressin (vasopressin) nitroglycerin Beta Blockers

What is the mnemonic to remember the cranial nerves?

oh, oh, oh, to touch and feel awesome green velvet, ah heaven

_?_ is when air is entering the intrapleural space through the chest wall

open pneumothorax

Endothelin Receptor Antagonists are given _?_ and are indicated for patients with class _?_ to _?_ symptoms. _?_ is a medication in this group that is hepatotoxic

orally; II thru IV Bosentan

_?_ - movement of fluid from area of lesser concentration to greater concentration of solutes (glucose is added to the dialysate and creates osmotic gradient across membrane)

osmosis

Chest Surgery Post-OP care: Optimize _?_ Positioning - _?_ lung down (good or bad?) _?_ management Increase activity and ROM Infection prevention and monitoring Nutritional status Chest tube system maintenance

oxygenation and ventilation good lung down Pain

Planning for IICP Overall goals: Maintain a patent airway Maintain PO2 _?_ Maintain PCO2 _?_ ICP within normal limits Normal fluid and electrolyte balance Normal nutritional balance No complications secondary to immobility and decreased LOC

pO2 > or = to 100 PCO2 between 35-45

With fractured ribs, always treat _?_ to help avoid respiratory failure

pain

2 Signs/symptoms of Acute descending aortic dissection:

pain in their back, abdomen, or legs; describe as "sharp", "worst ever", "tearing", "ripping" or "stabbing"

After cancer surgery, _?_ reduces mortality and morbidity by decreasing cardiopulmonary complications, shorter LOS Method: IV or epidural meds as NPO for 5-7 days.

pain management

Manifestations of hyperparthyroidism: 6 GI:

pain, anorexia, N/V, constipation, peptic ulcers, pancreatitis

Manifestations of hyperparthyroidism: 5 Musculoskeletal:

pain, weakness, osteoporosis, fractures, decreased muscle tone,

_?_ is the fourth leading cause of cancer related deaths in the US and has a poor 5 year survival rate. (ACS) Etiology remains unknown _?_ procedure may be part of treatment

pancreatic cancer Whipple procedure

The following meds are __?__: Creon, Zenpep, Pancrease

pancreatic enzyme replacement

Flail segment moves independently in response to intrathoracic pressure - __?_ breathing Usually result of _?_ & underlying lung may or may not be injured; may require extensive ventilator use!

paradoxical breathing; blunt trauma


Kaugnay na mga set ng pag-aaral

Eco 152- principles of microeconomics

View Set

Internal Combustion Engine Theory & Performance

View Set

Human Communication Test 3 (7, 8, 9)

View Set

Перевірка знання Закону України "Про освіту"

View Set